Está en la página 1de 98

Priscilla Alma Jose v. Ramon C. Javellana, et al., G.R. No.

158239, January 25, 2012

FACTS:

Margarita sold for consideration of 160K to Javellana by deed of conditional sale two parcels of land in which
they agreed that Javellana would pay 80K upon the execution of the deed and the balance of 80K upon the
registration of the land (Registered within the reasonable time); that should Margarita become incapacitated, her
son, Juvenal and her daughter, Priscilla, would receive the balance and proceed with the application. After
Margarita died and with Juvenal having predeceased Margarita, the said undertaking was passed upon Priscilla
being the sole surviving heir. However, Priscilla did not comply with the said undertaking. Thus, it prompted
Javellana to file an action for specific performance, injunction, and damages against Priscilla and to order
Priscilla to institute the registration proceedings and execute a final deed of sale. Priscilla filed a motion to
dismiss on the grounds that the complaint was barred by prescription and that the same did not state a cause of
action.

RTC – it denied the motion to dismiss. However, upon the motion for reconsideration, the RTC granted the said
motion. Thus, Javellana moved for reconsideration but was denied. Accordingly, Javellana filed a notice of
appeal. Priscilla countered that the RTC order was not appealable; that the appeal was not perfected on time;
and that Javellana was guilty of forum shopping. It appears that pending the appeal, Javellana also filed a
petition for certiorari in the CA to assail the June 24, 1999 and June 21, 2000 orders dismissing his complaint.

CA – dismissed the petition for certiorari. As to the notice of appeal, it reversed and set aside the RTC remanded
the said court ‘for further proceeding in accordance with law’. It also denied the motion for reconsideration filed
by Priscilla.

ISSUE:

WON THE RTC’S DECISION DENYING THE MOTION FOR RECONSIDERATION OF THE
ORDER OF DISMISSAL A FINAL ORDER AND THUS, APPEALABLE.

HELD: YES.

The denial of Javellana’s motion for reconsideration left nothing more to be done by the RTC because it
confirmed the dismissal of Civil Case No. 79-M-97. It was clearly a final order, not an interlocutory one. A
distinction between a final order and interlocutory order was provided in the said case. The former disposes of
the subject matter in its entirety or terminates a particular proceeding or action, leaving nothing more to be done
except to enforce by execution what the court has determined, but the latter does not completely dispose of the
case but leaves something else to be decided upon. An interlocutory order deals with preliminary matters and
the trial on the merits is yet to be held and the judgment rendered. The test to ascertain whether or not an order
or a judgment is interlocutory or final is: does the order or judgment leave something to be done in the trial
court with respect to the merits of the case? If it does, the order or judgment is interlocutory; otherwise, it is
final.

VINLUAN,  ALEXANDER  MARA  J.  2C  UST  LAW  


FREDESVINDO S. ALVERO v M.L. DE LA ROSA, et.al
G.R. No. L-286, March 29, 1946

FACTS:

Respondent filed a complaint in the CFI – Manila, against the petitioners, alleging two causes of action – (1) to
declare in force the contract of sale between Victoriano and Villarica, of 2 parcels of land with an area of
480sqm. for the sum of 100K and (2) to declare the said sale null and void. The petitioner admitted, in her
answer to the complaint, that he sold the said lands to Alvero but she cannot remember and offered to
repurchase the said lands but Alvero refused to accept the offer. Alvero, in the answer for the said complaint, he
denied the allegations and claimed exclusive ownership of the land in question.

RTC – rendered a decision which declared that the said two parcels of land had been sold by Villarica to
Victoriano, for the sum of 6K on the condition that the purchaser should make a down payment of P1,700 and a
monthly payment of P76.86 in equal monthly installments; that Victoriano continued paying such until was time
wherein he had an agreement with Villarica that the said payments shall be suspended until the restoration of
peace. Victoriano took possession of the said land and made improvements therein. Villarica had already
forgotten the said sale, Victoriano sold the same for 100K to Alvero. Alvero presented the deed of sale to the
register of deeds of Manila and afterwards took possession of the said land. However, it was found out that
there was an agreement between Victoriano and Villarica that upon the failure of Villarica to make payments
for 3 successive monthly installments, the vendor shall be free to sell the property again, forfeiting the payments
made. Also, despite the fact that Victoriano had in his hands the said deed, he failed to transfer the title to his
name. Thus, RTC rendered a decision in favor of Victoriano.

Alvero was notified by the said decision thus he filed a petition for reconsideration and new trial, which was
denied. Alvero filed his notice of appeal and record on appeal simultaneously in the lower court, without filing
the P60-appeal bond. Victoriano filed a petition to dismiss the appeal, and at the same time, asked for the
execution of the judgment. Alvero filed an opposition to said motion to dismiss, alleging that on the very same
day, January 15, 1946, said appeal bond for P60 had been actually filed, and allege as an excuse, for not filing
the said appeal bond, in due time, the illness of his lawyer's wife, who died. The RTC ordered the dismissal of
the appeal, declaring that, although the notice of appeal and record on appeal had been filed in due time, the
P60-appeal bond was filed too late.

CA – Alvero’s appeal was dimissed and a motion for reconsideration was subsequently denied.

ISSUE:

WON the petition is defective in form as well as in substance.

HELD: YES

The period for perfecting petitioner’s appeal commenced from November 28, 1945, when he was notified of the
judgment rendered in the case, and expired on December 28, 1945; and, therefore, his notice of appeal and
record on appeal filed on January 8, 1946, were filed out of time, and much more so his appeal bond, which was
only filed on January 15, 1946. Failure to perfect the appeal, within the time prescribed by the rules of court,
will cause the judgment to become final, and the certification of the record on appeal thereafter, cannot restore
the jurisdiction, which has been lost.

VINLUAN,  ALEXANDER  MARA  J.  2C  UST  LAW  


DOMINADOR B. BUSTOS vs. ANTONIO G. LUCERO
G.R. No. L-2068, October 20, 1948

FACTS:
Dominador Bustos, an accused in a criminal case, filed a motion with the CFI Pampanga after he had been tried
in the said trial court, praying that the record of the case be remanded to the justice of the peace court of
Masantol, the court of origin, in order that the Bustos might cross-examine the complainant and her witnesses in
connection with their testimony, on the strength of which warrant was issued for the arrest of the accused.
Bustos, assisted by counsel, appeared at the preliminary investigation. In that investigation, the justice of the
peace informed him of the charges and asked him if he pleaded guilty or not guilty, upon which he entered the
plea of not guilty. Then his counsel moved that the complainant present her evidence so that she and her
witnesses could be examined and cross-examined in the manner and form provided by law. The fiscal and the
private prosecutor objected, invoking section 11 of rule 108, which provides that “Evidence - which is the the
mode and manner of proving the competent facts and circumstances on which a party relies to establish the fact
in dispute in judicial proceedings”

RTC – the objection was sustained which denied the motion for reconsideration of the accused in which,
Bustos’ counsel announced his right to present evidence.

ISSUE:

WON SEC. 11 , RULE 108 OF THE RULES OF COURT INFRINGES SEC. 13, ARTICLE VIII, OF
THE CONSTITUTION WHICH DEALS WITH SUBSTANTIVE MATTERS AND IMPAIRS
SUBSTANTIVE RIGHTS.

HELD: NO. Section 11, Rule 108 is an adjective law and not a substantive law or substantive right.

A distinction between the substantive law and remedial law was provided in this case. Section 11 of Rule 108
denied to the defendant the right to cross-examine witnesses in a preliminary investigation, his right to present
his witnesses remains unaffected, and his constitutional right to be informed of the charges against him both at
such investigation and at the trial is unchanged. It is fundamentally a procedural law. The Supreme Court that
section 11 of Rule 108 does not curtail the sound discretion of the justice of the peace on the matter. Said
section defines the bounds of the defendant's right in the preliminary investigation. A divided court rendered the
foregoing decision. The minority went farther than the majority and denied even any discretion on the part of
the justice of the peace or judge holding the preliminary investigation to compel the complainant and his
witnesses to testify anew. Upon the foregoing considerations, the present petition is dismissed with costs against
the petitioner.

VINLUAN,  ALEXANDER  MARA  J.  2C  UST  LAW  


PANAY RAILWAYS INC., v. HEVA MANAGEMENT and DEVELOPMENT CORPORATION,
PAMPLONA AGRO-INDUSTRIAL CORPORATION, and SPOUSES CANDELARIA DAYOT and
EDMUNDO DAYOT, G. R. No. 154061, January 25, 2012

FACTS:

Panay Railways (PRI) entered into a Real Estate Mortgage (REM) with Traders Royal Bank (Traders) for a loan
amounting to P20M. The REM covers several parcels of land in which the PRI excluded certain portions
wherein it was sold Shell Co., a road, and a squatter area. The PRI failed to pay its obligation thus the properties
were extra judicially foreclosed by the Traders, and subsequently a certificate of sale was issued in favor of
traders as being the highest bidder and purchaser. Thus, the sold lot was registered and transferred the title in its
name on the basis of the Deed of Sale and an affidavit of consolidation.

RTC- Thereafter, Traders filed a Petition for Writ of Possession against the PRI. During the proceedings of
such, PRI filed a Manifestation and Motion to Withdraw Motion for Suspension of the Petition for the issuance
of a writ of possession. It was only in 1994 that petitioner realized that the extrajudicial foreclosure included
some excluded properties in the mortgage contract. Thus, it filed a Complaint for Partial Annulment of Contract
to Sell and Deed of Absolute Sale with Addendum; Cancellation of Title; and Declaration of Ownership of Real
Property with Reconveyance plus Damages.

Meanwhile, the respondents filed their respective Motions to Dismiss based on the following grounds - (1)
petitioner had no legal capacity to sue; (2) there was a waiver, an abandonment and an extinguishment of
petitioners claim or demand; (3) petitioner failed to state a cause of action; and (4) an indispensable party,
namely Traders, was not impleaded. The said motion was granted because the Manifestation and Motion filed
by petitioner was a judicial admission of TRBs ownership of the disputed properties. Petitioner filed a Notice of
Appeal without paying the necessary docket fees. Immediately thereafter, respondents filed a Motion to Dismiss
Appeal on the ground of nonpayment of docket fees. Petitioner alleged that its counsel was not yet familiar with
the revisions of the Rules of Court. Furthermore, it contended that the requirement for the payment of docket
fees was not mandatory. It therefore asked the RTC for a liberal interpretation of the procedural rules on
appeals. RTC dismissed the appeal citing Sec. 4 of Rule 14. Petitioner thereafter moved for a reconsideration of
the Order alleging that the trial court lost jurisdiction over the case after the former had filed the Notice of
Appeal. Petitioner also alleged that the court erred in failing to relax procedural rules for the sake of substantial
justice. But such motion was also denied.

CA - Petition for Certiorari and Mandamus under Rule 65 alleging that the RTC had no jurisdiction to dismiss
the Notice of Appeal, and that the trial court had acted with grave abuse of discretion when it strictly applied
procedural rules. It held that while the failure of petitioner to pay the docket and other lawful fees within the
reglementary period was a ground for the dismissal of the appeal the jurisdiction to do so belonged to the CA
and not the trial court. Thus, a motion for reconsideration was filed but such was also denied.

ISSUE:
WON THE CA ERRED IN SUSTAINING THE RTC DISMISSAL OF THE NOTICE OF APPEAL.

HELD: NO.
The argument that the CA had the exclusive jurisdiction to dismiss the appeal has no merit. When this Court
accordingly amended Sec. 13 of Rule 41 through A.M. No. 00-2-10-SC, the RTCs dismissal of the action may
be considered to have had the imprimatur of the Court. Thus, the CA committed no reversible error when it
sustained the dismissal of the appeal, taking note of its directive on the matter prior to the promulgation of its
Decision. It is also held that the payment of the full amount of the docket fees is an indispensable step for the
perfection of an appeal. The Court acquires jurisdiction over any case only upon the payment of the prescribed
docket fees.
VINLUAN,  ALEXANDER  MARA  J.  2C  UST  LAW  
Felix Martos, et al. v. New San Jose Builders, Inc., G.R. No. 192650
FACTS:

The respondent (New San Jose Builders, Inc.) is a domestic corporation duly organized and existing under the
laws of the Philippines and is engaged in the construction of road, bridges, buildings, and low cost houses
primarily for the government. One of the projects of petitioner is the San Jose Plains Project (SJPP), located in
Montalban, Rizal. SJPP, which is also known as the "Erap City" calls for the construction of low cost housing,
which are being turned over to the National Housing Authority (NHA) to be awarded to deserving poor
families. Private respondents alleged that, on various dates, petitioner hired them on different positions.
Petitioner was constrained to slow down and suspend most of the works on the SJPP project due to lack of
funds of the NHA. Thus, the workers were informed that many of them [would] be laid off and the rest would
be reassigned to other projects. Some of the employees were laid off and some (petitioners) were retained and
offered new positions but they refused to continue to work. Thereafter, 3 complaints for Illegal Dismissal and
for money claims were filed before the NLRC against the respondent and Acuzar. However, the respondent
denied such claims by the employees. The 3 complaints were consolidated and was brought before the LA. LA
rendered a decision in favor of the petitioners, which held that they were illegally dismissed and entitled to
monetary benefits.

NLRC – both parties appealed before the NLRC. Petitioners appealed that part which dismissed all the
complaints, without prejudice, except that of Martos. On the other hand, respondent appealed that part which
held that Martos was its regular employee and that he was illegally dismissed. NLRC resolved the appeal by
dismissing the one filed by respondent and partially granting that of the other petitioners.

CA - a petition for certiorari under Rule 65; rendered a decision reversing and setting aside the decision and
resolution of the NLRC and reinstating the decision of the LA. The CA explained that the NLRC committed
grave abuse of discretion in reviving the complaints of petitioners despite their failure to verify the same. The
CA also held that the NLRC gravely abused its discretion when it took cognizance of petitioners’ appeal
because Rule 41, Section 1(h) of the 1997 Rules of Civil Procedure, as amended, which is suppletory, provides
that no appeal may be taken from an order dismissing an action without prejudice.

ISSUE:

WON THE CA WAS CORRECT IN DISMISSING THE COMPLAINANTS FILED BY THOS


PETITIONERS WHO FAILED TO VERIFY THEIR POSITION PAPERS

HELD: YES.
Sections 4 and 5 of Rule 7 of the 1997 Rules of Civil Procedure provide: SEC. 4. Verification. – Except when
otherwise specifically required by law or rule, pleadings need not be under oath, verified or accompanied by
affidavit.

A pleading is verified by an affidavit that the affiant has read the pleadings and that the allegations therein are
true and correct of his personal knowledge or based on authentic records. A pleading required to be verified
which contains a verification based on "information and belief" or upon "knowledge, information and belief" or
lacks a proper verification, shall be treated as an unsigned pleading. The verification requirement is significant,
as it is intended to secure an assurance that the allegations in the pleading are true and correct and not the
product of the imagination or a matter of speculation, and that the pleading is filed in good faith. Verification is
deemed substantially complied with when, as in this case, one who has ample knowledge to swear to the truth
of the allegations in the complaint or petition signs the verification, and when matters alleged in the petition
have been made in good faith or are true and correct.
VINLUAN,  ALEXANDER  MARA  J.  2C  UST  LAW  
Maria Consolacion Rivera-Pascual v.Sps. Marilyn Lim and George Lim and the Registry of Deeds of
Valenzuela City, G.R. No. 191837, September 19, 2012

FACTS:

A petition was filed before the Office of the Regional Agrarian Reform (RARAD) to be recognized as a tenant
of a property located in Valenzuela City against Deato. During the pendency of the said petition, Deato sold the
property to Sps. Lim which was later in registered. This prompted the petitioner to file a motion to implead the
Sps. Lim. The Regional Adjudicator (RA) granted the petition. Thus, the petitioner filed a motion for execution
to which a writ of execution was issued by the RA. Petitioner filed a petition against the private respondents and
the Registrar of Deeds praying for the issuance of an order directing Spouses Lim to accept the amount of
P10million which she undertook to tender, declare the property redeemed and cancel the TCT.

RARAD – held that the property is lawfully redeemed and ordered the respondent to accept the amount
consigned with the DARAB, execute a deed of redemption and directed the RD to cancel the TCT.
DARAB – reversed the decision of the RARAD.

CA – petition for review under Rule 43; did not give due course to the petition due to the following technical
grounds: a) failure of counsel to indicate in the petition his MCLE Certificate of Compliance or Exemption
Number and b) the jurat of Consolacion’s verification and certification against non-forum-shopping failed to
indicate any competent evidence of Consolacion’s identity apart from her community tax certificate.

ISSUE:

WON THE PETITION SHOULD BE DENIED DUE TO THE UNEXPLAINED FAILURE TO


COMPLY WITH BASIC PROCEDURAL REQUIREMENTS OF THE RULES OF COURT.

HELD: YES.

Petitioner and her counsel claimed inadvertence and negligence but they did not explain the circumstances
thereof. Absent valid and compelling reasons, the requested leniency and liberality in the observance of
procedural rules appears to be an afterthought, hence, cannot be granted. The CA saw no compelling need
meriting the relaxation of the rules. Neither did the Court see any. This Court will not condone a cavalier
attitude towards procedural rules. It is the duty of every member of the bar to comply with these rules. They
are not at liberty to seek exceptions should they fail to observe these rules and rationalize their omission by
harking on liberal construction. While it is the negligence of petitioner’s counsel that led to this unfortunate
result, such binds her.

VINLUAN,  ALEXANDER  MARA  J.  2C  UST  LAW  


Commissioner of Internal Revenue vs Migrant Pagbilao Corporation, G.R. No. 159593, October 12, 2006

FACTS:

MPC is a domestic corporation engaged in the business of power generation and subsequent sale thereof. It is
registered with the BIR as VAT registered entity. For the period of April 1, 1996 to December 31, 1996, MPC
seasonably filed its quarterly VAT returns reflecting an accumulated input taxes in the amount of P 39,
330,500.00. These taxes were allegedly paid my MPC to the suppliers of capital goods and services. MPC filed
an application for a tax refund of the unutilized VAT paid on capital goods. MPC did not wait for an answer
from the BIR and filed a petition for review in order to toll the running period for claiming the refund.
The BIR Commissioner raised as a defense that the application for refund is still pending and therefore
premature. The Commissioner also argued that MPC must produce evidence to prove that it is entitled to the
refund, as tax refunds are construed strictly against the tax payer. While the case was pending, Revenue Officers
investigated MPC’s application and recommended that MPC’s input taxes should be reduced by P49, 616.40 for
unapplied input taxes on capital goods. A third party audit was also conducted and found that the input taxes
only amounted to P28, 745,502. The Court of Tax Appeals rule in favor of MPC and granted the tax refund. The
Court of Appeals denied the BIR’s petition for review, declaring that the BIR cannot validly change its theory
on the case on appeal.

ISSUE:

WON THE OBSERVANCE OF PROCEDURAL RULES SHOULD BE RELAXED.


WON MPC IS A PUBLIC UTILITY AND IS EXEMPT FROM PAYING VAT AND IS NOT
ENTITLED TO A REFUND.

HELD:

1. NO, the general rule is that a party cannot change his theory of the case on appeal. It was only after the CTA
ruled against the BIR, that the latter filed his petition for review before the CA, and for the first time averred
that the MPC was a public utility and does not pay VAT, thereby disqualifying it from claiming a refund.The
settled rule is that a party cannot change his theory of the case or his cause of action on appeal. It affirms
“courts of justice have no jurisdiction or power to decide a question not in issue. Such a rule has been expressly
adopted in Rule 44, Section 15 of the 1997 Rules of Civil Procedure, which provides –

SEC. 15. Questions that may be raised on appeal. – Whether or not the appellant has filed a motion for new
trial in the court below, he may include in his assignment of errors any question of law or fact that has been
raised in the court below and which is within the issues framed by the parties.

2. NO. The CTA found that MPC is registered as a VAT-taxpayer, as evidenced by its Certificate of
Registration, issued by the BIR Revenue District Office (RDO) No. 60, on 26 January 1996. The BIR
Commissioner does not contest this fact, and does not offer any explanation as to why the BIR RDO had
approved the registration of MPC as a VAT-taxpayer Well-settled principle in this jurisdiction is that this Court
is bound by the findings of fact of the CTA. Only errors of law, and not rulings on the weight of evidence, are
reviewable by this Court. Findings of fact of the CTA are not to be disturbed unless clearly shown to be
unsupported by substantial evidence.

VINLUAN,  ALEXANDER  MARA  J.  2C  UST  LAW  


SM Land, Inc., et al. v. City of Manila, et al., G.R. No. 197151
FACTS:

In accordance with the Revenue Code of Manila, the respondent assessed the petitioners, increased rates of
business taxes for 2003 and the first to third quarters of 2004. Petitioners paid the additional taxes under protest.
Subsequently, petitioners and their sister companies claimed with City Treasurer of Manila a credit or refund of
the increased business taxes, which they paid for the period abovementioned. However, the City Treasurer
denied their claim.

RTC – petitioners filed a Complaint for Refund and/or issuance of Tax Credit of Taxes Illegally Collected. It
rendered a summary judgment in favor of the petitioners. Tax Ordinance Nos. 7988 and 8011, which were the
bases of the City of Manila in imposing the assailed additional business taxes on petitioners and their co-
plaintiffs, had already been declared null and void by this Court in the case of Coca-Cola Bottlers Philippines,
Inc. v. City of Manila.

CTA – sustained the ruling of the RTC that Ordinance Nos. 7988 and 8011 are null and void. It held that herein
petitioners' claims for tax refund should be denied because of their failure to comply with the provisions of the
Rules of Court requiring verification and submission of a certificate of non-forum shopping.
Aggrieved, petitioners filed a petition for review with the CTA En Banc. The CTA En Banc rendered its
assailed Decision affirming in toto the judgment of the CTA Second Division.

ISSUE:
WON SEC. 11, RA 1125, AS AMENDED BY RA 9282, CLEARLY DID NOT INTEND FOR THE 30-
DAY PERIOD TO APPEAL DECISIONS OF THE RTC TO THE CTA TO BE EXTENDIBLE.

HELD: YES.

The period to appeal the decision or ruling of the RTC to the CTA via a Petition for Review is specifically
governed by Section 11 of Republic Act No. 9282, and Section 3 (a), Rule 8 of the Revised Rules of the CTA.
It is crystal clear from the afore-quoted provisions that to appeal an adverse decision or ruling of the RTC to the
CTA, the taxpayer must file a Petition for Review with the CTA within 30 days from receipt of said adverse
decision or ruling of the RTC. It is also true that the same provisions are silent as to whether such 30-day period
can be extended or not. However, Section 11 of Republic Act No. 9282 does state that the Petition for Review
shall be filed with the CTA following the procedure analogous to Rule 42 of the Revised Rules of Court.

Following by analogy, Section 1, Rule 42 of the Revised Rules of Civil Procedure, the 30-day original period
for filing a Petition for Review with the CTA under Section 11 of Republic Act No. 9282, as implemented by
Section 3 (a), Rule 8 of the Revised Rules of the CTA, may be extended for a period of 15 days. No further
extension shall be allowed thereafter, except only for the most compelling reasons, in which case the extended
period shall not exceed 15 days.

Court has held that rules of procedure are established to secure substantial justice. Being instruments for the
speedy and efficient administration of justice, they must be used to achieve such end, not to derail it. In
particular, when a strict and literal application of the rules on non-forum shopping and verification will result in
a patent denial of substantial justice, these may be liberally construed.

VINLUAN,  ALEXANDER  MARA  J.  2C  UST  LAW  


Pinga v. Heirs of Santiago, G.R. No. 170354, June 30, 2006
FACTS:

The petitioner, together with Saavedra was named as defendants in an injunction case filed by respondent which
alleged that Pinga and Saavedra had been unlawlly entering the coco lands of the respondents cutting trees and
bamboo grass and gathering fruits of the trees therein. Pinga disputed the ownership over the property by the
respondent and claimed his father who was already in possession thereof since 1930s owned the same.
Furthermore, Pinga alleged that as far back as 1968, there was already an order ejecting the respondents from
the property after the heirs of Edmundo Pinga filed a complaint for forcible entry.

RTC - trial of the case had not yet been completed due to the several faults of the Respondent (as plaintiff), as
when they failed to present evidence. RTC ordered the dismissal of the action on the ground of Plaintiff's fault
to prosecute the case for unreasonable length of time. At the same time, RTC also allowed Pinga to present his
evidence ex parte with respect to his counterclaim. The Respondents filed a Motion for Reconsideration praying
that the entire action be dismissed and Petitioner be disallowed from presenting evidence ex-parte. They argued
that the order of the RTC in allowing Pinga to present evidence ex parte would circumvent the principle laid
down in established jurisprudence, which hold that a counterclaim cannot exist independently from the principal
complaint and be adjudicated separately. Said motion was granted because Pinga filed no opposition.

Notably, the respondents filed an Opposition to Defendants’ Urgent Motion for Reconsideration, wherein they
argued that the prevailing jurisprudential rule is that "compulsory counterclaims cannot be adjudicated
independently of plaintiff’s cause of action," and " the dismissal of the complaint carries with it the dismissal of
the compulsory counterclaims." Aggrieved, Pinga elevated the case to the Supreme Court by way of petition for
review under Rule 45 on a pure question of law regarding the total dismissal of the case.

ISSUE: WON THE DISMISSAL OF THE COMPLAINT NECESSARILY

HELD: NO

The dismissal of the complaint does not necessarily carries with it the dismissal of the compulsory
counterclaim. Under Sec. 3 Rule 17 of the 1997 Rules of Civil Procedure:

SEC. 3. Dismissal due to fault of plaintiff.—If, for no justifiable cause, the plaintiff fails to appear on the date
of the presentation of his evidence in chief on the complaint, or to prosecute his action for an unreasonable
length of time, or to comply with these Rules or any order of the court, the complaint may be dismissed upon
motion of defendant or upon the court's own motion, without prejudice to the right of the defendant to prosecute
his counterclaim in the same or in a separate action. This dismissal shall have the effect of adjudication upon the
merits, unless otherwise declared by the court.

The present rule embodied in Sections 2 and 3 of Rule 17 ordains a more equitable disposition of the
counterclaims by ensuring that any judgment thereon is based on the merit of the counterclaim itself and not on
the survival of the main complaint. Certainly, if the counterclaim is palpably without merit or suffers
jurisdictional flaws, which stand independent of the complaint, the trial court is not precluded from dismissing it
under the amended rules, provided that the judgment or order dismissing the counterclaim is premised on those
defects. At the same time, if the counterclaim is justified, the amended rules now unequivocally protect such
complaint from peremptory dismissal by reason of the dismissal of the complaint.

VINLUAN,  ALEXANDER  MARA  J.  2C  UST  LAW  


In the Matter of the Petition for the Issuance of a Writ of Amparo in Favor of Lilibeth Ladaga v. Major
General Reynaldo Mapagu, et al., G.R. No. 189689/G.R. No. 189691, November 13, 2012

FACTS:

Petitioners share the common circumstance of having their names included in what is alleged to be a JCICC
“AGILA” 3rd Quarter 2007 Order of Battle Validation Result of the Philippine Army's 10th
Infantry Division (10th ID). They perceive that by the inclusion of their names in the said Order of Battle (OB
List), they become easy targets of unexplained disappearances or extralegal killings – a real threat to their
life, liberty and security. Atty. Ladaga, first came to know of the existence of the OB List from an
undisclosed source in which in the said list, it was reflected that the ULTIMATE GOAL is to TRY TO OUST
PGMA ON 30 NOV 2007. On the other hand, Atty. Trinidad delivered a privileged speech before the members
of the Sangguniang Panlungsod to demand the removal of her name from said OB List. The Commission on
Human Rights, for its part, announced the conduct of its own investigation into the matter. In the course of the
performance of her dutites and functions, she has not committed any act against national security that would
justify the inclusion of her name in the said OB List. Atty. Carlos Isagani T. Zarate was informed that he was
also included on the OB List. In his petition, he alleged that the inclusion of his name in the said OB List was
due to his advocacies as a public interest or human rights lawyer. The Petitioners assert that the OB List is
really a military hit-list as allegedly shown by the fact that there have already been three victims of extrajudicial
killing whose violent deaths can be linked directly to the OB List.

RTC - Petition for the Issuance of a Writ of Amparo. The RTC subsequently issued separate Writs of Amparo,
directing the respondents to file a verified written return. After submission of the parties’ respective Position
Papers, the RTC issued Orders finding no substantial evidence to show that the perceived threat to petitioners’
life, liberty and security was attributable to the unlawful act or omission of the respondents. The privilege of the
Writ was therefore denied.

ISSUE:

WON THE TOTALITY OF EVIDENCE SATISFIES THE DEGREE OF PROOF REQUIRED UNDER
THE WRIT OF AMPARO.

HELD: NO.

The evidence does not satisfy degree of proof for the issuance of the Writ of Amparo. The Writ of Amparo was
promulgated by the Court pursuant to its rule-making powers in response to the alarming rise in the number of
cases of enforced disappearances and extrajudicial killings. It is an extraordinary remedy intended to address
violations of, or threats to, the rights to life, liberty or security and that, being a remedy of extraordinary
character, is not one to issue on amorphous or uncertain grounds but only upon reasonable certainty. Under the
Rule on the Writ of Amparo, the parties shall establish their claims by substantial evidence, and if the
allegations in the petition are proven by substantial evidence, the court shall grant the privilege of the writ and
such reliefs as may be proper and appropriate Substantial evidence is that amount of relevant evidence which a
reasonable mind might accept as adequate to support a conclusion. Petitioners sought to prove that the inclusion
of their names in the OB List presented a real threat to their security by attributing the violent deaths of the
other known activists to the inclusion of their names or the names of their militant organizations in the subject
OB List. However, the existence of the OB List could not be directly associated with the menacing behaviour of
suspicious men or the violent deaths of certain personalities.

VINLUAN,  ALEXANDER  MARA  J.  2C  UST  LAW  


Audi AG v. Hon. Jules A. Mejia, in his capacity as Executive Judge
of the Regional Trial Court, Alaminos City; Auto Prominence
Corporation; and Proton Pilipinas Corporation
G.R. No. 167533, July 27, 2007

FACTS:

Audi AG, a non-resident foreign company engaged in the manufacture of Audi brand cars, is organized in the
Federal Republic of Germany, with principal office at Ingolstadt, Germany. It is not licensed to do business in
the Philippines but is suing on an isolated transaction. Audi AG entered into an Assembly and Distributorship
Agreement with Proton. According to the agreement, Proton shall be the sole assembler and distributor of Audi
cars in the Philippines; and Audi AG will make representations that Proton will be the exclusive assembler and
distributor of Audi cars and local parts manufacturer for export purposes. Relying on Audi AG’s
representations, Proton borrowed money to buy assembly plant and distributorship, tools and equipment,
showrooms and offices, and license fees and brochures, etc. Proton later discovers that Audi AG did not include
the Philippines in its ASEAN Assembly Strategy program, but only Malaysia. Audi AG terminating the
agreements later sent a letter.

RTC- Auto Prominence and Proton filed a complaint for specific performance and injunction against petitioner
Audi AG with the RTC, Alaminos City.

CA - After hearing, the Executive Judge ordered the issuance of a TRO and the maintenance of the status quo
ante, even prior to the raffle of the case. Audi AG filed a Petition for Certiorari under Rule 65 before the
Supreme Court. The respondents challenged the Petition for being filed without a prior Motion for
Reconsideration and failure to observe the hierarchy of courts, among others.

ISSUE:

WON THE PETITION SHOULD BE DISMISSED.

HELD: YES

The petitioner failed to file with the trial court the requisite motion for reconsideration of the challenged Order
before resorting to the instant recourse. The well-established rule is that a motion for reconsideration is an
indispensable condition before an aggrieved party can resort to the special civil action for certiorari under Rule
65 of the 1997 Rules of Civil Procedure, as amended. Moreover, petitioner, by filing directly with this Court its
petition, has ignored the established rule on hierarchy of courts. It must be stressed that the Court of Appeals
and the Supreme Court have original concurrent jurisdiction over petitions for certiorari. The rule on hierarchy
of courts determines the venue of appeals. Such rule is necessary to prevent inordinate demands upon the Courts
precious time and attention which are better devoted to matters within its exclusive jurisdiction, and to prevent
further overcrowding of the Court’s docket. Audi AG should have filed the petition before the Court of Appeals
instead.

Procedural rules are not to be disdained as mere technicalities. They may not be ignored to suit the convenience
of a party. It ensures the effective enforcement of substantive rights through the orderly and speedy
administration of justice. Public order and our system of justice are well served by a conscientious observance
by the parties of the procedural rules.

VINLUAN,  ALEXANDER  MARA  J.  2C  UST  LAW  


DESIDERIO DE LOS REYES and MYRNA VILLANUEVA v.
PEOPLE OF THE PHILIPPINES and HON. ANTONIO M. EUGENIO, JR.

FACTS:

A complaint was filed with the MTC of Calauan, Laguna by the Philippine Coconut Authority against the
petitioners for cutting down and processed a more or less 440 coconut trees without the required permit, thus in
violation of The Coconut Preservation Act of 1995.

MTC- ordered the accused, including petitioners, to file their counter-affidavits within ten (10) days from
notice. Petitioners, instead of submitting their counter-affidavits, filed a Motion for Preliminary Investigation.
Petitioners filed a Motion To Quash the complaint on the ground that the allegations therein do not constitute an
offense. MTC issued an Order denying the motion and requiring anew all the accused to file their counter-
affidavits within five (5) days from notice.

RTC - filed a petition for certiorari, prohibition, and mandamus, alleging that the MTC committed grave abuse
of discretion amounting to lack or excess of jurisdiction when it denied their Motion To Quash. RTC dismissed
the petition and ruled that the MTC did not gravely abuse its discretion considering that the allegations in the
complaint, if hypothetically admitted, are sufficient to constitute the elements of the offense. Subsequently,
motion for reconsideration was filed but was denied also.

CA- rendered its Decision affirming the RTC Orders, holding that since petitioners are raising a question of law,
they should have filed a petition for review on certiorari with the Supreme Court. A motion for reconsideration
was filed but it was denied.

ISSUE:

WON CERTIORARI WAS THE PROPER REMEDY

HELD: NO

Certiorari is not a substitute for a lost appeal. There was no procedural lapse when petitioners initially appealed
the RTC Orders to the CA. But what they should have done after the Appellate Court rendered its Decision
affirming the RTC Orders was to seasonably file with the SC an appeal by a petition for review on certiorari
pursuant to Rule 45 of the 1997 Rules of Civil Procedure, as amended. Moreover, it is a dictum that when a
motion to quash in a criminal case is denied, the remedy is not certiorari, but for petitioners to go to trial
without prejudice to reiterating the special defenses invoked in their motion to quash. In the event that an
adverse decision is rendered after trial on the merits, an appeal therefrom is the next legal step. The RTC, CA
and the SC have original concurrent jurisdiction over petitions for certiorari; the rule on hierarchy of courts
determines the venue of recourses to these courts. In original petitions for certiorari, the Supreme Court will not
directly entertain this special civil action unless the redress desired cannot be obtained elsewhere based on
exceptional and compelling circumstances justifying immediate resort to the Supreme Court.

VINLUAN,  ALEXANDER  MARA  J.  2C  UST  LAW  


COMMISSION ON ELECTIONS, COMELEC CHAIRMAN ALFREDO L. BENIPAYO, COMELEC
COMMISSIONERS RESURRECCION Z. BORRA and FLORENTINO A. TUASON, JR.,
petitioners, vs. JUDGE MA. LUISA QUIJANO-PADILLA, REGIONAL TRIAL COURT OF
QUEZON CITY, BRANCH 215 and PHOTOKINA MARKETING CORP., respondents.
FACTS:

The congress passed RA no. 8189 (Voter’s Registration Act of 1996) providing for the modernization and
computerization of the voters' registration list and the appropriate of funds therefor "in order to establish a
clean, complete, permanent and updated list of voters. The COMELEC promulgated a resolution approving in
principle the Voter's Registration and Identification System Project (VRIS). The VRIS Project envisions a
computerized database system for the May 2004 Elections. The idea is to have a national registration of voters
whereby each registrant's fingerprints will be digitally entered into the system and upon completion of
registration, compared and matched with other entries to eliminate double entries. OMELEC issued invitations
to pre-qualify and bid for the supply and installations of information technology equipment and ancillary
services for its VRIS Project. Photokina pre-qualified and was allowed to participate as one of the bidders. After
the public bidding was conducted, PHOTOKINA's bid in the amount of P6.588 Billion Pesos garnered the
highest total weighted score and was declared the winning bidder.
The COMELEC then issued a resolution, which approved the notice of award to Photokina, and then the parties
proceeded to formalize the contract. However, the COMELEC chairman raised objections. Photonika sent
several letters requesting the formal execution of the contract but to no avail. Thereafter,
Chairman Benipayo announced that the VRIS Project has been scrapped, dropped, junked, or set aside. He
further announced his plan to re-engineer the entire modernization program of the COMELEC, emphasizing his
intention to replace the VRIS Project with his own version, the Triple E Vision.

RTC - petition for mandamus, prohibition and damages (with prayer for temporary restraining order,
preliminary prohibitory injunction and preliminary mandatory injunction) against the COMELEC and all its
Commissioners. PHOTOKINA alleged three causes of action: first, the deliberate refusal of the COMELEC
and its Commissioners to formalize the contract rendered nugatory the perfected contract between
them; second, in announcing that the VRIS Project has been junked and that he has plans to re-engineer the
COMELECs entire modernization program, Chairman Benipayo committed grave abuse of
discretion; and third, the COMELECs failure to perform its duty under the contract has caused PHOTOKINA to
incur damages since it has spent substantial time and resources in the preparation of the bid and the draft
contract. It rendered a decision, which granted the application for a writ of preliminary injunction.
Respondent judge issued the second assailed Resolution denying the COMELECs Omnibus Motion and, this
time, granting PHOTOKINAs application for a writ of preliminary mandatory injunction
Hence, the instant petition for certiorari filed by the Office of the Solicitor General in behalf of then
COMELEC.
ISSUE:
WON THE OSG HAS NO STANDING TO FILE THE PRESENT PETITION SINCE ITS LEGAL
POSITION IS CONTRARY TO THAT ESPOUSED BY THE MAJORITY OF THE COMELEC
COMMISSIONERS.
WON THERE IS A BREACH OF VIOLATION OF HIERARCHY OF COURTS
WON A PETITION FOR MANDAMUS IS THE APPROPRIATE REMEDY TO ENFORCE
CONTRACTUAL OBLIGATIONS.
VINLUAN,  ALEXANDER  MARA  J.  2C  UST  LAW  
HELD.
1. NO. It is incumbent upon him (Solicitor General) to present to the court what he considers would legally
uphold the best interest of the government although it may run counter to a clients position. Hence, while
petitioners stand is contrary to that of the majority of the Commissioners, still, the OSG may represent the
COMELEC as long as in its assessment, such would be for the best interest of the government. For, indeed, in
the final analysis, the client of the OSG is not the agency but no less than the Republic of the Philippines in
whom the plenum of sovereignty resides. Moreover, it must be emphasized that petitioners are also public
officials entitled to be represented by the OSG. The OSG is the lawyer of the government, its agencies and
instrumentalities, and its officials or agents.

2. NO. Suffice it to say that it is not an ironclad dictum. On several instances where this Court was confronted
with cases of national interest and of serious implications, it never hesitated to set aside the rule and proceed
with the judicial determination of the case. The case at bar is of similar import. It is in the interest of the State
that questions relating to government contracts be settled without delay. This is more so when the contract, as in
this case, involves the disbursement of public funds and the modernization of our country’s election process, a
project that has long been overdue.

3. NO. No rule of law is better settled than that mandamus does not lie to enforce the performance of
contractual obligations. It must, therefore, appear upon every application for a mandamus that it is the legal duty
of the respondent to do that which it is sought to compels him to do, and that he has upon proper application
refused to perform that duty. Court hold that mandamus is not the proper recourse to enforce the COMELEC's
alleged contractual obligations with PHOTOKINA. It has other adequate remedy in law. Moreover, the judicial
caution that mandamus applies as a remedy only where petitioner's right is founded clearly in law
and not when it is doubtful. Legal rights may be enforced by mandamus only if those rights are well defined,
clear and certain. Here, the alleged contract, relied upon by PHOTOKINA as source of its rights, which it seeks
to be protected, is being disputed, not only on the ground that it was not perfected but also because it is illegal
and against public policy.

VINLUAN,  ALEXANDER  MARA  J.  2C  UST  LAW  


United Claimants Association of NEA v. National Electrification Administration, G.R. NO. 187107,
January 31, 2012
FACTS:
NEA is a government-owned and/or controlled corporation created in accordance with Presidential Decree No.
269. Under PD 269, the NEA Board is empowered to organize or reorganize NEA’s staffing structure. Electric
Power Industry Reform Act of 2001 (EPIRA Law), was enacted to enhance and accelerate the electrification of
the whole country, including the privatization of the National Power Corporation. Subsequently, the Rules and
Regulations to implement RA 9136 were issued wherein it was provided that all the NEA employees and
officers are considered terminated and the 965 plantilla positions of NEA vacant. Meanwhile, EO 119 was
issued directing the NEA Board to submit a reorganization plan. Thereafter, the NEA implemented an early
retirement program denominated as the "Early Leavers Program," giving incentives to those who availed of it
and left NEA before the effectivity of the reorganization plan. The other employees were subsequently
terminated. Petitioners are former employees of NEA who were terminated from their employment with the
implementation of the assailed resolutions. Petitioners initiated an original action for Injunction with the
Supreme Court to restrain and/or prevent the implementation of NEA Termination Pay Plan, issued by
respondent NEA Board. Petitioners contend that The NEA Board has no power to terminate all the NEA
employees, Executive Order No. 119 did not grant the NEA Board the power to terminate all NEA employees,
and the assailed resolution were carried out in bad faith. Respondents, on the other hand, argued that the Court
has no jurisdiction over the petition, that Injunction is improper since that assailed resolutions have long been
implemented, and the assailed resolution was carried out in good faith.

ISSUES:

WON THE COURT HAS JURISDICTION NOTWITHSTANDING THE FACT THAT THE ACTION
FOR INJUNCTION WAS FILED DIRECTLY WITH THE SC W/O REGARD TO THE DOCTRINE
OF HIERARCHY OF COURTS.
WON INJUNCTION IS AVAILABLE.
WON THE NEA BOARD HAD THE POWER TO PASS THE ASSAILED RESOLUTION
TERMINATING ALL OF ITS EMPLOYEES.

HELD:
1.   YES.   While  it  is  true  that  by  virtue  of  the  doctrine  of  hierarchy  of  courts,  the  instant  petition  should  have  been  
filed  with  the  RTC.  However,  as  an  exception  to  this  general  rule,  the  principle  of  hierarchy  of  courts  may  be  set  
aside   for   special   and   important   reasons.   Such   reason   exists   in   the   instant   case   involving   as   it   does   the   employment  
of   the   entire   plantilla   of   NEA,   more   than   700   employees   all   told,   who   were   effectively   dismissed   from   employment  
in  one  swift  stroke.  This  to  the  mind  of  the  Court  entails  its  attention.

2. YES. As a rule, the writ of prohibition will not lie to enjoin acts already done. However, as an exception to
the rule on mootness, courts will decide a question otherwise moot if it is capable of repetition yet evading
review. Similarly, in the instant case, while the assailed resolutions of the NEA Board may have long been
implemented, such acts of the NEA Board may well be repeated by other government agencies in the
reorganization of their offices. Petitioners have not lost their remedy of injunction.
3. YES. In Betoy v. The Board of Directors, National Power Corporation, the Court upheld the dismissal of all
the employees of the NPC pursuant to the EPIRA Law. In ruling that the power of reorganization includes the
power of removal, the Court explained that Reorganization involves the reduction of personnel, consolidation of
offices, or abolition thereof by reason of economy or redundancy of functions. It could result in the loss of one’s
position through removal or abolition of an office. However, for a reorganization for the purpose of economy or
to make the bureaucracy more efficient to be valid, it must pass the test of good faith; otherwise, it is void ab
initio.

VINLUAN,  ALEXANDER  MARA  J.  2C  UST  LAW  


Sinter Corporation and Phividec Industrial Authority v. Cagayan Electric Power and Light Co., Inc.,
G.R. No. 127371, April 25, 2002

FACTS:

A Cabinet Reform Policy was approved which provides: “Continue direct connection for industries authorized
under the BOI-NPC Memorandum of Understanding of 12 January 1981, until such time as the appropriate
regulatory board determines that direct connection of industry to NPC is no longer necessary in the franchise
area of the specific utility or cooperative. Determination shall be based in the utility or cooperatives meeting the
standards of financial and technical capability with satisfactory guarantees of non-prejudice to industry to be set
in consultation with NPC and relevant government agencies and reviewed periodically by the regulatory
board.” Pursuant to such, the respondent, grantee of a legislative franchise to distribute electric power to the
municipalities of Villanueva, Jasaan and Tagoloan, and the city of Cagayan de Oro, all of the province of
Misamis Oriental, filed with the Energy Regulatory Board (ERB) a petition entitled “In Re: Petition for
Implementation of Cabinet Policy Reforms in the Power Sector.” The petition sought the “discontinuation of all
existing direct supply of power by the National Power Corporation (NPC, now NAPOCOR) within
CEPALCO’s franchise area.”

The ERB granted the petition of CEPALCO. Subsequently, the motion for reconsideration of NAPOCOR was
denied. NAPOCOR then went to the Court of Appeals (CA), and filed a petition for review. The CA dismissed
the petition on the ground that the motion for reconsideration filed with the ERB was file out of time, and thus,
rendering the decision of the ERB final and executory and can no longer be the subject of a petition for review.
Thereafter, NAPOCOR went to the Supreme Court (SC) and filed a petition for review on certiorari. The SC
affirmed the CA’s decision.

For the implementation of the said decision, CEPALCO wrote the petitioner and advised them to have the
power supply of PSC, directly taken from NPC, disconnected, cut and transferred to CEPALCO. PSC however
refused.

RTC – PSC and PIA filed an action for injunction. RTC rendered judgment in favor PSC and PIA. Motion for
reconsideration filed by CEPALCO was denied.

CA – ruled in favor of CEPALC, and the motion for reconsideration was denied.

ISSUE:

WON A PRELIMINARY INJUNCTION WILL PROSPER AGAINST THE FINAL AND


EXECUTORY DECISION OF THE ERB.

HELD: NO

The rule is that after a judgment has attained finality, it becomes the ministerial duty of the court to
order its execution. No court can interfere by injunction or otherwise restrain such execution. However, this
rule admits of exceptions: when facts and circumstances later transpire that would render execution inequitable
or unjust, the interested party may ask a competent court to stay the judgment’s execution or prevent its
enforcement and a change in the situation of the parties can warrant an injunctive relief. The Court further said
that under Sec.10 Executive Oder No. 172 (the law creating the ERB) review of the decisions of the ERB is
lodged with the SC. Where the law provides for an appeal to the SC or CA from Administrative Agencies, this
means that the latter are co-equal with the RTCs in terms of rank and stature, and thus, beyond the RTCs
control. Hence, this being the case, the RTC cannot interfere with the decision of the ERB.

VINLUAN,  ALEXANDER  MARA  J.  2C  UST  LAW  


Omictin v. Court of Appeals, G.R. No. 148004, January 22, 2007

FACTS:

The petitioner in this case filed a complaint for two counts of estafa with the Office of the City Prosecutor of
Makati against the private respondent. He alleged that private respondent, despite repeated demands, refused to
return the two company vehicles entrusted to him when he was still the president of Saag Phils., Inc. The public
prosecutor recommended the indictment of private respondent, and on the same day, respondent was charged
with the crime of estafa under Article 315, par. 1(b) of the RPC before the RTC. Private respondent filed a
motion to suspend proceedings on the basis of a prejudicial question because of a pending petition with the SEC
involving the same parties. The said case under the SEC involved the declaration of nullity of the respective
appointments of Alex Y. Tan and petitioner as President Ad Interim and Operations Manager Ad Interim of
Saag Phils., Inc., declaration of dividends, recovery of share in the profits, involuntary dissolution and the
appointment of a receiver, recovery of damages and an application for a TRO and injunction against Saag (S)
Pte. Ltd., Nicholas Ng, Janifer Yeo, Tan and petitioner.

In a case for estafa, a valid demand made by an offended party is one of the essential elements. It appears from
the records that the delay of delivery of the motor vehicles by Lagos to Saag Corporation is by reason of his
contention that the demand made by Omictin and Atty. Tan to him to return the subject vehicles is not a valid
demand. As earlier mentioned, petitioner filed a case with the SEC questioning therein private respondents
appointment.

If the SEC should rule that the dissolution of Saag Phils. is proper, or that the appointments of private
respondents are invalid, the criminal case will eventually be dismissed due to the absence of one of the essential
elements of the crime of estafa. Based on the foregoing, it is clear that a prejudicial question exists which calls
for the suspension of the criminal proceedings before the lower court.

ISSUE:
WON A PREJUDICIAL QUESTION EXISTS TO WARRANT THE SUSPENSION OF THE
CRIMINAL PROCEEDINGS PENDING THE RESOLUTION OF THE INTRA-CORPORATE
CONTROVERSY THAT WAS ORIGINALLY FILED WITH THE SEC.

HELD: YES

Ultimately, the resolution of the issues raised in the intra-corporate dispute will determine the guilt or innocence
of private respondent in the crime of estafa filed against him by petitioner before the RTC of Makati. Since the
alleged offended party is Saag Phils., Inc., the validity of the demand for the delivery of the subject vehicles
rests upon the authority of the person making such a demand on the company’s behalf. Private respondent is
challenging Omictin’s authority to act for Saag Phils., Inc. in the corporate case pending before the RTC of
Mandaluyong. Taken in this light, if the supposed authority of petitioner is found to be defective, it is as if no
demand was ever made, hence, the prosecution for estafa cannot prosper.

By analogy, the doctrine of primary jurisdiction may be applied in this case. The issues raised by petitioner
particularly the status of Saag Phils., Inc. vis--vis Saag (S) Pte. Ltd., as well as the question regarding the
supposed authority of the latter to make a demand on behalf of the company, are proper subjects for the
determination of the tribunal hearing the intra-corporate case which in this case is the RTC of Mandaluyong.
These issues would have been referred to the expertise of the SEC in accordance with the doctrine of primary
jurisdiction had the case not been transferred to the RTC of Mandaluyong.

VINLUAN,  ALEXANDER  MARA  J.  2C  UST  LAW  


REPUBLIC OF THE PHILIPPINES, ET. AL. , Petitioner
vs. CARLITO LACAP, Respondent
G.R. NO. 158253, MARCH 2, 2007

FACTS:

The respondent, doing business under the name of Carwin Construction, sought to collect payment for the
completed project by virtue of the Contract Agreement with DPWH. The COA disapproved the final release of
funds because respondent’s license had expired at the time of the execution of the contract. DPWH Legal
Department opined that RA No. 4566 known as Contractor’s License Law, does not provide that a contract
entered into after the license has expired is void thus recommended the payment to respondent.

RTC- filed the complaint for Specific Performance and Damages. Petitioner, thru the Office of the Solicitor
General (OSG) filed a Motion to Dismiss on the ground that complaint states no cause of action and RTC had
no jurisdiction since respondent did not appeal the COA decision. Respondent filed an Opposition to the Motion
to Dismiss. RTC denied the Motion to Dismiss thus the OSG filed a Motion for Reconsideration but it was
likewise denied. OSG filed its Answer invoking defenses of non-exhaustion of administrative remedies and
doctrine of non-suability of the State. RTC rendered a decision in favor of Respondent.

CA- Dissatisfied, Republic filed an appeal. CA sustained the RTC since the case involved the application of the
principle of estoppel against the Republic, which is purely a legal question hence, this petition.

ISSUES:

WON THE RESPONDENT FAILED TO EXHAUST ADMINISTRATIVE REMEDIES.


WON THE COA HAS JURISDICTION TO RESOLVE THE RESPONDENT’S MONEY CLAIM

HELD:

1. NO. The CA did not err in holding that respondent did not fail to exhaust administrative remedies. The
general rule is that before a party may seek the intervention of the court, he should first avail of all means
afforded him by administrative processes. The doctrine of exhaustion of administrative remedies has exceptions
such as “where there is unreasonable delay or official inaction that will irretrievably prejudice the complainant”
and “where the question involved is purely legal and will ultimately have to be decided by the courts of justice.”
Whether a contractor with an expired license at the time of the execution of its contract is entitled to be paid for
completed projects is a pure question of law. RA No. 4566 merely provide for a fine for any contractor who, for
a price, contracts without first securing a license. It does not declare as void contracts such contracts. Thus
respondent should be paid for the projects he completed but such payment, however, is without prejudice to the
payment of fine prescribed under the law.

2. NO. The CA did not err in holding that COA has no primary jurisdiction because there involved a purely
legal question thus final determination on the matter rests not with them but with the courts of justice.

VINLUAN,  ALEXANDER  MARA  J.  2C  UST  LAW  


Gabriel Abad, et. al., vs Regional Trial Court of Manila G.R. No. L-65505, October 12, 1987

FACTS:

The petitioner filed a complaint against the respondent Philippines American General Insurance Company, Inc.
for the enforcement of contract and recovery of loss of money - payment of the money value of the respective
accumulated sick leave with pay of the separated employees of respondent company either thru retirement,
retrenchment or resignation. Instead of filing an answer thereto, PHILAMGEN moved to dismiss the complaint,
which the trial court granted. After a denial of their motion to reconsider, petitioners filed before this Court a
petition for Certiorari.

The case was remanded to the trial court, unfortunately a fire destroyed the sala wherein the entire records were
kept, however, it was reconstituted. Thereafter, Philamgen filed its Answer to the complaint. Judicial
reorganization took place by the passage of Executive Order No. 864 and the case was re-raffled to respondent
RTC of Manila. Respondent court motu proprio dismissed the complaint declaring that it lacked jurisdiction
over the subject matter, being money claims arising from employer-employee relations. Respondent judge
denied motion for reconsideration filed by petitioner. Hence, this petition for certiorari.

ISSUE:
WON THE RTC ERRED IN REVERSING MOTU PROPIO THE SC DECISION BY DISMISSING
AGAIN THE PETITIONER’S ACTION OF THE GROUND OF LACK OF JURISDICTION.

WON THE RTC IN HOLDING ITSELF A DIFFERENT COURT FROM THE COURT OF FIRST
INSTANCE WHOSE CASES WHERE MERELY TAKEN OVER BY THE RESPONDENT.

HELD:

1. NO. Section 1 of Executive Order No. 864 provides that the Courts of First Instance shall be deemed
automatically abolished upon the constitution and organization of the courts provided in BP Blg. 129. Insofar as
the reorganized courts vested with general jurisdiction, BP Blg. 129 was and stills the controlling law. When it
comes to labor-related actions, however, such as the one at bar, initial jurisdiction is vested on administrative
machineries provided for the expeditious settlement of labor or industrial disputes. This Court is not also
unaware of that portion of Section 44 of BP Blg. 129 providing that cases pending in the abolished courts shall
be transferred to the appropriate courts created in the Act, but it is evident that the phrase 'appropriate courts'
must have reference to those courts whose jurisdiction are clearly defined in other parts of the law, otherwise a
mere transitory provision will serve to negate the primary and avowed purpose of the judiciary reorganization
act. But be that as it may, this provision has hardly any application here because this case is being referred to
administrative machinery, which has better facilities of adjudicating the claim more expeditiously as they are
not hamstrung by the strict rules of procedure and evidence.

2. NO. One of the important features in the Judiciary Reorganization is defining the jurisdiction of the Regional
Trial Courts reading as follows: “In all cases not within the jurisdiction of any court, tribunal, person or body
exercising judicial or quasi-judicial functions.” The Regional Trial Courts of today are actually same courts that
functioned as Courts of First Instance. There might have been a change in the name and in some incidental
features, but essentially, they are the same. However, whereas before jurisdiction over money claims of laborers
and employees appertained to Courts of First Instance, the same are now to be taken cognizance of by proper
entities in the Department of Labor and Employment. The rule of adherence of jurisdiction until a cause is
finally resolved or adjudicated does not apply when the change in jurisdiction is curative in character.

VINLUAN,  ALEXANDER  MARA  J.  2C  UST  LAW  


OPTIMA REALTY CORPORATION vs HERTZ PHIL. EXCLUSIVE CARS, INC.
GR NO. 183035; JANUARY 9, 2013

FACTS:

Optima is engaged in the business of leasing and renting out commercial spaces and buildings to its tenants.
Optima and Respondent Hertz entered into a Contract of Lease over an office unit and parking slot in the
Optima Bulding for a period of 3 years. However, the lease agreement was amended by shortening the lease
period ot 2 years and 5 months. However, Hertz failed to pay its rentals from August to December 2005 and
January to February 2006 notwithstanding the fact that Optima granted the former’s request. It also failed to pay
its utility bills. Optima sent a letter to Hertz, reminding the latter if it will renew its contract by a new
negotiation between them and upon written notice by the lessee to the lessor at least 90 days before the
termination of the lease period. Since Hertz failed to send written notice renewing its contract and it’s desire to
negotiate, Optima did not renew the lease.

MeTC - filed a Complaint for Specific Performance, Injunction, Damages and Sum of money and prayed for the
issuance of a TRO and writ of preliminary Injunction against Optima. It sought the issuance of a TRO to enjoin
Optima from committing acts which would tend to disrupt it’s peaceful use and possession of the leased
premises and wit of preliminary injunction to order Optima to reconnect its utilities.

Optima demanded Hertz to surrender and vacate the leased premises and pay P420,967.28 covering rental
arrearages, unpaid utility bills and other charges. Due to Hertz’s refusal to vacate the leased premises, Optima
filed an action before the MeTC for Unlawful Detainer and Damages with Prayer for the Issuance of a TRO
and/or Preliminary Mandatory Injunction against Hertz. MeTC rendered a judgment in favor of Optima and
ordered Hertz to vacate the premises.

RTC – affirmed the decision of the MeTC; CA – reversed and set aside the decision of the RTC; ruled that due
to the improper service of summons, the MeTC failed to acquire jurisdiction over the person of respondent.
Hence, this petition for review on Certiorari under Rule 45.

ISSUES:
WON THE MeTC PROPERLY ACQUIRED JURISDICTION OVER THE PERSON OF
RESPONDENT
WON THE UNLAWFUL DETAINER CASE IS BARRED BY LITIS PENDENTIA
WON THE EJECTMENT OF HERTZ IS PROPER

HELD:
1.YES. Jurisdiction over the person of the defendant may be acquired either by service of summons or by the
defendant’s voluntary appearance in court and submission to its authority. In this case, the MeTC acquired
jurisdiction over the person of respondent Hertz by reason of the latter’s voluntary appearance in court. In spite
of the defective service of summons, the defendant opted to file an Answer with Counterclaim with Leave of
Court. Furthermore, it never raised the defense of improper service of summons in its answer with
counterclaim.

2. NO. The Court ruled that while there is identity of parties in both cases, the rights asserted and the reliefs
prayed for under the Complaint for Specific Performance and those under the present Unlawful Detainer
Complaint are different.

3. YES. Failure of Hertz to pay rental arrearages and uitility bills to Optima and the Expiration of the Contract
of Lease without any request from Hertz for a renegotiation at least 90 days prior to its expiration entitled
Lessor, Optima, to terminate the lease.
VINLUAN,  ALEXANDER  MARA  J.  2C  UST  LAW  
Afdal & Afdal v. Carlos, G.R.No. 173379, December 1, 2010

FACTS:

The respondent filed a complaint for unlawful detainer and damages against the petitioners before the MTC
Laguna. Respondent alleged that petitioners claiming rights under them were occupying, by mere tolerance, a
parcel of land in respondent’s name. Respondent claimed that petitioner sold the property to him but that he
allowed petitioners to stay in the property. Respondent demanded that petitioners claiming rights under them
turn over the property to him because he needed the property for his personal use. Respondent further alleged
that petitioners refused to heed his demand and he was constrained to file a complaint before the Lupon ng
Tagapamayapa. According to respondent, petitioners ignored the notices and the Lupon issued a "certificate to
file action." Then, respondent filed the complaint before the MTC. Three attempts to serve the summons and
complaint on petitioners but they failed to file an answer. Respondent filed an ex-parte motion and compliance
with position paper submitting the case for decision based on the pleadings on record.

MTC - ruled in favor of respondent and further issued a writ of execution; petitioners filed a petition for relief
from judgment with the MTC. Respondent filed a motion to dismiss or strike out the petition for
relief. Subsequently, petitioners manifested their intention to withdraw the petition for relief after realizing that
it was a prohibited pleading under the Revised Rule on Summary Procedure. The MTC granted petitioners’
request to withdraw the petition for relief.

RTC - petitioners filed the petition for relief; alleged that they are the lawful owners of the property which they
purchased and denied that they sold the property to respondent. They also pointed out that they never received
respondent’s demand letter nor were they informed of, much less participated in, the proceedings before
the Lupon. Moreover, petitioners said they were not served a copy of the summons and the complaint. RTC
dismissed the petition for relief - averred that it had no jurisdiction over the petition because the petition should
have been filed before the MTC. Hence, this petition.

ISSUES:

WON RTC ERRED IN DISMISSING THEIR PETITION FOR RELIEF FROM JUDGMENT.
WON THERE WAS A VALID SERVICE OF SUMMONS.

HELD:

1. NO. Petitioners cannot file the petition for relief with the MTC because it is a prohibited pleading in an
unlawful detainer case. Petitioners cannot also file the petition for relief with the RTC because the RTC has no
jurisdiction to entertain petitions for relief from judgments of the MTC. The remedy of petitioners in such a
situation is to file a petition for certiorari with the RTC under Rule 65 of the Rules of Court on the ground of
lack of jurisdiction of the MTC over the person of petitioners in view of the absence of summons to petitioners.

2. NO. SC held that the impossibility of personal service justifying availment of substituted service should be
explained in the proof of service; why efforts exerted towards personal service failed. The pertinent facts and
circumstances attendant to the service of summons must be stated in the proof of service otherwise, the
substituted service cannot be upheld. The indorsements in this case failed to state that prompt and personal
service on petitioners was rendered impossible. It failed to show the reason why personal service could not be
made. It was also not shown that efforts were made to find petitioners personally and that said efforts failed.
These requirements are indispensable because substituted service is in derogation of the usual method of
service. Failure to faithfully, strictly, and fully comply with the statutory requirements of substituted service
renders such service ineffective.
VINLUAN,  ALEXANDER  MARA  J.  2C  UST  LAW  
Fe V. Rapsing, et al. v. Hon. Judge Maximino R. Ables, et al., G.R. No.171855, October 15, 2012

FACTS:

Respondents, members of the Alpha Company, 9th Division of the Philippine Army based, alleged to have
received information about the presence of armed elements reputed to be New People’s Army partisans. Acting
on the information, they coordinated with the PNP and proceeded to the place. Thereat, they encountered armed
elements, which resulted in an intense firefight. When the battle ceased, seven persons were found sprawled on
the ground lifeless. The post-incident report of the Philippine Army states that a legitimate military operation
was conducted and in the course of which, the victims, armed with high-powered firearms, engaged in a
shootout with the military. Petitioners complained that there was no encounter that ensued and that the victims
were summarily executed in cold blood by respondents. After investigation, the NBI recommended to the
Provincial Prosecutor of Masbate City that a preliminary investigation be conducted against respondents for the
crime of multiple murder. NBI relied on the statements of witnesses who claim that the military massacred
helpless and unarmed civilians.

RTC - warrant for the arrest then was issued; but before respondents could be arrested, the Judge Advocate
General's Office (JAGO) of the AFP filed an Omnibus Motion with the trial court seeking the cases against
respondents be transferred to the jurisdiction of the military tribunal; denied the motion filed by the JAGO on
the ground that respondents have not been arrested. A Motion for Reconsideration was then filed by the JAGO
and the trial court granted the Omnibus Motion and the entire records of the case were turned over to the
Commanding General, for appropriate action.

ISSUE:

WON RESPONDENT JUDGE ACTED IN GADALEJ IN GRANTING THE MOTION TO TRANSFER


THE INSTANT CRIMINAL CASE TO THE JURISDICTION OF THE MILITARY COURT BASED
ON FACTS AND IN LAW, HAS NO JURISDICTION OVER THE MURDER CASE.

HELD: YES

It is an elementary rule of procedural law that jurisdiction over the subject matter of the case is conferred by law
and is determined by the allegations of the complaint irrespective of whether the plaintiff is entitled to recover
upon all or some of the claims asserted therein. As a necessary consequence, the jurisdiction of the court cannot
be made to depend upon the defenses set up in the answer or upon the motion to dismiss, for otherwise, the
question of jurisdiction would almost entirely depend upon the defendant. What determines the jurisdiction of
the court is the nature of the action pleaded as appearing from the allegations in the complaint. The averments in
the complaint and the character of the relief sought are the matters to be consulted.

In the case at bar, the information states that respondents, “conspiring together and mutually helping with one
another, taking advantage of their superior strength, as elements of the Philippine Army, armed with their
government-issued firearms with intent to kill, by means of treachery and evident premeditation, did then and
there willfully, unlawfully and feloniously attack, assault and shoot the victims, hitting them on different parts
of their bodies, thereby inflicting upon them multiple gunshot wounds which caused their deaths.”

In view of the provisions of R.A. 7055, the military tribunals cannot exercise jurisdiction over respondents' case
since the offense for which they were charged is not included in the enumeration of “service-connected offenses
or crimes” as provided for under Section 1 thereof. The said law is very clear that the jurisdiction to try
members of the AFP who commit crimes or offenses covered by the RPC, and which are not service-connected,
lies with the civil courts.
VINLUAN,  ALEXANDER  MARA  J.  2C  UST  LAW  
Mendoza v. Germino & Germino, G.R. No. 165676, November 22, 2010

FACTS:

The plaintiff was the owner of a 5-hectare land in Nueva Ecija wherein the respondents unlawfully entered the
said land by means of strategy and stealth, without the knowledge and consent of petitioner, and the said
respondent refused to vacate the land despite demands.

MTC - petitioner filed a complaint for forcible entry; Respondent filed an answer claiming that his brother
respondent Benigno was the plaintiff’s agricultural lessee and he merely helped the latter in the cultivation as a
member of the immediate farm household; plaintiffs moved to remand the case to the Dept. of Agriculture
Adjudication Board (DARAB), in view of the tenancy issue raised by respondent Narciso. Thus, it ordered the
remand of the case to DARAB.
PARAD (provincial agrarian reform adjudicators) - Plaintiff filed an amended complaint impleading
respondent Benigno (brother). They alleged that a different person was the agricultural lessee of the land,
contrary to respondent’s claim that it was his respondent Benigno; alleged that respondent Benigno unlawfully
entered the subject property through strategy and stealth and without their knowledge and consent, and
thereafter transferred possession to respondent Narciso. In both cases, respondents refused to vacate despite
demands, and also appropriated the fruits therein; PARAD ruled in favor of plaintiff and that respondents are
usurpers considering their failure to prove respondent Benigno as the agricultural lessee, thus, they should
vacate the land and pay for damages.
DARAB - respondents filed a notice of appeal; affirmed the PARAD holding that it acquired jurisdiction
because of the amended complaint that alleged an agrarian dispute.
CA - petition for review under Rule 43; ruled that the MTC erred in referring the case to the DARAB since the
material allegations and the relief sought were for forcible entry, and that the DARAB did not acquire
jurisdiction.
ISSUE:
WON THE DARAB HAD JURISDICTION OVER THE CASE.
HELD: NO.
It is a settled-rule that jurisdiction is conferred by law. Also, jurisdiction over the subject matter is determined
by the allegations in the complaint. Under the law, the MTC has exclusive jurisdiction over ejectment suits,
which shall be governed by the Revised Rule on Summary Procedure. On the other hand, the DARAB has
exclusive and primary jurisdiction to determine and adjudicate all agrarian disputes involving the
implementation of the CARP and other similar laws.
In this case, both the allegations and reliefs prayed for was clearly an action for forcible entry. Also,
respondent’s defense of tenancy did not automatically divest the court of its jurisdiction. Precisely, it had to
conduct a preliminary conference in order to determine whether it was an ejectment suit or an agrarian dispute
and whether it has jurisdiction or none. After all, jurisdiction is not affected by the pleas or theories set by
defendant in an answer or motion to dismiss. The amended complaint also did not confer jurisdiction to the
DARAB in the absence of any allegation of tenancy relationship between the parties.

VINLUAN,  ALEXANDER  MARA  J.  2C  UST  LAW  


Remedios Antonino v. The Register of Deeds of Makati City, et al., G.R. No. 185663, June 20, 2012

FACTS:

Petitioner was a lessee of Tian Tan Su in the latter’s residential property located in Makati City. The lease
contract between the parties granted petitioner the right of first refusal should Su sell the property. Later, Su and
Antonino had an Undertaking Agreement that Su would sell petitioner the subject property worth
P39,500,000.00. However, there was a disagreement as to who between them will pay the capital gains tax, and
the sale of the property did not proceed.

RTC – petitioner filed a complaint against Su for the reimbursement of the cost of repairs and payment of
damages, in the RTC of Makati. Later on, petitioner amended the complaint to include the enforcement of the
Undertaking Agreement. The RTC dismissed the complaint based on improper venue and the non-payment of
docket fees. The RTC ruled that the complaint was a personal action and the venue should thus be set based on
the respective residence of the plaintiff and the defendant. The petitioner, resided in Muntinlupa City while Su,
the defendant, resided in Manila City.

After the said dismissal, Su filed an Omnibus Motion praying for the cancellation of the notice of lis pendens on
the title of the subject property, and the issuance of a summary judgment. Petitioner moved to reconsider,
claiming that the action is a real action and that the place where the subject property is located should be the
basis for the venue of the complaint. She also presented evidence from the COMELEC, which stated that she is
a resident of Makati City. RTC denied the motion for reconsideration. It did not cancel the notice of lis pendens
and ruled that it did not acquire jurisdiction due to Petitioner’s failure to pay the docket fees. The RTC also
affirmed its ruling that the action is a personal action and the complaint was properly dismissed due to improper
venue.

CA - a petition for the annulment of judgment; dismissed the petition. It held that petitioner failed to prove any
exceptional circumstances warranting the remedy of annulment of judgment, and the fact that she failed to
appeal within the period allowed. The CA, however, ruled on the merits and further affirmed the RTC’s
findings that the action is indeed a personal action to enforce a contract, which further reinforced the dismissal
of the complaint based on improper venue. The CA also ruled that the petition for the annulment of judgment
only embraced two circumstances: 1) extrinsic fraud, and 2) lack of jurisdiction.

ISSUE:

WON THE PETITIONER’S RELIANCE ON THE REMEDY OF PETITION FOR ANNULMENT OF


JUDGMENT AGAINST A FINAL AND EXECUTORY ORDER OF THE RTC IS PROPER.

HELD: NO.

This remedy cannot be availed of if any of the two circumstances are not proven sufficiently. Annulment of
judgment is a challenge to the validity of a court’s judgment, and is not to be used lightly. Annulment of
judgment is not an alternative to appeal, as the grounds for appeal are different from the grounds for annulment
of judgment. Annulment of judgment cannot be a substitute for a lost appeal. In this case, petitioner failed to
appeal the case when she relied on the petition for the annulment of judgment. The RTC’s judgment was
already final and executory when the petition was filed. It was due to her own negligence that the period of
appeal lapsed before she could timely file one in the CA. Similarly; a second Motion for Reconsideration is an
improper remedy when a court denies the first Motion for Reconsideration. The purpose of a Motion for
Reconsideration is to allow a court to correct itself before elevating the case on a appeal to a higher court. It
cannot be filed twice as a substitute for an appeal.

VINLUAN,  ALEXANDER  MARA  J.  2C  UST  LAW  


DELFIN LAMSIS, MAYNARD MONDIGUING, JOSE VALDEZ, JR. and Heirs of AGUSTIN KITMA,
represented by EUGENE KITMA, Petitioners,
vs.
MARGARITA SEMON DONG-E, Respondent.
FACTS:

This case involves a conflict of ownership and possession over an untitled parcel of land located in Baguio City.
Petitioners are the actual occupants of Lot No. 1, respondent is claiming ownership thereof and is seeking to
recover its possession. According to respondent, her family’s ownership and occupation of Lot No. 1 can be
traced as far back as 1922 to her late grandfather, Ap-ap. The heirs of Ap-ap then executed, for a P500.00
consideration, a Deed of Quitclaim in favor of their brother Gilbert Semon (Respondent’s father). Gilbert
Semon together with his wife, allowed his in-laws Manolo Lamsis and Nancy Lamsis-Kitma, to stay on a
portion of Lot No. 1 together with their respective families, they were allowed to introduce improvements.
When Manolo Lamsis and Nancy Lamsis-Kitma died, their children, petitioners Delfin Lamsis (Delfin) and
Agustin Kitma (Agustin), took possession of certain portions of Lot No. 1. Delfin possessed 4,000 square
meters of Lot No. 1, while Agustin occupied 5,000 square meters thereof. Nevertheless, the heirs of Gilbert
Semon tolerated the acts of their first cousins.

RTC - respondent filed a complaint for recovery of ownership, possession, reconveyance and damages against
petitioners of Lot No. 1; prayed for the annulment of the sales to Maynard and Jose and for petitioners to vacate
the portions of the property which exceed the areas allowed to them by respondent; Petitioners denied
respondents’s claims of ownership and possession; according to Delfin and Agustin, Lot No. 1 is a public land
claimed by the heirs of Joaquin Smith (not parties to the case), which gave their permission for Delfin and
Agustin’s parents to occupy the land. petitioners’ claim that the Smiths owned the subject property, respondent
presented a certified copy of a Resolution from the Land Management Office denying the Smiths’ application
for recognition of the subject property as part of their ancestral land; respondent introduced as evidence an
unnumbered resolution of the Community Special Task Force on Ancestral Lands (CSTFAL) of the Department
of Environment and Natural Resources (DENR), acting favorably on her and her siblings’ ancestral land claim.
The RTC ruled in favor of the respondent.

CA - affirmed the RTC. CA held that the respondent was able to discharge her burden in proving her title and
interest to the subject property. Her documentary evidence were amply supported by the testimonial evidence of
her witnesses.

ISSUE:
WON THE CA DISREGARDED MATERIAL FACTS AND CIRCUMSTANCES IN AFFIRMING THE
RTC’S DECISION

WON THE PETITIONERS HAVE ACQUIRED THE SUBJECT PROPERTY BY PRESCRIPTION

WON THE RTC HAS JURISDICTION TO DECIDE THE CASE IN LIGHT OF THE EFFECTIVITY
OF THE IPRA LAW AT THE TIME THE COMPLAINT WAS INSTITUTED.

HELD:

1. NO. Both the trial and the appellate courts ruled that respondent has proven her claims of ownership and
possession with a preponderance of evidence. Petitioners argue that the two courts erred in their appreciation of
the evidence. Since it raises essentially questions of fact, this assignment of error must be dismissed for it is
settled that only questions of law may be reviewed in an appeal by certiorari.
VINLUAN,  ALEXANDER  MARA  J.  2C  UST  LAW  
2. NO. It is settled that possession, in order to ripen into ownership, must be in the concept of an owner, public,
peaceful and uninterrupted. Possession not in the concept of owner, such as the one claimed by petitioners,
cannot ripen into ownership by acquisitive prescription, unless the juridical relation is first expressly repudiated
and such repudiation has been communicated to the other party. Acts of possessory character executed due to
license or by mere tolerance of the owner are inadequate for purposes of acquisitive prescription. Possession by
tolerance is not adverse and such possessory acts, no matter how long performed, do not start the running of the
period of prescription. Petitioners made no effort to allege much less prove any act of repudiation sufficient for
the reckoning of the acquisitive prescription.

3. YES. They maintain that, under the IPRA, it is the NCIP which has jurisdiction over land disputes involving
indigenous cultural communities and indigenous peoples. As a rule, an objection over subject-matter
jurisdiction may be raised at any time of the proceedings. This is because jurisdiction cannot be waived by the
parties or vested by the agreement of the parties. Jurisdiction is vested by law, which prevails at the time of the
filing of the complaint. An exception to this rule has been carved by jurisprudence. In the seminal case of Tijam
v. Sibonghanoy, the Court ruled that the existence of laches will prevent a party from raising the court’s lack of
jurisdiction. Laches is defined as the "failure or neglect, for an unreasonable and unexplained length of time, to
do that which, by exercising due diligence, could or should have been done earlier; it is negligence or omission
to assert a right within a reasonable time, warranting the presumption that the party entitled to assert it either has
abandoned or declined to assert it." Wisely, some cases have cautioned against applying Tijam, except for the
most exceptional cases where the factual milieu is similar to Tijam.

In case at bar, the application of the Tijam doctrine is called for because the presence of laches cannot be
ignored. If the surety in Tijam was barred by laches for raising the issue of jurisdiction for the first time in the
CA, what more for petitioners in the instant case who raised the issue for the first time in their petition before
this Court. At the time that the complaint was first filed in 1998, the IPRA was already in effect but the
petitioners never raised the same as a ground for dismissal; instead they filed a motion to dismiss on the ground
that the value of the property did not meet the jurisdictional value for the RTC. They obviously neglected to
take the IPRA into consideration.

VINLUAN,  ALEXANDER  MARA  J.  2C  UST  LAW  


De Leon v. Court of Appeals, 245 SCRA 166 [1995]

FACTS:

Jesus Jalbuena entered into a verbal lease contract with Uldarico Inayan, for one year renewable for the same
period. Inayan was allowed to continue with the lease from year to year. Corazon Jalbuena de Leon is the
daughter of Jesus and the transferee of the subject property. Inayan ceased paying the agreed rental and instead,
asserted dominion over the land. When asked by De Leon to vacate the land, he refused to do so.

RTC - file a complaint for "Termination of Civil Law Lease; Recovery of Possession, Recovery of Unpaid
Rentals and Damages; Inayan claimed tenancy dispute thus the lower court issued an order adopting the
procedure in agrarian cases but still rendered decision Declaring the lease contract between plaintiff and
defendant as a civil law lease, and that the same has already been terminated due to defendant's failure to pay
his rentals from 1983 up to the present.

CA - Inayan raised the sole issue of jurisdiction and alleged that the lower court, acting as Court of Agrarian
Relations, had no jurisdiction over the action. The CA, at first affirmed the trial court's decision, but when an
MR filed by Inayan it then set aside its earlier decision and dismissed the civil case for want of jurisdiction; held
that petitioner's complaint below was anchored on acción interdictal , a summary action for recovery of physical
possession that should have been brought before the proper inferior court.

ISSUE:

WON THE RTC, THE ACTING AS A COURT OF AGRARIAN RELATIONS EMPLOYING


AGRARIAN PROCEDURE, HAD JURISDICTION TO TRY THE SUIT FILED BY DE LEON.

HELD: YES

Jurisdiction of the court over the subject matter is conferred only by the Constitution or by law. It is
determinable on the basis of allegations in the complaint. In order to determine whether the court below had
jurisdiction, it is necessary to first ascertain the nature of the complaint filed before it. A study of the complaint
instituted by petitioner in the lower court reveals that the case is, contrary to the findings of the respondent
appellate court, not one of unlawful detainer. Not being merely a case of ejectment, the regional trial court
possessed jurisdiction to try and resolve the case.

VINLUAN,  ALEXANDER  MARA  J.  2C  UST  LAW  


SERAFIN TIJAM, ET AL., Plaintiffs-Appellees, VS.
MAGDALENO SIBONGHANOY ALIAS GAVINO SIBONGHANOY, ET AL., Defendants,
MANILA SURETY AND FIDELITY CO., INC. (CEBU BRANCH) bonding Company and defendant-
appellant

FACTS:

CFI – Sps. Tijam and Tagalog filed a civil case against the spouses Magdaleno Sibonghanoy and Lucia Baguio
to recover from them the sum of P1,908.00, with legal interest; a writ of attachment was issued by the court
against the Sibonghanoys’ properties, but was soon dissolved after the filing of a counter-bond by the latter and
the Manila Surety and Fidelity Co., Inc; ruled in favor the plaintiffs.

The plaintiffs moved for the issuance of a writ of execution against the Surety’s bond after the writ of execution
against the defendants was returned unsatisfied. The Surety prayed the Court not only to deny the motion for
execution against its counter-bond but also the following affirmative relief: “to relieve the herein bonding
company of its liability, if any, under the bond in question”. Such motion was denied as there was no previous
demand made on the Surety for the satisfaction of the judgment. Thereafter the necessary demand was made,
and upon failure of the Surety to satisfy the judgment, the plaintiffs filed a second motion for execution against
the counter-bond.

CA - Affirmed the orders appealed from the CFI.

Surety filed a motion to dismiss, alleging that since the case was filed on July 19, 1948, a month after the
effectively of the Judiciary Act of 1948, Section 88 of which placed within the original exclusive jurisdiction of
inferior courts all civil actions where the value of the subject-matter or the amount of the demand does not
exceed P2,000.00, exclusive of interest and costs, the CFI therefore had no jurisdiction to try and decide the
case.

ISSUE:

WON THE SURETY IS ESTOPPED FROM QUESTIONING THE JURISDICTION OF THE CFI
CEBU FOR THE FIRST TIME UPON APPEAL.

HELD: YES

The SC believes that the Surety is now barred by laches from invoking such plea, almost fifteen years from the
commencement of the action on July 19, 1948, before filing its motion to dismiss on January 12, 1963, raising
the question of lack of jurisdiction for the first time. A party may be estopped or barred from raising a question
in different ways and for different reasons. Thus we speak of estoppel in pais, of estoppel by deed or by record,
and of estoppel by laches. Laches, in a general sense, is failure or neglect, for an unreasonable and unexplained
length of time, to do that which, by exercising due diligence, could or should have been done earlier; it is
negligence or omission to assert a right within a reasonable time, warranting a presumption that the party
entitled to assert it either has abandoned it or declined to assert it. The question of whether the court had
jurisdiction either of the subject matter of the action or of the parties was not important in such cases because
the party is barred from such conduct, not because the judgment or order of the court is valid and conclusive as
an adjudication, but for the reason that such a practice cannot be tolerated — obviously for reasons of public
policy.

VINLUAN,  ALEXANDER  MARA  J.  2C  UST  LAW  


FIRST CORPORATION, Petitioner, v. FORMER SIXTH DIVISION OF THE COURT OF APPEALS,
BRANCH 218 OF THE REGIONAL TRIAL COURT OF QUEZON CITY, EDUARDO M. SACRIS,
and CESAR A. ABILLAR, Respondents.

FACTS:

The petitioner (First Corporation) is a corporation duly organized and existing under the PH laws and engaged
primarily in trade. The private respondent (Sacris) is the alleged creditor of the petitioner, while another private
respondent (Abillar) had served as the President and Chairman of the Board of the petitioner corporation from
1993 until February 1998. The corporate officers of the petitioner convinced Sacris to invest in their business
as the petitioner needed a fresh equity infusion, particularly in its Rema Tip Top Division, to make viable its
continuous operation. The petitioner made a promise of turning such equity into shareholding in the petitioner.
While the conversion of such investment into shareholding was still pending, Sacris and the petitioner agreed to
consider the same as a loan which shall earn an interest of 1%/month. However, petitioner failed to convert
Sacris's investment/loan into equity or shareholding in the petitioner. Meanwhile, Abillar was no long re-elected
because they had lost their confidence for he had been involved in various anomalies and irregularities during
his tenure. Thus, Abillar was ousted.

Sacris excuted a Deed of Assignment in favor Abillar. Later on, Abillar, by the virtue of the said deed, filed a
complaint for Sum of Monet with Prayer for a Writ of Preliminary Attachment and Damages before the RTC of
Pasig against the petitioner. While the said case was still pending, Sacris and Abarilla agreed to rescind the said
deed for failure of Abillar fo comply with his undertaking to pay Sacris. Thus, Sacris and Abillar executed a
Deed of Rescission of the said deed. Sacris filed a Motion for Intervention attaching his Complain-in-
Intervention. The RTC Pasig denied the said Motion for Intervention but, however, the trial court admitted the
complain in intervention filed by Sacris and dismissed the complaint originally filed by Abillar against the
petitioner. The admission of the said complaint prompted the petitioner to file a Petition for Certiorari and
Prohibition before the CA. The CA granted the petition filed by the petitioner and issued a writ of certiorari in
which the order of the RTC Pasig were set aside. Thus, the CA directed Judge Hernandez to dismiss the
complaint with prejudice and to deny the Motion in Intervention without prejudice.

RTC - Sacris filed a complaint for Sum of Money with Damages against the petitioner to recover the alleged
collectible sum due from the petitioner. It rendered a decision in favor of Sacris and Abillar (private
respondents) ordering the petitioner to pay the said balance plus an interest of 24%/annum; pay Abillar
P20,000.00 and Sacris P50,000.00; attorney’s fees and cost of suit.

CA - The petitioner appealed the decision of the RTC QC in which the CA rendered a decision dismissing the
appeal filed because it did find any reversible error in the decision of the said RTC. A motion for
reconsideration was filed but was also denied by the CA because the CA had already passed upon the issues
raised.

ISSUE:

WON THE REMEDY OF CERTIORARI PROVIDED FOR UNDER THE RULE 65 WAS PROPERLY
APPLIED IN THE CASE.

VINLUAN,  ALEXANDER  MARA  J.  2C  UST  LAW  


HELD: NO.

The petitioner evidently availed itself of the wrong mode of appeal. It is a well-entrenched rule that this Court is
not a trier of facts. This Court will not pass upon the findings of fact of the trial court, especially if they have
been affirmed on appeal by the CA. Unless the case falls under the recognized exceptions, the rule should not be
disturbed.

In the case at bar, the findings of the RTC QC as well as the CA are properly supported by evidence on record.
Both courts found that the alleged loans extended to the petitioner by Sacris were reflected in the petitioner ‘s
financial statements, particularly in the years 1992-1993, were contrary to the claim of petitioner. The said
financial statements of the petitioner were not the sole bases used by the RTC QC and by the CA in its findings
of liability against the petitioner. The RTC QC also took into consideration the pieces of documentary evidence
which likewise became the grounds for its findings that indeed, Sacris had extended a loan to petitioner, and
that the same was given to Abillar, and received by the petitioner. Those pieces of documentary evidence very
well supported the claim of Sacris that the petitioner received money from him through its former President,
Abillar. Thus, petitioner cannot claim that it never consented to the act of Abillar of entering into a
loan/investment transaction with Sacris, for there are documents that would prove that the money was received
by the petitioner, and the latter acknowledged receipt of said money. The same pieces of evidence likewise
confirm the findings of the RTC QC that the petitioner benefited from the said transaction; therefore, it should
be held liable for the same amount of its unpaid obligation to Sacris. As the findings of the RTC QC and the CA
are supported by evidence, this Court finds no reason to deviate from the heretofore cited rule.

Settled is the rule that the proper remedy from an adverse decision of the CA is an appeal under Rule 45 and
not a Petition for Certiorari under Rule 65. Hence, petitioner could have raised the CA, affirming the assailed
decision of the RTC QC to this Court via an ordinary appeal under Rule 45 of the 1997 Revised Rules of Civil
Procedure. It should be emphasized that the extraordinary remedy of certiorari will not lie when there are other
remedies available to the petitioner. Therefore, in availing itself of the extraordinary remedy of certiorari, the
petitioner corporation resorted to a wrong mode of appeal.

VINLUAN,  ALEXANDER  MARA  J.  2C  UST  LAW  


CHESTER DE JOYA v. JUDGE PLACIDO C. MARQUEZ, G.R. No. 162416, January 31, 2006

FACTS:

This is a petition of certiorari and prohibition, petitioner De Joya seeks to nullify and set aside the warrant of
arrest issued by respondent judge against petitioner in Criminal Case No. 03-219952 for violation of Article
315, par. 2(a) of the Revised Penal Code in relation to Presidential Decree (P.D.) No. 1689. Petitioner asserts
that respondent judge erred in finding the existence of probable cause that justifies the issuance of a warrant of
arrest against him and his co-accused.

ISSUES:

WON THE SC CAN REVIEW THE FACTUAL FINDINGS OF THE RTC

WON THE WARRANT OF ARREST WAS VALIDLY ISSUED

WON THE PETITIONER IS ENTITLED TO SEEK THE RELIEF WITHOUT BEING SUBMITTED
TO THE JURISDICTION OF THE COURT.

HELD:

1. YES. The general rule is that the Supreme Court does not review the factual findings of the trial court, which
include the determination of probable cause for the issuance of warrant of arrest. It is only in exceptional cases
where the Court sets aside the conclusions of the prosecutor and the trial judge on the existence of probable
cause, that is, when it is necessary to prevent the misuse of the strong arm of the law or to protect the orderly
administration of justice. The facts obtaining in this case do not warrant the application of the exception.

2. YES. The Court found that the documents sufficiently establish the existence of probable cause as required
under Section 6, Rule 112 of the Revised Rules of Criminal Procedure. Probable cause to issue a warrant of
arrest pertains to facts and circumstances, which would lead a reasonably discreet and prudent person to believe
that an offense has been committed by the person sought to be arrested. It bears remembering "in determining
probable cause, the average man weighs facts and circumstances without resorting to the calibrations of our
technical rules of evidence of which his knowledge is nil. Rather, he relies on the calculus of common sense of
which all reasonable men have an abundance.” Thus, the standard used for the issuance of a warrant of arrest is
less stringent than that used for establishing the guilt of the accused. As long as the evidence presented shows
a prima facie case against the accused, the trial court judge has sufficient ground to issue a warrant of arrest
against him.

3. NO. Petitioner is not entitled to seek relief from Supreme Court nor from the trial court as he continuously
refuses to surrender and submit to the court's jurisdiction. There is no exceptional reason in this case to allow
petitioner to obtain relief from the courts without submitting to its jurisdiction. On the contrary, his continued
refusal to submit to the court's jurisdiction should give this Court more reason to uphold the action of the
respondent judge. The purpose of a warrant of arrest is to place the accused under the custody of the law to hold
him for trial of the charges against him. His evasive stance shows an intent to circumvent and frustrate the
object of this Legal process. It should be remembered that he who invokes the court's jurisdiction must first
submit to its jurisdiction.
VINLUAN,  ALEXANDER  MARA  J.  2C  UST  LAW  
A.L. Ang Network, Inc. v. Emma Mondejar, et al., G.R. No. 200804, January 22, 2014

FACTS:

Petitioner filed a complaint for sum of money under the Rule of Procedure for Small Claims Cases before the
MTCC, seeking to collect from respondent, which represented her unpaid water bills. Petitioner claimed that it
was duly authorized to supply water to and collect payment therefor from the homeowners of Regent Pearl
Subdivision. In defense, respondent contended that since April 1998 up to February 2003, she religiously paid
petitioner the agreed monthly flat rate of P75.00 for her water consumption. Notwithstanding their agreement
that the same would be adjusted only upon prior notice to the homeowners, petitioner unilaterally charged her
unreasonable and excessive adjustments.

MTCC - Respondent should be considered to have fully paid petitioner. Aggrieved, petitioner filed a petition for
certiorari under Rule 65 of the Rules of Court before the RTC, ascribing grave abuse of discretion on the part of
the MTCC.

RTC - Dismissed the petition for certiorari, finding that the said petition was only filed to circumvent the non-
appealable nature of small claims cases.

ISSUE:

WON THE RTC ERRED IN DISMISSING PETITIONER’S RECOURSE UNDER RULE 65 OF THE
RULES OF COURT ASSAILING THE PROPRIETY OF THE MTCC DECISION IN THE SUBJECT
SMALL CLAIMS CASE.

HELD: YES

Section 23 of the Rule of Procedure for Small Claims Cases states that: After the hearing, the court shall render
its decision on the same day, based on the facts established by the evidence (Form 13-SCC). The Clerk of Court
in the court docket for civil cases and a copy thereof forthwith served on the parties shall immediately enter the
decision. The decision shall be final and unappealable. Considering the final nature of a small claims case
decision under the above-stated rule, the remedy of appeal is not allowed, and the prevailing party may, thus,
immediately move for its execution. Nevertheless, the proscription on appeals in small claims cases, similar to
other proceedings where appeal is not an available remedy, does not preclude the aggrieved party from filing a
petition for certiorari under Rule 65 of the Rules of Court.

VINLUAN,  ALEXANDER  MARA  J.  2C  UST  LAW  


Fiorello R. Jose v. Roberto Alfuerto, et al., G.R. No. 69380, November 26, 2012
FACTS:

This case involves a parcel of land registered in the name of Rodolfo Chua Sing which he purchased in 1991.
Chua Sing leased the property to the petitioner. Their contract of lease was neither notarized nor registered with
the Parañaque City Registry of Deeds. Significantly, the respondents already occupied the property even before
the lease contract was executed. Soon after Chua Sing and the petitioner signed the lease contract, the petitioner
demanded in writing that the respondents vacate the property within 30 days and that they pay a monthly rental
of P1,000.00 until they fully vacate the property. The respondents refused to vacate and to pay rent.

MeTC - petitioner filed an ejectment case against the respondents; in this complaint, no mention was made of
any proceedings before the barangay. Jose then brought the dispute before the barangay for conciliation.
Thereafter, Jose was then able to file an amended complaint, incorporating the proceedings before the barangay
before the summons and copies of the complaint were served upon the named defendants. MeTC resolved the
case in the petitioner’s favor; held that the respondents had no right to possess the land and that their occupation
was merely by the owner’s tolerance. It further noted that the respondents could no longer raise the issue of
ownership, as this issue had already been settled: the respondents previously filed a case for the
annulment/cancellation of Chua Sing’s title before the RTC.

RTC - the respondents raised the issue, among others, that no legal basis exists for the petitioner’s claim that
their occupation was by tolerance, "where the possession of the defendants was illegal at the inception as
alleged in the complaint, there can be no tolerance." The RTC affirmed the MeTC decision, reiterating the
MeTC’s ruling that a case for ejectment was proper. The petitioner, as lessee, had the right to file the ejectment
complaint; the respondents occupied the land by mere tolerance and their possession became unlawful upon the
petitioner’s demand to vacate.

CA - reversed the RTC and MeTC decisions. It ruled that the respondents’ possession of the land was not by the
petitioner or his lessor’s tolerance. It defined tolerance not merely as the silence or inaction of a lawful
possessor when another occupies his land; tolerance entailed permission from the owner by reason of familiarity
or neighborliness.

ISSUE:

WON AN ACTION FOR UNLAWFUL DETAINER IS THE PROPER REMEDY.

HELD: NO.

Unlawful detainer is a summary action for the recovery of possession of real property. A lessor, vendor, vendee,
or other person against whom the possession of any land or building is unlawfully withheld after the expiration
or termination of the right to hold possession by virtue of any contract, express or implied may file this action.
In unlawful detainer, the possession of the defendant was originally legal, as his possession was permitted by
the plaintiff on account of an express or implied contract between them. However, the defendant’s possession
became illegal when the plaintiff demanded that the defendant vacate the subject property due to the expiration
or termination of the right to possess under the contract, and the defendant refused to heed such demand. A case
for unlawful detainer must be instituted one year from the unlawful withholding of possession.The allegations
in the complaint determine both the nature of the action and the jurisdiction of the court. The complaint must
specifically allege the facts constituting unlawful detainer. In the absence of these allegations of facts, an action
for unlawful detainer is not the proper remedy and the municipal trial court or the MeTC does not have
jurisdiction over the case.

VINLUAN,  ALEXANDER  MARA  J.  2C  UST  LAW  


Gegare v. CA, G.R. No. 83907, September 13, 1989
FACTS:

The case involves a land wherein the parties obtained equal shares by virtue of a Board Resolution, while
respondent does not contest any objections with regard to her share; the petitioner herein wants the entire
property. A reversion case was filed by the Republic of the Philippines against Paulino Elma in the CFI South
Cotabato, wherein in due course a decision was rendered on declaring the title of Paulino Elma null and void
and the same was ordered cancelled. The lot was reverted to the mass of public domain subject to disposition
and giving preferential right to its actual occupant, Napoleon Gegare.

Board of Liquidators - ruled in favor of the petitioner on the ground that he acquired the said land by way of
negotiated sale. The Board in Resolution favorably considered a motion for reconsideration filed by private
respondent. Thus, the Board directed the chief of LASEDECO to investigate the occupancy and area of the lot.
In this investigation, it was found that only private respondent was the actual occupant so the LASEDECO chief
recommended the division of the property between petitioner and private respondent. The Board Resolution on
approving said recommendation by dividing the lot equally between the parties at 135.5 square meters each to
be disposed to them by negotiated sale.

RTC - Petitioner filed an action for "Annulment and Cancellation of Partition of Lot to Declare them Null and
Void" against private respondent and the Board. Private respondent filed a motion to dismiss the complaint on
the following grounds: (1) lack of jurisdiction over the subject matter; (2) petitioner has no capacity to sue; (3)
petitioner is not a real party-in-interest; and (4) the action is barred by prior judgment. Private respondent added
another ground (5) lack of conciliation efforts.

CA - respondent filed a petition for certiorari and prohibition, in which the said petition was granted.

ISSUE:

WON THE CA ERRED IN DECIDING WITHOUT FIRST SERVING THE SUMMONS AND A COPY
OF THE PETITION TO THE RESPONDENT (NOW PETITIONER)

WON THE COURT ERRED IN DISMISSING THE SAID CASE FOR THE RESPONDENTS FAILED
TO COMPLY WITH THE PROVISIONS OF PD 1508.

HELD:

1. NO. Petitioner's counsel was also served a copy of the resolution. Moreover, petitioner's counsel filed a
motion seeking a reconsideration of the decision of respondent court which was denied. Therefore, petitioner
voluntarily submitted to the jurisdiction of the respondent court and was never deprived of due process.

2. NO. Section 6. Conciliation, pre-condition to filing of complaint. No complaint, petition, action or


proceeding involving any matter within the authority of the Lupon as provided in Section 2 hereof shall be filed
or instituted in court or any other government office for adjudication unless there has been a confrontation of
the parties before the Lupon Chairman or the Pangkat and no conciliation or settlement has been reached as
certified by the Lupon Secretary or the Pangkat Secretary attested by the Lupon or Pangkat Chairman, or unless
the settlement has been repudiated. The purpose of this confrontation is to enable the parties to settle their
differences amicably. If the other only contending party is the government or its instrumentality or subdivision
the case falls within the exception but when it is only one of the contending parties, a confrontation should still
be undertaken among the other parties.
VINLUAN,  ALEXANDER  MARA  J.  2C  UST  LAW  
CLAUDIA RIVERA SANCHEZ v. HONORABLE MARIANO C. TUPAS, Presiding Judge of the
Regional Trial Court, Branch XII of Davao City and Private Respondent ALFONSO ESCOVILLA, G.R.
No. 76690 February 29, 1988

FACTS:
The petitioner and private respondents are both occupants of a public agricultural land located in Davao City.
Sanchez claims that the area of 450 square meters, more or less, has been in her possession since 1947, long
before Escovilla came in and occupied another portion of Lot 595. On the other hand, Escovilla stated that the
area being claimed by petitioner is a part of his three-fourth (3/4) of a hectare parcel, the right to which he
acquired from its former possessor and owner of the improvements thereon. Sometime in 1966, out of charity
and upon their agreement that petitioner will vacate the premises upon demand, he granted petitioner's request
to build her house inside the land occupied by him.

City Court of Davao - Escovilla filed an ejectment case; court rendered judgment through a “Judgment by
Compromise”.

RTC - Sanchez filed a petition to annul judgment; alleged that she was an illiterate and did not know that what
he counsel presented which she had signed using her thumbmark was a Compromise Agreement which
recognized Escovilla’s prior occupancy of the land in dispute; that she had never intended to recognize the
private respondent as having prior possession and occupancy of the land; that the private respondent caused the
survey of the entire Lot No. 595, which survey became null and void after the same was formally opposed. A
motion to dismiss was filed based on the grounds – the filing of the said motion was beyond the period
prescribed and such states no cause of action. RTC sustained private respondent's Motion to Dismiss by
dismissing the case for lack of cause of action or prematurity for not having passed the Barangay Court.

ISSUE:
WON THE RTC GRAVELY ERRED IN DISMISSING THE PETITION FOR ANNULMENT OF
JUDGMENT BECAUSE IT DID NOT PASS BARANGAY CONCILIATION.
WON THE COMPROMISE AGREEMENT SHALL BE NULL AND VOID.

HELD:
1. YES. PD 1508 requires that the parties who actually reside in the same city or municipality should bring their
controversy first to the Barangay Court for possible amicable settlement before filing a complaint in court. This
requirement is compulsory and non-compliance of the same could affect the sufficiency of the cause of action
and make the complaint vulnerable to dismissal on the ground of lack of cause of action or prematurity. In the
instant case, it will be noted that the ejectment case in the City Court of Davao, was filed on September 18,
1980, when PD 1508 was already enforced. However, the records do not show that there was an opposition to
the filing of the said ejectment case on the ground that the dispute had not been submitted to the Barangay Court
for possible amicable settlement under P.D. 1508. The only logical conclusion therefore is that either such
requirement had already been complied with or had been waived

2. NO. While there can be no question as to the right of any person adversely affected by a judgment to
maintain an action and to have the decision declared a nullity, such an action to annul a judgment upon the
ground of fraud, will not lie unless the fraud be extrinsic or collateral and committed by the adverse party, not
by one's own counsel. Said ruling was reiterated in a subsequent case where it was stressed that the fraud
mentioned in Rule 38 is the fraud committed by the adverse party. In the instant case, petitioner, in her action
for annulment of judgment, never made any allegation that private respondent had anything to do with such
actuation other lawyer. Such being the case, the most that she has is an action against her own lawyer and not
against the private respondent.
VINLUAN,  ALEXANDER  MARA  J.  2C  UST  LAW  
PETRA VDA. DE BORROMEO, vs. HON. JULIAN B. POGOY, Municipality/City Trial Court of Cebu
City, and ATTY. RICARDO REYES, G.R. No. L-63277. November 29, 1983
FACTS:
The intestate estate of the late Vito Borromeo is the owner of a building bearing the deceased’s name, located at
Cebu City. Said building has been leased and occupied by petitioner Petra Vda. de Borromeo at a monthly
rental of P500.00 payable in advance within the first 5 days of the month. Private respondent Atty. Ricardo
Reyes, administrator of the estate and a resident of Cebu City, served upon petitioner a letter demanding that
she pay the overdue rentals corresponding to the period from March to September 1982, and thereafter to vacate
the premises.

MTC - petitioner failed to do so, Atty. Reyes instituted an ejectment case. Petitioner moved to dismiss the case,
advancing, among others, the want of jurisdiction of the trial court. Pointing out that the parties are residents of
the same city, as alleged in the complaint, petitioner contended that the court could not exercise jurisdiction
over the case for failure of respondent Atty. Reyes to refer the dispute to the Barangay Court. Thus, the said
court denied the motion to dismiss.

Unable to secure a reconsideration of said order, petitioner came to this Court through this petition for
certiorari. In both his comment and memorandum, private respondent admitted not having availed himself of
the barangay conciliation process, but justified such omission by citing paragraph 4, section 6 of PD 1508
which allows the direct filing of an action in court where the same may otherwise be barred by the Statute of
Limitations, as applying to the case at bar.

ISSUE:
WON BARANGAY CONCILIATION IS NECESSARY AS A CONDITION PRECEDENT IN THIS
CASE.

HELD: NO
The excuse advanced by private respondent is unsatisfactory. Under Article 1147 of the Civil Code, the period
for filing actions for forcible entry and detainer is one year, and this period is counted from demand to vacate
the premises. In the case at bar, the letter-demand was dated August 28, 1982, while the complaint for
ejectment was filed in court on September 16, 1982. Between these two dates, less than a month had elapsed,
thereby leaving at least eleven (11) full months of the prescriptive period provided for in Article 1147 of the
Civil Code. Under the procedure outlined in Section 4 of PD 1508, 3 the time needed for the conciliation
proceeding before the Barangay Chairman and the Pangkat should take no more than 60 days. Giving private
respondent nine (9) months — ample time indeed — within which to bring his case before the proper court
should conciliation efforts fail. Thus, it cannot be truthfully asserted, as private respondent would want the
Court to believe, that his case would be barred by the Statute of Limitations if he had to course his action to the
Barangay Lupon. With certain exceptions, PD 1508 makes the conciliation process at the Barangay level a
condition precedent for filing of actions in those instances where said law applies - that all the barangays within
your respective jurisdictions have organized their Lupons provided for in PD No. 1508, otherwise known as the
Katarungang Pambarangay Law, in implementation of the barangay system of settlement of disputes, you are
hereby directed to desist from receiving complaints, petitions, actions or proceedings in cases falling within the
authority of said Lupons." While respondent acknowledged said Circular, , he nevertheless chose to overlook
the failure of the complaint to allege compliance with the requirement of PD 1508. Neither did he cite any
circumstance as would place the suit outside the operation of said law. Instead, he insisted on relying upon the
pro tanto presumption of regularity in the performance by the clerk of court of his official duty, which to
Court’s mind has been sufficiently overcome by the disclosure by the Clerk of Court that there was no
certification to file action from the Lupon or Pangkat secretary attached to the complaint. Be that as it may, the
instant petition should be dismissed. Under Section 4(a) of PD No. 1508, referral of a dispute to the Barangay
Lupon is required only where the parties thereto are "individuals."

VINLUAN,  ALEXANDER  MARA  J.  2C  UST  LAW  


Elmer Peregrina, Adelaida Peregrina and Cecilia Peregrina v. Hon. Domingo Panis, G.R.No. L-56011
October 31, 1984
FACTS:

Spouses Sanchez filed a complaint against the petitioners in a civil action for damages for alleged disrespect for
the dignity, privacy and peace of mind of the spouses under Article 26 of the Civil Code, and for alleged
defamation under Article 33 of the same Code. Admittedly, the parties are actual residents of the same barangay
in Olongapo City. In fact, they are neighbors. Unquestionably, too, no conciliation proceedings were filed
before the Lupon. Thus the complaint filed by the petitioners is silent regarding compliance with the mandatory
requirement, nor does it allege that the dispute falls within the excepted cases.

LOWER COURT - petitioners moved for the dismissal of the complaint; holding that under Section 6(3) of P.D.
No. 1508, the parties may go directly to the Courts if the action is coupled with a provisional remedy such as
preliminary attachment. Dismissed the Complaint for failure of the spouses to comply with the pre-condition for
amicable settlement under P.D. No. 1508, stating that the application for a provisional remedy was merely an
afterthought.

MOTION FOR RECONSIDERATION - respondent Judge denied the Motion to Dismiss on the ground that
under Rule 57, Section 1 of the Rules of Court, the application for attachment can be made at the
commencement of the action or any time thereafter.

ISSUE:
WON BARANGAY CONCILIATION IN THIS CASE IS A PRECONDITION FOR FILING A
COMPLAINT IN THE RTC.

HELD: YES.

Section 3 of P.D. No. 1508 specifically provides: Disputes between or among persons actually respectively in
the same barangay shall be brought for amicable settlement before the Lupon of said barangay. Section 6.
Conciliation, pre-condition to filing of complaint. — No complaint, petition, action or proceeding involving any
matter within the authority of the Lupon as provided. in Section 2 hereof shall be filed or instituted in court or
any other government office for adjudication unless there has been a confrontation of the parties before the
Lupon Chairman or the Pangkat and no conciliation or settlement has been reached as certified by the Lupon
Secretary or the Pangkat Secretary, attested by the Lupon or Pangkat Chairman, or unless the settlement has
been repudiated. The parties herein fall squarely within the ambit of P.D. No. 1508. They are actual residents in
the same barangay and their dispute does not fall under any of the excepted cases. It will have to be held,
therefore, that respondent Judge erred in reconsidering his previous Order of dismissal on the ground that the
provisional remedy of attachment was seasonably filed. Not only was the application for that remedy merely an
afterthought to circumvent the law, but also, fundamentally, a Writ of Attachment is not available in a suit for
damages where the amount, including moral damages, is contingent or unliquidated. Prior referral to the Lupon
for conciliation proceedings, therefore, was indubitably called for.

VINLUAN,  ALEXANDER  MARA  J.  2C  UST  LAW  


Librada M. Aquino vs Ernest S. Aure, G.R. No. 153567, February 18, 2008

FACTS:

Aure and Es Aure Lending Investors alleged that they acquired the subject property from Spouses Aquino. Aure
alleged that after they paid the spouses, the latter refused to vacate the property.
MeTC - Aure filed a complaint for ejectment against the spouses; spouses counter that Aure lacks cause of
action because the latter does not have any legal right over the subject property. They reasoned that Aure did
not comply with their MoA. It ruled that since the question of ownership was put in issue, the action was
converted to a suit, which is incapable of pecuniary estimation, which properly rests within the original
exclusive jurisdiction of the RTC. It also ruled that non-compliance with barangay conciliation process lead to
the dismissal of the complaint.

RTC - affirmed the dismissal of the complaint on the same ground that the dispute was not brought before the
Barangay Council for conciliation before it was filed in court.

CA - reversed the MeTC and RTC decisions and remanded the case to the MeTC for further proceedings.

ISSUES:

WON NON-COMPLIANCE WITH THE BARANGAY CONCILIATION PROCEEDINGS IS A


JURISDICTIONAL DEFECT THAT WARRANTS THE DISMISSAL OF THE COMPLAINT.

WON THE ALLEGATION OF OWNERSHIP OUSTS THE METC OF ITS JURISDICTION OVER
AN EJECTMENT CASE.

HELD:

1. NO. Conciliation process is not a jurisdictional requirement, so that non-compliance therewith cannot affect
the jurisdiction, which the court has otherwise acquired over the subject matter or over the person of the
defendant. However, non-compliance will not prevent a court from exercising its power of adjudication over the
case before it where the defendants failed to object to such exercise of jurisdiction in their answer and even
during the entire proceedings a quo. In the present case, Aquino cannot be allowed to attack the jurisdiction of
the MeTC after having participated in the proceedings without objecting. By Aquino’s failure to seasonably
object to the deficiency in the complaint, she is deemed to have waived any defect. The issue of non-compliance
should be pleaded in the answer.

2. NO. Jurisdiction in ejectment cases is determined by the allegations pleaded in the complaint. As long as
these allegations demonstrate a cause of action either for forcible entry or for unlawful detainer, the court
acquires jurisdiction over the subject matter. The law provides instead that when the question of possession
cannot be resolved without deciding the issue of ownership, the issue of ownership shall be resolved only to
determine the issue of possession.

VINLUAN,  ALEXANDER  MARA  J.  2C  UST  LAW  


Crisanta Alcaraz Miguel v. Jerry D. Montanez, G.R. No. 191336, January 25, 2012

FACTS:

The respondent secured a loan of (P143,864.00), payable in one (1) year, or until February 1, 2002, from the
petitioner. The respondent gave as collateral therefor his house and lot. Due to the respondents failure to pay the
loan, the petitioner filed a complaint against the respondent before the Lupong Tagapamayapa of Barangay. The
parties entered into a Kasunduang Pag-aayos wherein the respondent agreed to pay his loan in installments in
the amount of P2,000.00/month, and in the event the house and lot given as collateral is sold, the respondent
would settle the balance of the loan in full. However, the respondent still failed to pay, and on December 13,
2004, the Lupong Tagapamayapa issued a certification to file action in court in favor of the petitioner.
MeTC - petitioner filed a complaint for Collection of Sum of Money. Respondent raised the defense of improper
venue considering that the petitioner was a resident of Bagumbong, Caloocan City while he lived in San Mateo,
Rizal.

RTC – affirmed the MeTC.

CA - granted the petition of the respondent, reversing and setting aside the judgment made RTC. A new
judgment is entered dismissing respondents complaint for collection of sum of money, without prejudice to her
right to file the necessary action to enforce the Kasunduang Pag-aayos.

ISSUES:

WON A COMPLAINT FOR SUM OF MONEY IS THE PROPER REMEDY FOR THE PETITIONER,
NOTWITHSTANDING THE KASUNDUAN PAG-AAYOS.

WON THE CA SHOULD HAVE DECIDED THE CASE ON THE MERITS RATHER THAT REMAND
THE CASE FOR THE ENFORCEMENT OF THE KASUNDUANG PAG-AAYOS.

HELD:

1. YES. Due to the respondent failed to comply with the terms of the Kasunduang Pag-aayos, said agreement is
deemed rescinded pursuant to Article 2041 of the New Civil Code and the petitioner can insist on his original
demand. Perforce, the complaint for collection of sum of money is the proper remedy. The petitioner points out
that the cause of action did not arise from the Kasunduang Pag-aayos but on the respondents’ breach of the
original loan agreement. This Court agrees with the petitioner. It is true that an amicable settlement reached at
the barangay conciliation proceedings, like the Kasunduang Pag-aayos in this case, is binding between the
contracting parties and, upon its perfection, is immediately executory insofar as it is not contrary to law, good
morals, good customs, public order and public policy. In the instant case, the respondent did not comply with
the terms and conditions of the Kasunduang Pag-aayos. Such non-compliance may be construed as repudiation
because it denotes that the respondent did not intend to be bound by the terms thereof, thereby negating the very
purpose for which it was executed.

2. YES. Considering that the Kasunduang Pag-aayos is deemed rescinded by the non-compliance of the
respondent of the terms thereof, remanding the case to the trial court for the enforcement of said agreement is
clearly unwarranted.

VINLUAN,  ALEXANDER  MARA  J.  2C  UST  LAW  


Remedio Flores v. Hon. Judge Heila Mallare-Phillipps, Ignacio Binongcal and Fernando Calion
G.R. No. L-66620, September 24, 1986

FACTS:

Respondent Binongcal filed a Motion to Dismiss on the ground of lack of jurisdiction since the amount of the
demand was only P11,643.00 and refused to pay representing cost of truck tires which he purchased on credit.
Fernando Calion allegedly indebted to petitioner joined in moving for the dismissal of the complaint on the
ground of lack of jurisdiction.

RTC - Counsel for petitioner opposed the Motion to Dismiss. The trial court dismissed the complaint for lack of
jurisdiction.

CA - Petitioner appealed by certiorari from the order of Judge Mallare-Phillipps who dismissed his complaint
for lack of jurisdiction.

ISSUE:

WON THE CASE SHOULD BE DISMISSED FOR LACK OF JURISDICTION

HELD: YES

In cases of permissive joinder of parties, whether as plaintiffs or as defendants, under Section 6 of Rule 3, the
total of all the claims shall now furnish the jurisdictional test. Needless to state, if the causes of action are
separate and independent, their joinder in one complaint is permissive and not mandatory, and any cause of
action where the amount of the demand is twenty thousand pesos or less may be the subject of a separate
complaint filed with a metropolitan or municipal trial court. In the case at bar, the lower court correctly held
that the jurisdictional test is subject to the rules on joinder of parties pursuant to Section 5 of Rule 2 and Section
6 of Rule 3 of the Rules of Court and that, after a careful scrutiny of the complaint, it appears that there is a
misjoinder of parties for the reason that the claims against respondents Binongcal and Calion are separate and
distinct and neither of which falls within its jurisdiction.

VINLUAN,  ALEXANDER  MARA  J.  2C  UST  LAW  


Ramon Ching and Po Wing Properties v. Hon. Jansen Rodriguez et al., G.R. NO. 192828 November 28,
2011
FACTS:

The private respondents filed a Complaint against the Ramon, Po Wing Properties and Stronghold Insurance
Company, Global Business Bank, Inc. (formerly PhilBank), Elena Tiu Del Pilar, Asia Atlantic Resources
Ventures, Inc., Registers of Deeds of Manila and Malabon, and all persons claiming rights or titles from Ramon
and his successors-in-interest. In the Complaint, the respondents alleged several causes of action. Since Ramon
is at large, his wife, Belen Dy Tan Ching, now manages Antonio's estate. She has no intent to convey to the
respondents their shares in the estate of Antonio. The Amended Complaint, which impleaded Metrobank as
successor-in-interest of Global, also added a seventh cause of action relative to the existence of a Certificate of
Premium Plus Acquisition (CPPA) in the amount of P4,000,000.00 originally issued by PhilBank to Antonio.
The respondents prayed that they be declared as the rightful owners of the CPPA and that it be immediately
released to them. Alternatively, the respondents prayed for the issuance of a hold order relative to the CPPA to
preserve it during the pendency of the case. Petitioners filed a Motion to Dismiss on the respondents' Amended
Complaint on the alleged ground of the RTC's lack of jurisdiction over the subject matter of the Complaint. The
petitioners argued that since the Amended Complaint sought the release of the CPPA to the respondents, the
latter's declaration as heirs of Antonio, and the propriety of Ramon's disinheritance, the suit partakes of the
nature of a special proceeding and not an ordinary action for declaration of nullity. Hence, jurisdiction pertains
to a probate or intestate court and not to the RTC acting as an ordinary court.

RTC - denied the petitioners' Motion to Dismiss on the ground that the action delves mainly on the question of
ownership of the properties described in the complaint. Also, the issue of disinheritance can be fully settled
after a trial on the merits.

CA – affirmed the decision of the RTC.

ISSUE:

WON RTC SHOULD HAVE GRANTED THE MOTION TO DISMISS ON THE GROUND THAT THE
FILIATION WITH ANTONIO OF RAMON AND THE DETERMINATION OF THE EXTENT OF
ANTONIO’S ESTATE CAN ONLY BE RESOLVED IN A SPECIAL PROCEEDINGS.

HELD: NO

RTC acted correctly by denying the said motion to dismiss. Although the respondents' Complaint and Amended
Complaint sought, among others, the disinheritance of Ramon and the release in favor of the respondents of the
CPPA now under Metrobank's custody, remains to be an ordinary civil action, and not a special proceeding
pertaining to a settlement court. To initiate a special proceeding, a petition and not a complaint should be filed.
An action for reconveyance and annulment of title with damages is a civil action, whereas matters relating to
settlement of the estate of a deceased person such as advancement of property made by the decedent, partake of
the nature of a special proceeding, which concomitantly requires the application of specific rules as provided for
in the Rules of Court. Under Article 916 of the NCC, disinheritance can be effected only through a will wherein
the legal cause therefor shall be specified. While the respondents in their Complaint and Amended Complaint
sought the disinheritance of Ramon, no will or any instrument supposedly effecting the disposition of Antonio's
estate was ever mentioned.

VINLUAN,  ALEXANDER  MARA  J.  2C  UST  LAW  


Paglaum Management and Development Corp. and Health Marketing Technologies, Inc., v. Union Bank
of the Philippines, Notary Public John Doe, and
Register of Deeds of Cebu City and Cebu Province
G.R. No. 179018, April 17, 2013
FACTS:

Union Bank of the Philippines extended HealthTech a credit line in the amount of ₱10 Million. To secure the
obligation, PAGLAUM executed three Real Estate Mortgages over its three real properties in Cebu Province on
behalf of HealthTech and in favor of Union Bank. The original contract states that the venue of all suits and
actions shall be commenced in “Makati, Metro Manila, or in the place where any of the Mortgaged Properties is
located, at the absolute option of the Mortgagee, the parties hereto waiving any other venue.” Health Tech and
Union Bank thereafter increased the credit line but omitted Makati City as the venue in the subsequent
agreements. A restructuring agreement was entered stating that all actions shall be commenced in in Makati
City, with both parties waiving any other venue.

RTC - Health Tech defaulted still. Union Bank extrajudicially foreclosed the three mortgaged properties. Health
Tech thereafter filed a Complaint for Annulment of Sale and Titles with Damages. Union Bank filed a motion
to dismiss based on improper venue, among others. The RTC dismissed the case.

CA – affirmed the decision of the RTC.

ISSUE:

WON MAKATI CITY IS THE PROPER VENUE TO ASSAIL THE FORECLOSURE OF THE
SUBJECT REAL ESTATE MORTGAGE.

HELD: YES

The complaint filed in this case is a real action. Accordingly, Sec. 1, Rule 4 provides that the said case should
be tried “in the proper court which has jurisdiction over the area wherein the real property involved, or a portion
thereof, is situated.” But Sec. 3 (b), Rule 4 allows real actions to bee commenced and tried in a court other than
where the property is situated in instances where the parties have previously and validly agreed in writing on the
exclusive venue thereof.

In this case, such an agreement exists. The Restructuring Agreement clearly reveals the intention of the parties
to implement a restrictive venue stipulation, which applies not only to the principal obligation, but also to the
mortgages. The phrase waiving any other venue plainly shows that the choice of Makati City as the venue for
actions arising out of or in connection with the Restructuring Agreement and the Collateral, with the Real Estate
Mortgages being explicitly defined as such, is exclusive.

VINLUAN,  ALEXANDER  MARA  J.  2C  UST  LAW  


Sps. Domingo M. Belen and Dominga P. Belen vs. Hon. Pablo R. Chavez et al., G.R. No. 175334, March
26, 2008
FACTS:
Sps. Pacleb filed a petition against Sps. Belen before the RTC Batangas. It was alleged that spouses Pacleb
secured a judgment by default in a case rendered by a Judge John W. Green of the Superior Court of the State of
California. In this judgment, spouses Belen were ordered to pay Sps. Pacleb the amount of $56,204.69
representing loan repayment and share in the profits plus interest and costs of suit. The summons was served on
sps. Belen’s address in San Gregorio, Alaminos, Laguna, and was received by Marcelo M. Belen.

RTC – Sps. Belen failed to attend the scheduled pre-trial conference, and because of this, the RTC ordered the
ex parte presentation of evidence for sps. Pacleb before the branch clerk of court. Atty. Alcantara filed a motion
to dismiss, citing the judgment of dismissal issued by the Superior Court of the State of California, which
allegedly dismissed the case before it involving the same parties. RTC denied the motion to dismiss; sought the
reinstatement of the motion to dismiss by attaching a copy of the said foreign judgment. Sps. Pacleb filed a
motion for the amendment stating that they were constrained to withdraw their complaint because of the
prohibitive cost of litigation. Sps. Belen and Atty. Alcantaea still failed to appear at the re-scheduled pre-trial,
thus, RTC declare them in default and allowed sps. Belen to present evidence ex parte. RTC rendered a
Decision ordering sps. Belen to pay sps. Pacleb.

After the promulgation of the RTC decision, Sps. Pacleb filed an ex parte motion for preliminary attachment.
Afterward, RTC directed the issuance of a writ of execution. the real properties belonging to spouses Belen
were levied upon and the public auction. Subsequently, Atty. Culvera entered his appearance as counsel for sps.
Belen. He then filed a Motion to Quash Writ of Execution, and A Notice of Appeal from the RTC Decision.
RTC denied the motion. CA - dismissed the Petition for Certiorari.

ISSUES:
WON THE RTC ACQUIRED JURISDICTION OVER SPS. BELEN THROUGH EITHER THE
PROPER SERVICE OF SUMMONS OR THE APPEARANCE OF THE LATE ATTY. ALCANTARA.
WON THERE WAS A VALID SERVICE OF THE COPY OF THE RTC DECISION ON THEM.

HELD:
1. YES. Courts acquire jurisdiction over the plaintiffs upon the filing of the complaint. On the other hand,
jurisdiction over the defendants in a civil case is acquired either through the service of summons upon them or
through their voluntary appearance in court and their submission to its authority. Jurisdiction over the person of
a resident defendant who does not voluntarily appear in court can be acquired by personal service of summons.
If he cannot be personally served, substituted service may be made. However, in an action in personam in which
the defendant is a non-resident who does not voluntarily submit himself to the authority of the court, personal
service of summons within the state is essential to the acquisition of jurisdiction over his person. This is only
possible if the defendant is physically present in the country. Otherwise, the court cannot acquire jurisdiction
over his person and therefore cannot validly try and decide the case against him. The action filed against sps.
Belen is in the nature of an action in personam because the latter are suing to enforce their personal rights under
said judgment.

2. NO. As a general rule, when a party is represented by counsel of record, service of orders and notices must
be made upon said attorney and notice to the client and to any other lawyer, not the counsel of record, is not
notice in law. The exception to this rule is when the court has ordered service upon the party himself. In this
case, a copy of the RTC decision was sent first to Atty. Alcantara, petitioners’ counsel of record, but it was
returned unserved in view of the demise of Atty. Alcantara. The subsequent service on petitioners’ purported
“last known address” by registered mail is defective because it does not comply with the requisites on service
by registered mail, which presupposes that the present address of the party is known, and the person receiving it
must be authorized to receive such service.
VINLUAN,  ALEXANDER  MARA  J.  2C  UST  LAW  
Planters Development Bank v. Julie Chandumal, G. R. No. 195619,
FACTS:

BF Homes and Julie Chandumal entered into a contract to sell a parcel of land located in Las Pinas. Later, BF
Homes sold to PDB all its rights over the contract. Chandumal paid her monthly amortizations until she
defaulted in her payments. So, PDB sent a notice to Chandumal with a demand to vacate the land within
30days, otherwise all of her rights will be extinguished and the contract will be terminated and deemed
rescinded. In spite of the demand, Chandumal failed to settle her account.

RTC - PDB filed an action for judicial confirmation of notarial rescission and delivery of possession but still
Chandumal refused to do so. Summons were then issued and served by deputy sheriff Galing but its was
unavailing as she was always out of her house on the dates the summons were served. RTC then issued an order
granting the motion of PDB. Chandumal filed an urgent motion to set aside order of default and to admit
attached answer. Chandumal said that she did not receive the summons and was not notified of the same and her
failure to file an answer within the reglementary period was due to fraud. RTC denied Chandumal's motion to
set aside the order of default.

CA - Chandumal appealed to the CA. CA nullified the RTC's decision.

ISSUE:

WON THERE WAS A VALID SUBSTITUTED SERVICE OF SUMMONS.


WON CHANDUMAL VOLUNTARILY SUBMITTED TO THE JURISDICTION OF THE RTC.
WON THERE WAS A PROPER RESCISSION BY NOTARIAL ACT OF THE CONTRACT TO SELL.

HELD:

1. NO. Correctly ruled that the sheriff’s return failed to justify a resort to substituted service of summons.
According to the CA, the Return of Summons does not specifically show or indicate in detail the actual exertion
of efforts or any positive step taken by the officer or process server in attempting to serve the summons
personally to the defendant.

2. YES. The Court notes that aside from the allegation that she did not receive any summons, Chandumal’s
motion to set aside order of default and to admit attached answer failed to positively assert the trial court lack of
jurisdiction. In fact, what was set forth therein was the substantial claim that PDB failed to comply with the
requirements of R.A. No. 6552 on payment of cash surrender value, which already delves into the merits of
PDB’s cause of action. In addition, Chandumal even appealed the RTC decision to the CA, an act which
demonstrates her recognition of the trial court’s jurisdiction to render said judgment.

3. NO. R.A. No. 6552 recognizes the right of the seller to cancel the contract but any such cancellation must be
done in conformity with the requirements therein prescribed. In addition to the notarial act of rescission, the
seller is required to refund to the buyer the cash surrender value of the payments on the property. The actual
cancellation of the contract can only be deemed to take place upon the expiry of a thirty (30)-day period
following the receipt by the buyer of the notice of cancellation or demand for rescission by a notarial act and the
full payment of the cash surrender value.

VINLUAN,  ALEXANDER  MARA  J.  2C  UST  LAW  


Yu v. Pacleb, etc., G.R. No. 172172, February 24, 2009
FACTS:

Respondent and his wife, are the registered owners of a parcel of land, which became the subject of 3
documents purporting to transfer its ownership. a Deed of Absolute Sale was entered into between Spouses
Baltazar N. Pacleb and Angelita Chan and Rebecca Del Rosario. Subsequently, a Deed of Absolute Sale was
entered into between Rebecca Del Rosario and Ruperto L. Javier, and thereafter a Contract to Sell was entered
into between Javier and petitioner spouses Ernesto V. Yu and Elsie Ong Yu. Javier undertook to deliver
possession of the Langcaan Property and to sign a deed of absolute sale within 30 days from execution of the
contract. All these aforementioned sales were not registered.

RTC (1) – sps. Yu filed a Complaint for specific performance and damages against Javier to compel the latter to
deliver to them ownership and possession, as well as title to the subject property. Javier failed to comply with
his obligations. Javier did not appear in the proceedings and was declared in default. Judgment was eventually
rendered against him by the RTC, and the same attained finality. Petitioner sps. and Ramon Pacleb and the
latter’s wife, then executed an agreement whereby petitioner spouses paid Ramon the amount of P500,000 in
exchange for the waiver of his tenancy rights over the property.

Respondent filed a Complaint for annulment of deed of sale and other documents arising from it. He alleged
that the deed of sale purportedly executed between him and his late first wife and Rebecca Del Rosario was
spurious as their signatures thereon were forgeries. Respondent moved to have summons served upon Rebecca
Del Rosario by publication since the latter’s address could not be found. Thus, the RTC denied.

MTC – petitioner spouses filed an action for forcible entry against respondent. MTC ruled in favor of petitioner
spouses, which decision was affirmed by the Regional Trial Court. However, the Court of Appeals set aside the
decisions of the lower courts and found that it was respondent who had prior physical possession of the property
as shown by his payment of real estate taxes thereon.

RTC (2) - dismissed respondent’s case and held that petitioner spouses are purchasers in good faith. Further, it
held that the previous decision on petitioner spouses’ action for specific performance against Javier is already
final and can no longer be altered. Accordingly, it ordered the cancellation of the certificate of title of the
property in the name of respondent and the issuance of a new title in the name of petitioner spouses.

CA - reversed and set aside the decision of the trial court. The Court of Appeals ruled that petitioner spouses
are not purchasers in good faith and that the previous decision did not transfer ownership of the property to
them.

ISSUES:

WON PETITIONER SPOUSES ARE INNOCENT PURCHASERS FOR VALUE IN GOOD FAITH.

WON OWNERSHIP OVER THE SUBJECT PROPERTY WAS PROPERLY VESTED IN


PETITIONER SPOUSES AND SUCH IS BINDING & CONCLUSIVE TO THE RESPONDENTS.

HELD:

1. NO. At the outset, it must be noted that in petitioner Yu’s testimony, he stated that he inspected the property
and talked with the tenant, Ramon, before he purchased the same. However, in his Complaint for specific
performance and damages which he filed against Javier, he alleged that it was only after he had entered into an
VINLUAN,  ALEXANDER  MARA  J.  2C  UST  LAW  
Agreement for the sale of the property and his initial payment that he discovered that the property was indeed
being tenanted by Ramon who lives in the said farm. Second, a perusal of the two deeds of absolute sale reveals
that they were executed only about two (2) months apart and that they contain identical provisions. Third, it is
undisputed that the property is in the possession of Ramon, the son of the registered owner. Regardless of the
representations given by the latter, this bare fact alone should have made petitioner spouses suspicious as to the
veracity of the alleged title of their vendor.

2. NO. It has been held in an unbroken string of cases that an action for specific performance is an action in
personam. It is well-settled that an action for specific performance praying for the execution of a deed of sale in
connection with an undertaking in a contract, such as the contract to sell, in this instance, is an action in
personam. The action for specific performance and damages filed by petitioner spouses against Javier is to
compel performance of the latter’s undertakings under their Contract to Sell. As correctly held by the Court of
Appeals, its object is to compel Javier to accept the full payment of the purchase price, and to execute a deed of
absolute sale over the property in their favor. The obligations of Javier under the contract to sell attach to him
alone, and do not burden the subject property. Being a judgment in personam, it is binding only upon the parties
properly impleaded therein and duly heard or given an opportunity to be heard. Therefore, it cannot bind
respondent since he was not a party therein. Neither can respondent be considered as privy thereto since his
signature and that of his late first wife, Angelita Chan, were forged in the deed of sale.

VINLUAN,  ALEXANDER  MARA  J.  2C  UST  LAW  


Goodland Company, Inc. v. Asia United Bank, et al., G.R. No. 195561
FACTS:

Petitioner mortgaged its two parcels of land situated in Sta. Rosa, Laguna and covered by Transfer Certificate of
Title. The Third Party REM secured the loans extended by respondent AUB to Smartnet, Inc. doing business as
Smartnet Philippines, under the latter’s P250,000,000.00 Omnibus Credit Line with AUB. Petitioner repudiated
the REMs by claiming that AUB and its officers unlawfully filled up the blank mortgage forms and falsified the
entries therein. The Laguna properties were the subject of two suits filed by petitioner to forestall their
imminent foreclosure, and petitioner involving the Makati property, which is the subject of the present case,
likewise instituted similar actions. However, the only subject of the present petition is the Makati Property. The
Laguna Properties were the subject of separate petitions.

RTC - Petitioner instituted two suits involving the Makati Property. The first suit filed by petitioner was an
action for an annulment of the REM covering the Makati Property on the ground of its fraudulent and irregular
execution and registration. Second suit filed by petitioner prayed for injunctive relief and/or nullification of the
extrajudicial foreclosure sale which petitioner alleged to be procedurally and legally defective. The RTC issued
an Order denying petitioner’s application for the issuance of a writ of preliminary injunction, as well as
respondents’ motion to dismiss based on forum shopping, non-payment of correct docket fees and failure to
state a cause of action. However, the court reserved the issuance of the corresponding order requiring petitioner
to pay the appropriate docket fees after respondents shall have submitted what they believed should have been
the correct computation thereof.

Respondents moved to dismiss with prejudice the two cases on the grounds of forum shopping, and that no
jurisdiction was acquired by the RTC for failure to pay the proper docket and other legal fees.

CA - Petitioner again filed separate appeals, which were docketed under only one case. However, it sustained
the dismissal made by trial court on the ground of forum shopping, but not on the ground of improper docket
and other legal fees

ISSUE:
WON PETITIONER VIOLATED THE RULE AGAINST FORUM SHOPPING.

HELD: YES

For forum shopping can be said to exist, the following must concur: (1) identity of parties, or at least such
parties as represent the same interests in both actions, (2) identity of rights asserted and relief prayed for, the
relief being founded on the same facts, and (3) the identity of the two preceding particulars is such that any
judgment rendered in the other action will, regardless of which party is successful, amount to res judicata in the
action under consideration. The Court ruled that these elements are present in the instant case. The prayer for
relief in the two cases was based on the same attendant facts in the execution of REMs over petitioner’s
properties in favor of AUB. While the extrajudicial foreclosure of mortgage, consolidation of ownership in
AUB and issuance of title in the latter’s name were set forth only in the second case, these were simply the
expected consequences of the REM transaction in the first case. With respect to identity of cause of action, a
cause of action is defined in Section 2, Rule 2 of the Rules of Court as the act or omission by which a party
violates the right of another. The Court made reference to the test in determining whether or not the causes of
action in the first and second cases are identical, to wit: would the same evidence support and establish both the
present and former cause of action? If so, the former recovery is a bar; if otherwise, it does not stand in the way
of the former action. Considering that the aforesaid violations of petitioner’s right as owner in the two cases
both hinge on the binding effect of the REM, i.e., both cases will rise or fall on the issue of the validity of the
REM, it follows that the same evidence will support and establish the first and second causes of action.
VINLUAN,  ALEXANDER  MARA  J.  2C  UST  LAW  
Relucio v. Lopez, 373 SCRA 578
FACTS:

Private respondent filed a petition for APPOINTMENT AS SOLE ADMINISTRATRIX OF CONJUGAL


PARTNERSHIP OF PROPERTIES, FORFEITURE, ETC., against defendant and petitioner Imelda Relucioin a
special proceeding. In the petition, private-respondent alleged that sometime in 1968, defendant, who is legally
married to the private respondent, abandoned the latter and their four legitimate children; that he arrogated unto
himself full and exclusive control and administration of the conjugal properties, spending and using the same
for his sole gain and benefit to the total exclusion of the private respondent and their four children; that
defendant, after abandoning his family, maintained an illicit relationship and cohabited with herein petitioner
since 1976.

Furthermore, it alleged that defendant Lopez and petitioner Relucio, during their period of cohabitation since
1976, have amassed a fortune consisting mainly of stockholdings in Lopez-owned or controlled corporations,
residential, agricultural, commercial lots, houses, apartments and buildings, cars and other motor vehicles, bank
accounts and jewelry. In order to avoid defendant Lopez obligations as a father and husband, he excluded the
private respondent and their four children from sharing or benefiting from the conjugal properties and the
income or fruits therefrom. He placed substantial portions of these conjugal properties in the name of petitioner
Relucio.

Motion to Dismiss the Petition was filed by herein petitioner on the ground that private respondent has no cause
of action against her. However, such was denied on the ground that she is impleaded as a necessary or
indispensable party because some of the subject properties are registered in her name and defendant Lopez, or
solely in her name.

ISSUES:
WON RESPONDENTS PETITION FOR APPOINTMENT AS SOLE ADMINISTRATION OF THE
CONJUGAL PROPERTY, AGAINST HER HUSBAND ESTABLISHED A CAUSE OF ACTION
AGAINST PETITIONER.

WON PETITIONER’S INCLUSION AS A PARTY DEFENDANT IS ESSENTIAL IN THE


PROCEEDINGS FOR A COMPLETE ADJUDICATION OF THE CONTROVERSY.

HELD:

1. NO. The allegations does it appear that relief is sought against petitioner. Respondent’s causes of action were
all against her husband. The first cause of action is for judicial appointment of respondent as administratrix of
the conjugal partnership or absolute community property arising from her marriage to Alberto J.
Lopez. Petitioner is a complete stranger to this cause of action. Article 128 of the Family Code refers only to
spouses. The second cause of action is for an accounting by respondent husband. The accounting of conjugal
partnership arises from or is an incident of marriage. The third cause of action is essentially for forfeiture of
Alberto J. Lopez share in property co-owned by him and petitioner.

2. NO. Petitioner would not be affected by any judgment in Special Proceeding. If petitioner is not a real party
in interest, she cannot be an indispensable party. An indispensable party is one without whom there can be no
final determination of an action. Petitioner’s participation in Special Proceeding is not indispensable.The trial
court can issue a judgment ordering Alberto J. Lopez to make an accounting of his conjugal partnership with
respondent, and give support to respondent and their children, and dissolve Alberto J. Lopez conjugal
partnership with respondent, and forfeit Alberto J. Lopez share in property co-owned by him and
petitioner. Such judgment would be perfectly valid and enforceable only against Alberto J. Lopez.
VINLUAN,  ALEXANDER  MARA  J.  2C  UST  LAW  
JUANA COMPLEX I HOMEOWNERS ASSOCIATION, ET. AL., Petitioner
vs. FIL-ESTATE LAND INC., ET. AL., Respondent
G.R. NO. 152272, MARCH 5, 2012
FACTS:

Juana Complex I together with its individual residents and other neighboring subdivisions instituted a complaint
for damages as a class suit representing the regular commuters and motorists of Juana Complex I (Juana) and
neighboring subdivisions who were deprived of the use of La Paz Road (Road) , against Fil-Estate (Fil-Estate).
The complaint alleged that Juana et.al. were regular commuters and motorists who constantly travelled Road for
more than ten years until Fil-Estate excavated and permanently closed the Road.

RTC - Juana prayed for the immediate issuance of a TRO or a writ of preliminary injunction to enjoin Fil-Estate
from stopping and intimidating them in their use of the Roas. A TRO was issued ordering Fil-Estate for 20 days
to stop preventing or harassing Juana from using the Road. Thus, the trial court onducted several hearings to
determine the propriety of the issuance of WPI. Fil-Estate filed a Motion to Dismiss arguing that complaint
failed to state a cause of action and that it was improperly files as a class suit. RTC granted WPI thus Fil-Estate
filed a Motion for Reconsideration. RTC then issued Omnibus Order denying both Motion to Dismiss and
Motion for Reconsideration file by Fil-Estate.

CA - Fil-Estate filed a petition foe certiorari and prohibit before the CA to annul the Order and Omnibus Order
issued by RTC contending that Juana failed to state a cause of action, improperly filed class suit and failed to
show that Juana had a clear and unmistakable right to the use of the Road since Road was a Torrens registered
private road and there was neither a voluntary nor legal easement constituted over it. CA partially granted
petition on the merit of the last contention.

ISSUES:
WON THE COMPLAINT STATES A CAUSE OF ACTION
WON THE COMPLAINT HAS BEEN PROPERLY FILED AS A CLASS SUIT
WON A WRIT OF PRELIMINARY INJUCTION IS WARRANTED.

HELD:
1. YES. The complaint states a cause of action. Whether the complaint states a cause of action is determined by
its averments regarding the acts committed by the defendant. Thus, contains a concise statement of the ultimate
or essential facts constituting plaintiff’s cause of action. First, Juana’s averments show a demandable right over
Road. Second, there is an alleged violation of such right by Fil-Estate when they excavated and prevented Juana
from using it. Third, Juana consequently suffered injury.
2. YES. The complaint has been properly filed as a class suit. The necessary elements of a class suit are present
in this case namely: common interest, numerous parties affected and sufficient number of parties bringing the
class suit. The suit is clearly one that benefits all commuters and motorists who use the Road.
3. NO. WPI is not warranted. A WPI under Sec. 3, Rule 58 is available to prevent a threatened or continuous
irremediable injury to parties before their claims can be thoroughly studied and adjudicated. The requisites for
its issuance are (1) existence of clear and unmistakable right that must be protected and (2) an urgent and
paramount necessity for the writ to prevent serious damage. Juana failed to establish a prima facie proof of
violation of their right to justify issuance of WPI. Their right to use the Road is disputable since Juana has no
clear legal right therein. Juana merely anchor their purported right over the Road on the bare allegation that they
use the Road for more than ten yeas. A mere allegation does not meet the standard of proof that would warrant
issuance of WPI.

VINLUAN,  ALEXANDER  MARA  J.  2C  UST  LAW  


Misamis Occidental II Cooperative, Inc. v. David, 468 SCRA 63
FACTS:

Private respondent, a supplier of electrical hardware, filed a case for specific performance and damages against
MOELCI II, a rural electric cooperative in Misamis Occidental. The case was essentially a collection suit,
predicated on a document attached as Annex A to the Amended Complaint that according to David is the
contract pursuant to which he sold to MOELCI II one unit of 10 MVA Transformer. MOELCI II filed its
Answer to Amended Complaint, affirmative defenses which constitute grounds for dismissal. These grounds
were lack of cause of action; they’re being allegedly no enforceable contract between the parties under the
Statute of Frauds and improper venue. MOELCI II in essence argued that the document attached as Annex A
was only a quotation letter and not a contract as alleged by David. Thus, it contends that David`s Amended
Complaint is dismissable for failure to state a cause of action. David contended in the main that because a
motion to dismiss on the ground of failure to state a cause of action is required to be based only on the
allegations of the complaint, the “quotation letter”, being merely an attachment to the complaint and not part of
its allegations, cannot be inquired into.

RTC - order denying MOELCI II`s motion for preliminary hearing of affirmative defenses.

CA - dismissed MOELCI II`s petition holding that the allegations in David`s complaint constitutes a cause of
action.

ISSUE:
WON THE CA ERRED IN DISMISSING THE PETITION FOR CERTIORARI AND IN HOLDING
THAT THE TRIAL COURT DID NOT COMMIT GRAVE ABUSE OF DISCRETION IN DENYING
PETITIONER’S MOTION.

HELD: NO

Such interpretation is now specifically expressed in the 1997 Rules of Civil Procedure. Section 6, Rule 16
provides that a grant of preliminary hearing rests on the sound discretion of the court. Moreover, as MOELCI
II`s Motion is anchored on the ground that the Complaint allegedly stated no cause of action, a preliminary
hearing thereon is more than unnecessary as it constitutes an erroneous and improvident move. No error
therefore could be ascribed to the trial court in the denial of such Motion. The test of sufficiency of facts alleged
in the complaint as constituting a cause of action is whether or not admitting the facts alleged, the court could
render a valid verdict in accordance with the prayer of said complaint. It has been hypothetically admitted that
the parties had entered into a contract sale, David bound himself to supply petitioner 1 unit of MVA Power
transformer plus 69 KV Line Accessories; that despite written and verbal demands, petitioner has failed to pay
the price thereof. Finally, we do not agree with MOELCI II`s contention that the Court of Appeals sanctioned
the trial court`s deferment of the resolution of MOELCI II`s Motion. The trial court squarely denied the Motion
and not merely deferred its resolution. Thus, there is no deferment to speak of that should be enjoined.

VINLUAN,  ALEXANDER  MARA  J.  2C  UST  LAW  


PNB vs GATEWAY PROPERTY HOLDINGS, INC.
GR NO. 181485; FEBRUARY 15, 2012
FACTS:

Respondent GPHI was a subsidiary company of Gateway Electronics Company (GEC). GEC obtained long
term loans from the Land Bank of the Philippines (LBP) in the amount of P600 million and the loans were
secured by mortgages executed by GEC over its properties. LBP invited other banks to lend money to GEC. It
is alleged that LBP agreed to submit the mortgaged properties to it by GEC as part of the latters assets that will
be covered by a Mortgage Trust Indenture (MTI), ensuring that all participating banks in the loan syndicate will
have equal security position. LBP and a consortium of banks entered into a Memorandum of Understanding
(MOU), whereby LBP agreed to release the mortgaged properties to the consortium of banks on the basis of an
MTI. The participating banks released funds in favor of GEC. Petitioner PNB became part of this consortium of
creditor banks. GEC requested to convert its long-term loans due to its difficulty in paying its obligations. PNB
approved such subject to certain conditions – GPHI was obligated to execute in favor of PNB a REM over 2
parcels of land. GEC filed a complaint for specific performance against LBP due to the latter’s refusal to share
the mortgaged properties with the consortium of creditor banks. PNB demanded payment from GEC and when
the latter discovered the former’s intent to foreclose the REM, it prayed that a (TRO) be issued to enjoin PNB
from foreclosing on the properties of GPHI and judgment be issued declaring that the real estate mortgage
involving the properties of GPHI and executed in favor of PNB is null and void. No TRO was issued by the
Court, PNB was able to foreclosed the mortgaged properties. Thereafter, GPHI filed a Petition for Annulment
of Foreclosure of Mortgage. GPHI argued that, in conducting the foreclosure proceedings, the sheriff failed to
observe the requirement of Section 4 of Act No. 3135 that the sale shall be made at public auction.
RTC - dismissed the case and ruled that both the civil cases involved the same parties, substantially
identical causes of action and reliefs prayed for, the reliefs being founded on the same facts. Where a
single cause of action has been split and pursuant to Rule 16, Section 1(e) of the 1997 Rules on Civil
Procedure, the Motion to Dismiss filed by PNB, on the ground that there is another action pending
between the same parties for the same cause, or litis pendentia is proper.
CA - set aside the decision of the RTC and ruled that the third requisite of litis pendentia (the identity in the two
cases should be such that the judgment that may be rendered in one would, regardless of which party is
successful, amount to res adjudicata to the other) is not present .

ISSUE:
WON THE REQUISITE OF LITIS PENDENTIA EXIST TO WARRANT THE DISMISSAL OF THE
ANNULMENT OF THE FORECLOSURE SALE.

HELD: YES.

All the requisite of litis pendentia exist. As to the first requisite, GPHI is the plaintiff in both civil cases while
PNB is the party against whom GPHI is asserting a claim. As to the second requisite, allegations in Civil for
Annulment of the Real Estate Mortgage and Annulment of the Foreclosure Sale reveal that the said cases
invoke the same fundamental issue which is the temporary nature of the security that was to be provided by the
mortgaged properties of GPHI. As to the third requisite, While the appeal of the dismissal of Annulment of the
Foreclosure Sale was still pending with the Court of Appeals, GPHI filed on a Motion for Leave to Amend
Complaint to Conform to the Evidence in Civil Case Annulment of the Real Estate Mortgage. GPHI stated
therein that after the parties presented their evidence, the fact of foreclosure and the acquisition of the
mortgaged properties by PNB were duly established. In the accompanying Amended Complaint in Annulment
of the Real Estate Mortgage, GPHI prayed, for the declaration of the nullity of the foreclosure and auction sale
of the mortgaged properties. As a consequence of such an action, the two cases that GPHI filed before the court
a quo henceforth contained an identity of rights asserted and reliefs prayed for, the relief being founded on the
same factual allegations.
VINLUAN,  ALEXANDER  MARA  J.  2C  UST  LAW  
JOVITO R. SALONGA vs. WARNER, BARNES AND CO., LTD G.R. No. L-2246 January 31,
1951

FACTS:

Westchester Fire Insurance Company of New York entered into a contract with Tina J. Gamboa whereby said
company insured one case of rayon yardage which said Tina J. Gamboa shipped from San Francisco, California
to Manila, Philippines and consigned to Jovito Salonga, plaintiff herein. According to the contract of insurance,
the insurance company undertook to pay to the sender or her consignee the damages that may be caused to the
goods shipped subject to the condition that the liability of the company will be limited to the actual loss which
the insured may suffer not to the exceed the sum of P2,000. The ship arrived in Manila, and the shipment was
later on examined by C. B. Nelson and Co., marine surveyors, at the request of the plaintiff, and in their
examination the surveyors found a shortage in the shipment in the amount of P1,723,12.

RTC - Plaintiff filed a claim for damages in the amount of P1,723.12 against the American President Lines,
agents of the ship, demanding settlement, and when no action was taken on this claim, plaintiff demanded
payment thereof from Warner, Barnes and Co., Ltd., as agent of the insurance company in the Philippines, and
this agent having refused to pay the claim, plaintiff instituted the present action. The trial court rendered
judgment against Jovito Salonga. The motion for reconsideration filed by the defendant having been denied, the
case was appealed to this court.

ISSUE:

WON THE RTC ERRED IN HOLDING THAT THE DEFENDANT IS A REAL PARTY-IN-
INTEREST.

HELD: NO

Section 2, Rule 3 of the Rules of Court requires that "every action must be prosecuted in the name of the real
party in interest." In the case at bar, the defendant issued upon in its capacity as agent of Westchester Fire
Insurance Company of New York in spite of the fact that the insurance contract has not been signed by it. As
aforementioned, the defendant did not assume any obligation thereunder either as agent or as a principal. It
cannot, therefore, be made liable under said contract, and hence it can be said that this case was filed against
one who is not the real party in interest. The court further held that the action should have been filed against its
principal, the Westchester Fire Insurance Company of New York.

If the party sued upon is not the proper party, any decision that may be rendered against him would be futile, for
it cannot be enforced or executed. The effort that may be employed will be wasted. Such would be the result of
this case if it will be allowed to proceed against the defendant, for even if a favorable judgment is obtained
against it, it cannot be enforced because the real party is not involved. The defendant cannot be made to pay for
something it is not responsible. The court stated that the correct remedy would be for the Plaintiff to bring the
principal into this case or make it come under the courts in this jurisdiction in accordance with the procedure
indicated in section 14, Rule 7, of the Rules of Court concerning litigations involving foreign corporations. This
rule says that if the defendant is a foreign corporation and it has not designated an agent in the Philippines on
whom service may be made in case of litigation, such service may be made on any agent it may have in the
Philippines. The Westchester Fire Insurance Company of new York comes within the import of this rule for
even if it has not designated an agent as required by law, it has however a settling agent who may serve the
purpose. In other words, an action may be brought against said insurance company in the Philippines and the
process may be served on the defendant to give our courts the necessary jurisdiction.

VINLUAN,  ALEXANDER  MARA  J.  2C  UST  LAW  


HON. CARLOS O. FORTICH, et. al v. HON. RENATO C. CORONA
G.R. No. 131457. April 24, 1998
FACTS:

This case concerns the motion for reconsideration of the court’s resolution dated November 17, 1998 and
motion to refer the case to the Court en banc. In previous case, the Court voted two-two on the separate motions
for reconsideration as a result of which the decision was affirmed. The Court noted in a resolution dated January
27, 1999 that the movants have no legal personality to seek redress before the Court as their motion to intervene
was already denied and that the motion to refer the case to the Court en banc is akin to a second MR, which is
prohibited. In this motion, both respondents and intervenors prayed that the case be referred to the Court en
banc inasmuch as their earlier MR was resolved by a vote of two-two, the required number to carry a decision
under the Constitution was not met.

ISSUE:

WON THE REFERRAL TO THE COURT EN BANC PARTAKES OF THE NATURE OF A SECOND
MOTION FOR RECONSIDERATION.

HELD: YES.

The contention, therefore, that the Resolution of November 17, 1998 did not dispose of the earlier MR of the
Decision dated April 24, 1998 is flawed. Consequently, the present MR necessarily partakes of the nature of a
second motion for reconsideration, which, according to the clear and unambiguous language of Rule 56, Section
4, in relation to Rule 52, Section 2, of the 1997 Rules of Civil Procedure, is prohibited. True, there are
exceptional cases when this Court may entertain a second motion for reconsideration, such as where there are
extraordinarily persuasive reasons. Even then, we have ruled that such second MRs must be filed with express
leave of court first obtained. In this case, not only did movants fail to ask for prior leave of court, but more
importantly, they have been unable to show that there are exceptional reasons for us to give due course to their
second motions for reconsideration. Stripped of the arguments for referral of this incident to the Court en banc,
the motions subject of this resolution are nothing more but rehashes of the motions for reconsideration, which
have been denied in the Resolution of November 17, 1998. To be sure, the allegations contained therein have
already been raised before and passed upon by this Court in the said Resolution.

VINLUAN,  ALEXANDER  MARA  J.  2C  UST  LAW  


Samaniego, et al. v. Aguila, et al., GR No. 125567, June 27, 2000.

FACTS:

Petitioners are tenants in a landholding owned by respondents’ mother with an aggregate area of 10.4496
hectares in Patul, Santiago, Isabela. The subject land was identified by the DAR-Region 2 as covered by the
Operation Land Transfer Program of the government. After sometime, respondent’s mother on their behalf filed
a petition for exemption from the coverage of PD 27 (Land Reform Program). Thus, petitioners opposed the
application for respondents’ mother transferred the title of the lands to respondents in violation of the rules and
regulations of the DAR.

DAR - granted the application for exemption. The decision was affirmed on appeal to the DAR, which was
reversed by the same on motion of the petitioners. Thus, DAR denied the application for exemption and
declared petitioners as rightful farmer-beneficiaries of the land.

Office of the President - Respondents appealed but such set aside the decision of the DAR and reinstated DAR’s
prior decision.

Court of Appeals - the petition was dismissed for failure to implead the Office of the President (OP) as an
indispensable party (as the one whose decision and resolution is being questioned). Joinder of indispensable
parties is mandatory. Failure to implead the OP was fatal and the petition must be dismissed.

ISSUE:
WON THE OP WAS AN INDIPENSABLE PARTY AND HAD TO BE IMPLEADED.

HELD: NO

At the time the petitioners brought the case to the CA, the rule on appeals to the said court from quasi-judicial
agencies was that, “petition for review shall (a) state the full names of the parties, without impleading the
court or agencies either as petitioners or respondents.” Also, the CA was not an indispensable party or a
party in interest without whom no final determination can be had of an action without being impleaded.
Indispensable parties are those with such an interest in the matter that a final decree would necessarily affect
their rights and the court cannot proceed without their presence. The word “interest” in this rule should be
material, directly in issue and to be affected by the decree, as distinguished from a mere incidental interest in
the question involved, as opposed to a nominal or pro forma party who is joined as a plaintiff or defendant,
not because such party has any real interest in the subject matter or because any relief is demanded, but merely
because the technical rules of pleadings require the presence of such party on the record. Thus, the OP not
having any interest in the case except to entertain appeals from the DAR, is not an indispensable party.

VINLUAN,  ALEXANDER  MARA  J.  2C  UST  LAW  


Theodore and Nancy Ang, etc. v. Spouses Alan and Em Ang, G.R. No. 186993, August 22, 2012

FACTS:

The respondents, obtained a loan worth $300,000 from Theodore and Nancy Ang, the petitioners. When the
loan became demandable, respondent spouses executed a Promissory Note promising to pay the loan and 10%
annual interest on demand. Petitioners made several demands but respondents failed to pay, their obligation
amounting to $719, 627.21 inclusive of the 10% interest. The petitioners were residing in Los Angeles,
California. They executed a Special Power of Attorney in favor of one Atty. Eldrige Marvin Aceron in order for
him to file an action against the respondent spouses.

RTC - filed a complaint for the collection of a sum of money with the RTC of Quezon City, his residence. The
respondent spouses moved to dismiss, on the ground of improper venue. They contend that the complaint
against them may be filed in the where either the petitioners or respondents reside. Respondent spouses reside in
Bacolod City. The RTC denied the motion to dismiss, ruling that Atty. Aceron as the attorney-in-fact may use
his residence as basis for the venue of the action.

CA - filed a petition for certiorari with the CA based on their earlier ground and on the ground that Atty.
Aceron, a mere attorney-in-fact, is not the real party in interest; that his residence should not be considered in
determining the proper venue for the complaint. The CA reversed the RTC and dismissed the complaint filed by
the petitioners. The CA held that the complaint should have been filed in Bacolod City instead. Petitioners
moved to reconsider but were denied by the CA.

ISSUE:

WON THE CA ERRER WHEN IT RULED THAT THE COMPLAINT MUST BE DISMISSED ON
THE GROUND OF IMPROPER VENUE.

HELD: NO.

While the fixing of the venue in personal actions may be done for the convenience of the plaintiffs and their
witnesses, the choice is not left to the caprice of the plaintiff. The Rules of court still regulate the fixing of the
venue for filing a personal action.

The collection of a sum of money is a personal action in order to enforce a contract. The Rules of Court give the
plaintiff the choice where to file the action; whether in the place where the plaintiff resides or in the place where
the defendant resides. However, the plaintiffs in this case do not reside in the Philippines. Philippine Courts do
not have jurisdiction over persons residing abroad. Hence, the only choice left for venue was the residence of
the defendant.

VINLUAN,  ALEXANDER  MARA  J.  2C  UST  LAW  


SIMNY G. GUY, GERALDINE G. GUY, GLADYS G. YAO, and the HEIRS OF THE LATE GRACE G.
CHEU,Petitioners, vs. GILBERT G. GUY, Respondent.

FACTS:

Gilbert G. Guy son of spouses Simny and Francisco Guy, claims to own 80% of their multi-million family
corporation GoodGold Realty Development Inc, stating that he owns 519, 997 shares (fully paid upon
incorporation) out of the 650,000 subscribed capital stock. His mother Simny however contends that it was she
and her husband who established the corporation and only placed the bulk of the shares in Gilbert’s name
because being their son, they had entrusted to him the future of their corporations. She further claims that
during the incorporation of GoodGold, they were advised by their lawyers to issue the stock certificates with
corresponding blank endorsements signed by Francisco as President and Atty. Paras as Corporate Secretary.;
including Stock Certificate Nos. 004-014 under Gilbert’s name. In 1999, Francisco gave instructions to
redistribute the shares of the corporation evenly among his children while maintaining a proportionate share for
himself and Simny. Hence, GoodGold’s certificates were cancelled and new ones were issued showing that the
4 siblings had 65,000 shares each while the spouses had 195,000 shares each. Five years after the redistribution,
Gilbert brought an action against his mother Simny and his sisters for the annulment of the said transfers of
shares along with some corporate documents, alleging fraud and that his signatures at the back of the stock
certificates which purportedly endorsed the same were forged and must be nullified. NBI reports on the
examination of signatures however showed them to be authentic. Gilbert withdrew his complaint. Three years
thereafter, a new action was filed by Gilbert with the caption “Intra-corporate Controversy: For the Declaration
of Nullity of Fraudulent Transfers of Shares of Stock Certificates, Fabricated Shares of Stocks, Falsified
General Information Sheets, Minutesof Meetings, etc…” against his mother and sisters. Gilbert claims that he is
“unaware of any document signed by him that would justify and support the transfer of his shares to herein
petitioners.” Simny and daughters filed their manifestation that the action filed by Gilbert was a mere nuisance
and harassment suit under Sec 1(b), Rule 1 of the Interim Rules of Procedure on Intra-Corporate Controversies.
RTC dismissed the case as a nuisance and harassment suit, CA reversed RTC.

ISSUE:

WON THE ACTION SHOULD BE DECLARED NULL AND VOID FOR WANT OF AUTHORITY.

HELD:

The absence of an indispensable party in a case renders all subsequent actions of the court null and void for
want of authority to act, not only as to the absent parties but even as to those present. It bears emphasis that this
controversy started with Gilbert’s complaint filed with the RTC of Mandaluyong City in his capacity as
stockholder, director and Vice-President of GoodGold. Gilbert’s complaint essentially prayed for the return of
his original 519,997 shares in GoodGold, by praying that the court declare that "there were no valid transfers of
the contested shares to defendants and Francisco.” The transfer of the shares cannot be, as Gilbert wanted,
declared entirely fraudulent without including those of Francisco who owns almost a third of the total number.
Francisco, in both the 2004 and 2008 complaints, is an indispensable party without whom no final
determination can be had for the following reasons: (a) the complaint prays that the shares now under the name
of the defendants and Francisco be declared fraudulent; (b) Francisco owns 195,000 shares some of which,
Gilbert prays be returned to him; (c) Francisco signed the certificates of stocks evidencing the alleged
fraudulent shares previously in the name of Gilbert. The inclusion of the shares of Francisco in the complaint
makes Francisco an indispensable party. Moreover, the pronouncement about the shares of Francisco would
impact on the hereditary rights of the contesting parties or on the conjugal properties of the spouses to the effect
that Francisco, being husband of Simny and father of the other contesting parties, must be included for,
otherwise, in his absence, there cannot be a determination between the parties already before the court which is
effective, complete, or equitable.
VINLUAN,  ALEXANDER  MARA  J.  2C  UST  LAW  
LIVING @ SENSE, INC. vs. MALAYAN INSURANCE COMPANY, INC.
G.R. No. 193753 September 26, 2012
FACTS:

Petitioner was the main contractor of the FOC Network Project of Globe Telecom in Mindanao. Petitioner
entered into a Sub-Contract Agreement (Agreement) with Dou Mac, Inc. (DMI), under which the latter was
tasked to undertake an underground open-trench work. Petitioner required DMI to give a bond, in the event that
DMI fails to perform its obligations under the Agreement. Thus, DMI secured surety and performance bonds.
The DPWH issued a work-stoppage order against DMI after finding the latter’s work unsatisfactory. DMI failed
to adopt corrective measures, prompting petitioner to terminate the Agreement and seek indemnification from
respondent in the total amount of P 1,040,895.34. However, respondent effectively denied petitioner’s claim on
the ground that the liability of its principal, DMI, should first be ascertained before its own liability as a surety
attaches. Hence, the instant complaint, premised on respondent’s liability under the surety and performance
bonds secured by DMI.

Respondent claimed that DMI is an indispensable party that should be impleaded and whose liability should
first be determined before respondent can be held liable. On the other hand, petitioner asserted that respondent
is a surety who is directly and primarily liable to indemnify petitioner, and that the bond is "callable on
demand" in the event DMI fails to perform its obligations under the Agreement.

ISSUE:
WON DMI IS AN INDISPENSABLE PARTY.

HELD: NO.

DMI is not an indispensable party because petitioner can claim indemnity directly from respondent, having
made itself jointly and severally liable with DMI for the obligation under the bonds. Therefore, the failure to
implead DMI is not a ground to dismiss the case, even if the same was without prejudice. The term "jointly and
severally" expresses a solidary obligation-granting petitioner, as creditor, the right to proceed against its
debtors. The nature of the solidary obligation under the surety does not make one an indispensable party. The
presence of indispensable parties is necessary to vest the court with jurisdiction, thus, without their presence to
a suit or proceeding, the judgment of a court cannot attain real finality. The absence of an indispensable party
renders all subsequent actions of the court null and void for want of authority to act, not only as to the absent
parties but even as to those present. When DMI secured the surety and performance bonds from respondent in
compliance with petitioner’s requirement, respondent bound itself "jointly and severally" with DMI for the
damages and actual loss that petitioner may suffer should DMI fail to perform its obligations.

VINLUAN,  ALEXANDER  MARA  J.  2C  UST  LAW  


PHILIP L. GO, PACIFICO Q. LIM and ANDREW Q. LIM Petitioners,
vs.
DISTINCTION PROPERTIES DEVELOPMENT AND CONSTRUCTION, INC. Respondent.
G.R. No. 194024 April 25, 2012
FACTS:

Petitioners are registered individual owners of condominium units in Phoenix Heights Condominium Pasig
City, Metro Manila. Respondent Distinction Properties Development and Construction, Inc. (DPDCI) is the real
estate developer of Phoenix Heights Condominium, with principal office at Binondo, Manila. One of the
petitioners executed a Master Deed and Declaration of Restrictions (MDDR) of Phoenix Heights Condominium,
which was filed with the Registry of Deeds. As the developer, DPDCI undertook, among others, the marketing
aspect of the project, the sale of the units and the release of flyers and brochures. Thereafter, Phoenix Heights
Condominium Corporation (PHCC) was formally organized and incorporated. Sometime in 2000, DPDCI
turned over to PHCC the ownership and possession of the condominium units, except for the two saleable
commercial units/spaces. Petitioner Lim filed an Application for Alteration of Plan pertaining to the
construction of 22 storage units in the spaces adjunct to the parking area of the building, but was disapproved as
such would obstruct light and ventilation.

HLURB - filed a complaint against DPDCI for unsound business practices and violation of the MDDR, alleging
that the latter committed misrepresentation in their circulated flyers and brochures as to the facilities or
amenities that would be available in the condominium and failed to perform its obligation to comply with the
MDDR. However, it ruled in favor of the petitioners, holding as invalid the agreement entered into between
DPDCI and PHCC, as to the alteration or conversion of the subject units into common areas, which it
previously approved, for the reason that it was not approved by the majority of the members of PHCC as
required under Section 13 of the MDDR.

CA - Petition for Certiorari and Prohibition on the ground that the HLURB decision was a patent nullity
constituting an act without or beyond its jurisdiction and that it had no other plain, speedy and adequate remedy
in the course of law. Favored DPDCI, setting aside HLURB’s decision and holds that HLURB had no
jurisdiction over the complaint filed by petitioners as the controversy did not fall within the scope of the
administrative agency’s authority under P.D. No. 957. The HLURB not only relied heavily on the brochures
which, according to the CA, did not set out an enforceable obligation on the part of DPDCI, but also
erroneously cited Section 13 of the MDDR to support its finding of contractual violation.

ISSUES:
WON THE HLURB HAS JURISDICTION OVER THE COMPLAINT FILED BY THE
PETITIONERS.
WON PHCC IS AN INDISPENSABLE PARTY.
WON THE RULE ON EXHAUSTION OF ADMINISTRATIVE REMEDIES APPLIES IN THIS CASE.

HELD:
1. NO. Jurisdiction over the subject matter of a case is conferred by law and determined by the allegations in the
complaint which comprise a concise statement of the ultimate facts constituting the plaintiff's cause of action.
The nature of an action, as well as which court or body has jurisdiction over it, is determined based on the
allegations contained in the complaint of the plaintiff, irrespective of whether or not the plaintiff is entitled to
recover upon all or some of the claims asserted therein. The averments in the complaint and the character of the
relief sought are the ones to be consulted. Once vested by the allegations in the complaint, jurisdiction also
remains vested irrespective of whether or not the plaintiff is entitled to recover upon all or some of the claims
asserted therein. Thus, it was ruled that the jurisdiction of the HLURB to hear and decide cases is determined by
VINLUAN,  ALEXANDER  MARA  J.  2C  UST  LAW  
the nature of the cause of action, the subject matter or property involved and the parties. To determine if
HLURB has jurisdiction over petitioners’ cause of action, an examination of the laws defining the HLURB’s
jurisdiction and authority becomes imperative. P.D. No. 957, specifically Section 3, granted the National
Housing Authority (NHA) the "exclusive jurisdiction to regulate the real estate trade and business." Then came
P.D. No. 134421 expanding the jurisdiction of the NHA.

2. YES. The acts being assailed are those of PHHC; this case cannot prosper for failure to implead such proper
party. An indispensable party is defined as one who has such an interest in the controversy or subject matter that
a final adjudication cannot be made, in his absence, without injuring or affecting that interest. Under Section 7,
Rule 3 of the Rules of Court, "parties in interest without whom no final determination can be had of an action
shall be joined as plaintiffs or defendants." If there is a failure to implead an indispensable party, any judgment
rendered would have no effectiveness. It is "precisely ‘when an indispensable party is not before the court (that)
an action should be dismissed.’ The absence of an indispensable party renders all subsequent actions of the
court null and void for want of authority to act, not only as to the absent parties but even to those present." The
purpose of the rules on joinder of indispensable parties is a complete determination of all issues not only
between the parties themselves, but also as regards other persons who may be affected by the judgment. A
decision valid on its face cannot attain real finality where there is want of indispensable parties. From all
indications, PHCC is an indispensable party and should have been impleaded, either as a plaintiff or as a
defendant, in the complaint filed before the HLURB as it would be directly and adversely affected by any
determination therein.

3. NO. The doctrine of exhaustion of administrative remedies is a cornerstone of our judicial system. The thrust
of the rule is that courts must allow administrative agencies to carry out their functions and discharge their
responsibilities within the specialized areas of their respective competence. The challenged decision of the
HLURB is patently illegal having been rendered in excess of jurisdiction, if not with grave abuse of discretion
amounting to lack or excess of jurisdiction. Also, the issue on jurisdiction is purely legal which will have to be
decided ultimately by a regular court of law. There is a question of law when the doubt or difference arises as to
what the law is on a certain state of facts, and not as to the truth or the falsehood of alleged facts. Said question
at best could be resolved only tentatively by the administrative authorities. The final decision on the matter rests
not with them but with the courts of justice. Exhaustion of administrative remedies does not apply, because
nothing of an administrative nature is to be or can be done. The issue does not require technical knowledge and
experience but one that would involve the interpretation and application of law.

VINLUAN,  ALEXANDER  MARA  J.  2C  UST  LAW  


PEDRO SEPULVEDA, SR., substituted by SOCORRO S. LAWAS, Administratrix of His Estate,
petitioner, vs. ATTY. PACIFICO S. PELAEZ, respondent.

FACTS:

The case at bar is a petition for review on certiorari under Rule 45 of the Rules of Court. The private respondent
(Atty. Pelaez) filed a complaint against his grand uncle (Sepulveda, Sr.) with the CFI Cebu for the recovery of
possession and ownership of his ½ undivided share of several parcels of land; his undivided 1/3 share in several
other lots and for the partition among the co-owners. The 11 lots among the 25 parcels of lands which Atty.
Pelaez’s mother, Dulce Sepulveda, inherited from her grandmother under the project of partition which was
submitted by Sepulveda, Sr. as the administrator of the formers estate, duly approved by the said CFI in a
Special Proceeding. Under the said deed, Sepulveda, Sr. appeared to be the owner of an undivided portion of
Lot No. 28199, while his brother and Dulces uncle Santiago Sepulveda, was the undivided owner of ½ of the
parcels of land covered by T.D. Nos. 18197, 18193 and 28316. Dulce and her uncles, Pedro and Santiago, were
likewise indicated therein as the co-owners of the 11 other parcels of land, each with an undivided 1/3 share
thereof.
RTC - ruled that the private respondents action for reconveyance based on constructive trust had not yet
prescribed when the complaint was filed; that he was entitled to a share in the proceeds of the sale of the
property to Danao City; and that the partition of the subject property among the adjudicatees thereof was in
order.
CA - It affirmed the decision of the RTC.

ISSUE:
WON THE REGIONAL TRIAL COURT IN DANAO CITY THAT PAYMENT WAS MADE BY
DANAO CITY FOR ONE (1) OF THE ELEVEN (11) PARCELS INVOLVED IN THE CASE AND OF
WHICH HEREIN RESPONDENT SHOULD BE PAID BY PETITIONER ONE THIRD (1/3) OF THE
PURCHASE PRICE.

HELD: YES.
As gleaned from the material averments of the complaint and the reliefs prayed for therein, the private
respondent, as plaintiff therein, sought the recovery of the ownership and possession of the ten (10) parcels of
land and the partition thereof; and for the payment of his share in the proceeds of the sale of the property which
Pedro Sepulveda, Sr. sold to Danao City amounting to P7,492.00, which Pedro Sepulveda, Sr. claimed was left
unpaid. It appears that when the private respondent filed the complaint, his father, Rodolfo Pelaez, was still
alive. Thus, when his mother Dulce Pelaez died intestate on March 2, 1944, her husband Rodolfo and their son,
the private respondent, survived her. Under Article 996 of the New Civil Code, Rodolfo Pelaez, as surviving
spouse, is entitled to a portion in usufruct equal to that corresponding by way of legitime to each of the
legitimate children who has not received any betterment. The rights of the usufructuary are provided in Articles
471 to 490 of the old Civil Code. Section 1, Rule 69 of the Rules of Court provides that in an action for
partition, all persons interested in the property shall be joined as defendants. Thus, all the co-heirs and persons
having an interest in the property are indispensable parties; as such, an action for partition will not lie without
the joinder of the said parties. The mere fact that Pedro Sepulveda, Sr. has repudiated the co-ownership between
him and the respondent does not deprive the trial court of jurisdiction to take cognizance of the action for
partition, for, in a complaint for partition, the plaintiff seeks, first, a declaration that he is a co-owner of the
subject property; and, second, the conveyance of his lawful shares. In the present action, the private respondent,
as the plaintiff in the trial court, failed to implead the following indispensable parties: his father, Rodolfo
Pelaez; the heirs of Santiago Sepulveda, namely, Paz Sepulveda and their children; and the City of Danao which
purchased the property from Pedro Sepulveda, Sr. and maintained that it had failed to pay for the purchase price
of the property. Rodolfo Pelaez is an indispensable party he being entitled to a share in usufruct, equal to the
share of the respondent in the subject properties. There is no showing that Rodolfo Pelaez had waived his right
to usufruct.
VINLUAN,  ALEXANDER  MARA  J.  2C  UST  LAW  
VICTORIANO BORLASA, ET AL. v. VICENTE POLISTICO, ET AL., G.R. No. L-22909 January 28,
1925

FACTS:

An action was instituted by petitioner against respondent in the Court of First Instance for the purpose of
securing the dissolution of a voluntary association named Turuhan Polistico & Co., and to compel the
defendants to account for and surrender the money and property of the association in order that its affairs may
be liquidated and its assets applied according to law. In an amended answer the defendants raised the question
of lack of parties and set out a list of some hundreds of persons whom they alleged should be brought in as
parties defendant on the ground, among others, that they were in default in the payment of their dues to the
association. The court made an order requiring the plaintiffs to amend their complaint within a stated period so
as to include all of the members of the Turnuhan Polistico & Co. either as plaintiffs or defendants. The trial
judge having sustained a demurrer for defect of parties and the plaintiffs electing not to amend, the cause was
dismissed, and from this order the plaintiffs took an appeal to this court.

ISSUE:

WON ALL THE MEMBERS OF THE ASSOCIATION MUST BE IMPLEADED EITHER AS


PLAINTIFFS OR DEFENDANTS.

HELD: NO

To require all members to appear would be quite impossible. Hence, some members must be made to sue but
only in behalf of all the members who are not around and it is impracticable to bring them all to the court. A
number of them may sue for the benefit of all.

The general rule with reference to the making of parties in a civil action requires, of course, the joinder of all
necessary parties wherever possible, and the joinder of all indispensable parties under any and all conditions,
the presence of those latter being a sine qua non of the exercise of judicial power. The class suit contemplates
an exceptional situation where there are numerous persons all in the same plight and all together constituting a
constituency whose presence in the litigation is absolutely indispensable to the administration of justice. Here
the strict application of the rule as to indispensable parties would require that each and every individual in the
class should be present. But at this point the practice is so far relaxed as to permit the suit to proceed, when the
class is sufficient represented to enable the court to deal properly and justly with that interest and with all other
interest involved in the suit. In the class suit, then, representation of a class interest, which will be affected by
the judgment, is indispensable; but it is not indispensable to make each member of the class an actual party.

VINLUAN,  ALEXANDER  MARA  J.  2C  UST  LAW  


Newsweek Inc., v. IAC, 142 SCRA 171

FACTS:

Private respondents, incorporated sugarcane planters in Negros Occidental claiming to have 8,500 members and
several individual sugar planters, filed Civil Case No. 15812 in their own behalf and/or as a class suit in behalf
of all sugarcane planters in the province of Negros Occidental, against petitioner and two of petitioners' non-
resident Newsweek correspondents Fred Bruning and Barry Came. The complaint alleged that petitioner and the
other defendants committed libel against them by the publication of the article "An Island of Fear" in the
February 23, 1981 issue of petitioner's weekly news magazine Newsweek. The article supposedly portrayed the
island province of Negros Occidental as a place dominated by big landowners or sugarcane planters who not
only exploited the impoverished and underpaid sugarcane workers/laborers, but also brutalized and killed them
with impunity. Complainants therein alleged that said article, taken as a whole, showed a deliberate and
malicious use of falsehood, slanted presentation and/or misrepresentation of facts intended to put them
(sugarcane planters) in bad light, expose them to public ridicule, discredit and humiliation here in the
Philippines and abroad, and make them objects of hatred, contempt and hostility of their agricultural workers
and of the public in general.

RTC - denied the motion to dismiss. Complaint on its face states a valid cause of action; and the question as
to whether the printed article sued upon its actionable or not is a matter of evidence. Petitioner argued that the
complaint failed to state a cause of action.

ISSUE:
WON A CLASS SUIT IS PROPER
WON THE CASE SHOULD BE DISMISSED FOR LACK OF CAUSE OF ACTION.

HELD:

1. NO. The class suit is not proper. In the case of Corpuz and Cuaderno, the court has ruled that in order to
maintain a libel suit, it is essential that the victim must be identifiable. For a defamation to be directed at a
particular class, it is essential that the allegation must be so sweeping and all embracing that an individual can
prove that a defamatory statement is directed to him. The disputed portion not the articles which he claims to be
libelous was never pointed out.

2. YES. It is evident from the xxx ruling that where the defamation is alleged to have been directed at a group
or class, it is essential that the statement must be so sweeping or all-embracing as to apply to every individual in
that group or class, or sufficiently specific so that each individual in the class or group can prove that the
defamatory statement specifically pointed to him, so that he can bring the action separately, if need be. The case
at bar is not a class suit. It is not a case where one or more may sue for the benefit of all (Mathay vs.
Consolidated Bank and Trust Company, 58 SCRA 559) or where the representation of class interest affected by
the judgment or decree is indispensable to make each member of the class an actual party (Borlaza vs. Polistico,
47 Phil. 348). We have here a case where each of the plaintiffs has a separate and distinct reputation in the
community. They do not have a common or general interest in the subject matter of the controversy.

VINLUAN,  ALEXANDER  MARA  J.  2C  UST  LAW  


Oposa vs Factoran
GR No. 101083; July 30 1993
FACTS:

A Civil Case was filed before Regional Trial Court. The principal plaintiffs therein, now the principal
petitioners, are all minors duly represented and joined by their respective parents. The original defendant was
the Honorable Fulgencio S. Factoran, Jr., then Environment and Natural Resources Secretary. His substitution
in this petition by the new Secretary, the Honorable Angel C. Alcala, was subsequently ordered upon proper
motion. The complaint was instituted as a taxpayers' class suit and alleges that the plaintiffs "are all citizens of
the Republic of the Philippines, taxpayers, and entitled to the full benefit, use and enjoyment of the natural
resource treasure that is the country's virgin tropical rainforests." The same was filed for themselves and others
who are equally concerned about the preservation of said resource but are "so numerous that it is impracticable
to bring them all before the Court." The minors further asseverate that they "represent their generation as well as
generations yet unborn." Consequently, it is prayed for that judgment be rendered, ordering defendant to cancel
all existing timber license agreements in the country. Factoran filed a Motion to Dismiss the complaint stating
the plaintiffs have no cause of action against him and the issue is a political question which properly pertains to
the legislative or executive branches. Subsequently, respondent Judge issued an order granting the motion to
dismiss. The respondent Judge ruled that the granting of the reliefs prayed for would impair contracts. Plaintiffs
thus filed the instant special civil action for certiorari under Rule 65 of the Rules of Court asking for rescission
and setting aside the dismissal order since the respondent Judge gravely abused his discretion in dismissing the
action.

ISSUE:

WON A CLASS SUIT WAS THE PROPER ACTION.

HELD: YES

The case however has a special and novel element. The personality of the minors to sue for the succeeding
generations is based on the concept of inter-generational responsibility insofar as a balanced and healthful
ecology is concerned. Every generation has a responsibility to preserve the ecology. The minors’ right to a
sound environment constitutes at the same time the performance of the obligation to ensure the protection of the
rights or the generations to come. The subject matter of the complaint is of common and general interest not
just to several, but to all citizens of the Philippines. Consequently, since the parties are so numerous, it becomes
impracticable, if not totally impossible, to bring all of them before the court. We likewise declare that the
plaintiffs therein are numerous and representative enough to ensure the full protection of all concerned interests.
Hence, all the requisites for the filing of a valid class suit under Section 12, Rule 3 of the Revised Rules of
Court are present both in the said civil case and in the instant petition, the latter being but an incident to the
former.

VINLUAN,  ALEXANDER  MARA  J.  2C  UST  LAW  


Ortigas & Co., Limited Partnership v. Ruiz, 148 SCRA 326

FACTS:

Petitioner is the duly registered owner of several adjacent parcels of land. Pedro del Rosario filed a class suit on
behalf of 104 other residents seeking the titles of petitioner to be held null and void. Inocencio Bernardo et al
also filed a classs suit against the same petitioner's construction of fences and high walls, roads, streets and
canals on the land in dispute.

ISSUE:

WON THERE IS A VALID CLASS SUIT.

HELD: NO

A class suit is not proper in this case as such presupposes a common and general interest by several plaintiffs in
a single specific thing under Section 12, Rule 3 of the Rules of Court. Consequently, it cannot be maintained
when each of those impleaded as alleged plaintiffs "has only a special or particular interest in the specific thing
completely different from another thing in which the defendants have a like interest." It is not a case where one
or more may sue for the benefit of all or where the representation of class interest affected by the judgment or
decree is indispensable to make each member of the class an actual party.

In the case at bar, a class suit would not lie because each of the defendants has an interest only in the particular
portion of the land he is actually occupying, and not in the portions individually occupied by the other
defendants. They do not have a common or general interest in the subject matter of the controversy

VINLUAN,  ALEXANDER  MARA  J.  2C  UST  LAW  


RIVIERA FILIPINA, INC., v. COURT OF APPEALS, JUAN L. REYES, PHILIPPINE CYPRESS
CONSTRUCTION & DEVELOPMENT CORPORATION, CORNHILL TRADING CORPORATION
and URBAN DEVELOPMENT BANK, G.R. No. 117355, April 5, 2002

FACTS:

Respondent Reyes executed a ten year renewable Contract of Lease with Riviera involving a 1,018 square meter
parcel of land which was a subject of a Real Estate Mortgage executed by Reyes in favor of Prudential Bank.
But the loan with Prudential Bank remained unpaid upon maturity so the bank foreclosed the mortgage thereon
and emerged as the highest bidder at the public auction sale. Reyes decided to sell the property offered it to
Reviera. After seven months, Riviera offered to buy the property but Reyes denied it and increased the price of
the property. Reyes’ counsel informed Riviera that he is selling the property for P6,000 per square meter and to
confirm their conversation, Riviera sent a letter stating his interest in buying the property for the fixed and final
price of P5,000 per square meters but Reyes did not accede to said price.

Then Reyes confided to Traballo and the latter expressed interest in buying the said property for P5,300 per
square meter but he did not have enough amount so he looked for a partner. Despite of the impending expiration
of the redemption period of the foreclosed mortgaged property and the deal between Reyes and Traballo was
not yet formally concluded, Reyes decided to approach Riviera and requested Atty. Alinea to approach Angeles
and find out if the latter was still interested in buying the subject property and ask him to raise his offer for the
purchase of the said property a little higher but Riviera said that his offer is P5,000 per square meter so Reyes
did not agree.

Cypress and Trading Corporation, were able to come up with the amount sufficient to cover the redemption
money, with which Reyes paid to the Prudential Bank to redeem the subject property and Reyes executed a
Deed of Absolute Sale covering the subject property. Cypress and Cornhill mortgaged the subject property to
Urban Development Bank. Riviera sought from Reyes, Cypress and Cornhill a resale of the subject property to
it claiming that its right of first refusal under the lease contract was violated but his attempts were unsuccessful.
Riviera filed the suit to compel Reyes, Cypress, Cornhill and Urban Development Bank to transfer the
disputed title to the land in favor of Riviera upon its payment of the price paid by Cypress and Cornhill.

Both the trial court and the appellate court rendered judgment in favor of Reyes.

ISSUE:
WON PETITIONER HAS LOST ITS RIGHT OF FIRST REFUSAL.

HELD: NO. The distinctions between Rule 45 and 65 are far and wide, the most notable of which is that errors
of jurisdiction are best reviewed in a special civil action for certiorari under Rule 65, while errors of judgment
are correctible only by appeal in a petition for review under Rule 45. The rationale for the distinction is simple.
When a court exercises its jurisdiction an error committed while so engaged does not deprive it of the
jurisdiction being exercised when the error is committed. If it did, every error committed by a court would
deprive it of its jurisdiction and every erroneous judgment would be a void judgment. This cannot be allowed.
The administration of justice would not countenance such a rule. Thus, an error of judgment that the court may
commit in the exercise of its jurisdiction is not correctible through the original special civil action of certiorari.
Appeal from a final disposition of the Court of Appeals, as in the case at bar, is by way of a petition for review
under Rule 45. In the petition at bar, Riviera posits the view that its right of first refusal was totally disregarded
or violated by Reyes by the latter’s sale of the subject property to Cypress and Cornhill. It contends that the
right of first refusal principally amounts to a right to match in the sense that it needs another offer for the right
to be exercised.
VINLUAN,  ALEXANDER  MARA  J.  2C  UST  LAW  
SOCORRO SEPULVEDA LAWAS, vs. COURT OF APPEALS, HON. BERNARDO LL. SALAS, [as
Judge, CFI, Cebu, Branch VIII], and PACIFICO PELAEZ, G.R. No. L-45809, December 12, 1986

FACTS:

Private respondent Pacifico Pelaez filed a Complaint against petitioner's father, Pedro Sepulveda, for ownership
and partition of certain parcels of land. Defendant Pedro Sepulveda filed his Answer resisting the claim and
raising the special defenses of laches, prescription and failure to ventilate in a previous special proceeding.
During the presentation of evidence for the plaintiff, the defendant died. Counsels for the deceased defendant
filed a notice of death wherein were enumerated the thirteen children and surviving spouse of the deceased.

Petitioner filed a petition for letters of administration and she was appointed judicial administratrix of the estate
of her late father. At the hearing of the case, Attys. Domingo Antigua and Serafin Branzuela, former counsels
for the deceased defendant, manifested in open court that with the death of their client, their contract with him
was also terminated and none of the thirteen children nor the surviving spouse had renewed the contract, but
instead they had engaged the services of other lawyers in the intestate proceedings. Notwithstanding the
manifestation of the former counsels of the deceased defendant, the respondent trial judge set the case for
hearing and sent the notice of hearing to said counsels.

RTC - issued three orders. The first order substituted the heirs of the deceased defendant, namely, his thirteen
children and surviving spouse, as defendants; the second order authorized Atty. Teodoro Almase, counsel for
the plaintiff, to present his evidence in the absence of Attys. Antigua and Branzuela and the third order treated
the case submitted for decision, after the plaintiff had presented his evidence and rested his case, and directed
that said counsels and the fourteen heirs of the deceased defendant be furnished copies thereof. Then after, it
rendered a decision against the heirs of the deceased defendant. The ten of the children of the deceased
defendant, who apparently did not know that a decision had already been rendered, filed an Answer in-
substitution of the deceased defendant through their counsel Atty. Jesus Yray. This was denied admission by the
respondent trial judge for being already moot and academic because of the earlier decision.
CA - filed a special civil action of certiorari with the Court of Appeals to annul the proceedings in the
respondent trial court. However, the Court of Appeals dismissed the petition for certiorari. Hence, the present
appeal.

ISSUE:
WON THE CFI AND CA ERRED IN ITS DECISION FOR FAILURE TO COMPY WITH SEC. 16,
RUL3 3.

HELD: YES
SEC 16, RULE 1 - Duty of attorney upon death, incapacity, or incompetency of party. — Whenever a party to a
pending case dies, becomes incapacitated or incompetent, it shall be the duty of his attorney to inform the court
promptly of such death, incapacity or incompetency, and to give the name and residence of his executor,
administrator, guardian or other legal representative. The former counsels for the deceased defendant, Pedro
Sepulveda, complied with this rule by filing a notice of death on May 21, 1975. They also correctly manifested
in open court at the hearing of the case on November 27, 1975, that with the death of their client their contract
with him was also terminated and none of the heirs of the deceased had renewed the contract, and the heirs had
instead engaged the services of other lawyers in the intestate proceedings. Both the respondent trial judge and
the CA erred in considering the former counsels of the deceased defendant as counsels for the heirs of the
deceased. The statement in the decision of the CA that "the appearance of the lawyers of their deceased father in
court on January 13, 1976 carries the presumption that they were authorized by the heirs of the deceased
defendant" is erroneous. Moreover, such a presumption was not warranted in view of the manifestation of said
lawyers in open court on November 27, 1975 that they were not representing the heirs of the deceased
defendant.
VINLUAN,  ALEXANDER  MARA  J.  2C  UST  LAW  
Armand Nocum and the Philippine Daily Inquirer v. Lucio Tan, G.R. No. 145022, September 23, 2005

FACTS:

Lucio Tan filed a complaint against the petitioners with the RTC of Makati seeking moral and exemplary
damages for the alleged malicious and defamatory imputations contained in a news article. INQUIRER and
NOCUM filed their joint answer, wherein they alleged that: (1) the complaint failed to state a cause of action;
(2) the defamatory statements alleged in the complaint were general conclusions without factual premises; (3)
the questioned news report constituted fair and true report on the matters of public interest concerning a public
figure and therefore, was privileged in nature; and (4) malice on their part was negated by the publication in the
same article of plaintiffs or PALs side of the dispute with the pilots union. ALPAP and UMALI likewise filed
their joint answer, and alleged therein that: (1) the complaint stated no cause of action; (2) venue was
improperly laid; and (3) plaintiff Lucio Tan was not a real party in interest. It appeared that the complaint failed
to state the residence of the complainant at the time of the alleged commission of the offense and the place
where the libelous article was printed and first published. However, the case was dismissed by the RTC based
on the ground of improper venue.

RTC - admitted the amended complaint and deemed set aside the previous order of dismissal, inter alia, that:
The mistake or deficiency in the original complaint appears now to have been cured in the Amended Complaint
which can still be properly admitted, pursuant to Rule 10 of the 1997 Rules of Civil Procedure, inasmuch as the
Order of dismissal is not yet final. Besides, there is no substantial amendment in the Amended Complaint which
would affect the defendants’ defenses and their Answers. The Amendment is merely formal, contrary to the
contention of the defendants that it is substantial.

CA - Two petitions for certiorari were filed, one filed by petitioners and the other by defendants Umali and
ALPAP. The two petitions were consolidated. In April 2000, it ordered the dismissal of the petition and thereby
affirming the RTC decision. Motions for reconsideration was filed but was denied.

ISSUE:
WON THE RTC OF MAKATI HAS JURISDICTION OVER THE CASE UPON FILING OF THE
ORIGINAL COMPLAINT FOR DAMAGES.

HELD: YES.

It is settled that jurisdiction is conferred by law based on the facts alleged in the complaint since the latter
comprises a concise statement of the ultimate facts constituting the plaintiff's causes of action. In the case at bar,
after examining the original complaint, we find that the RTC acquired jurisdiction over the case when the case
was filed before it. From the allegations thereof, respondent’s cause of action is for damages arising from libel,
the jurisdiction of which is vested with the RTC. In the case at bar, the additional allegations in the Amended
Complaint that the article and the caricature were printed and first published in the City of Makati referred only
to the question of venue and not jurisdiction. These additional allegations would neither confer jurisdiction on
the RTC nor would respondents failure to include the same in the original complaint divest the lower court of its
jurisdiction over the case. Respondents failure to allege these allegations gave the lower court the power, upon
motion by a party, to dismiss the complaint on the ground that venue was not properly laid.

VINLUAN,  ALEXANDER  MARA  J.  2C  UST  LAW  


Paglaum Management & Development Corp. and Health Marketing Technologies, Inc., vs Union Bank of
the Philippines, Notary Public John Doe, and Register of Deeds of Cebu City and Cebu Province, G.R.
No. 179018, June 18, 2012
FACTS:

Paglaum Management and Development Corporation is the registered owner of three parcels of land located in
Cebu. Union Bank extended HealthTech a credit line and to secure this obligation Paglaum exectuted three
Real Estate Mortgages in favor of Union Bank. The parties entered into a Restructuring Agreement, which
states that any action or proceeding arising out of the transaction shall be commenced in Makati City, with both
parties waiving any other venue. The restructuring was due to HealthTech’s failure to meet its obligations after
the Asian financial crisis adversely affected its business. Heatltech still failed to pay its obligation which
prompted Union Bank to institute foreclosure proceedings. Union Bank extra-judicially foreclosed the
mortgaged properties. The bank filed a Petition for Consolidation of Title after it won the auction sale.

HealthTech later filed a Complaint for Annulment of Sale and Titles with Damages and Application for
Temporary Restraining Order and Writ of Injunction. The complaint was filed in Makati City. Union Bank filed
a motion to dismiss of the grounds of, lack of jurisdiction over the issuance of the injunctive relief, improper
venue, and lack of authority if the person who signed the Complaint. This motion was granted resulting in the
dismissal of the case. Paglaum and HealhTech elevated the case to the CA but was denied.

ISSUE:
WON THE MAKATI CITY IS THE PROPER VENUE TO ASSAIL THE FORECLOSURE OF THE
SUBJECT REM.

HELD: YES.

In the present case, although the action is a real action, where the properties are situated in Cebu,
Paglaum and Union Bank have stipulated that the venue of any case arising from their transaction would
be in Makati City. The phrase “ waive any other venue” shows that the choice of venue was only Makati
City. According to Section 1of Rule 4 of the Rules of Court: Venue of real actions. Actions affecting
title to or possession of real property, or interest therein, shall be commenced and tried in the proper
court which has jurisdiction over the area wherein the real property involved, or a portion thereof, is
situated. According to the Supreme Court in Sps. Lantin v. Lantion, “the general rules on venue of
actions shall not apply where the parties, before the filing of the action, have validly agreed in writing on
an exclusive venue. The mere stipulation on the venue of an action, however, is not enough to preclude
parties from bringing a case in other venues. The parties must be able to show that such stipulation is
exclusive. In the absence of qualifying or restrictive words, the stipulation should be deemed as merely
an agreement on an additional forum, not as limiting venue to the specified place.”

VINLUAN,  ALEXANDER  MARA  J.  2C  UST  LAW  


Polytrade Corp. v. Blanco, 30 SCRA 187

FACTS:

A suit was filed before CFI Bulacan on four causes of action to recover the purchase price of rawhide delivered
by plaintiff to defendant. Plaintiff corporation has its principal office and place of business in Makati, Rizal.
Defendant is a resident of Meycauayan, Bulacan. Defendant moved to dismiss upon the ground of improper
venue. He claims that by contract suit may only be lodged in the courts of Manila. The Bulacan court overruled
him.

ISSUE:

WON VENUE WAS PROPERLY LAID IN BULACAN

HELD: YES.

According to Section 2 (b), Rule 4 of the Rules of Court on venue of personal actions triable by courts of first
instance — and this is one — provides that such "actions may be commenced and tried where the defendant or
any of the defendants resides or may be found, or where the plaintiff or any of the plaintiffs resides, at the
election of the plaintiff." Qualifying this provision in Section 3 of the same Rule which states that venue may be
stipulated by written agreement — "By written agreement of the parties the venue of an action may be changed
or transferred from one province to another." No such stipulation appears in the contracts covering the first two
causes of action. The general rule set forth in Section 2 (b), Rule 4, governs, and as to said two causes of action,
venue was properly laid in Bulacan, the province of defendant's residence. The stipulation adverted to is only
found in the agreements covering the third and fourth causes of action. An accurate reading, however, of the
stipulation, "The parties agree to sue and be sued in the Courts of Manila," does not preclude the filing of suits
in the residence of plaintiff or defendant. The plain meaning is that the parties merely consented to be sued in
Manila. Qualifying or restrictive words which would indicate that Manila and Manila alone is the venue are
totally absent therefrom. We cannot read into that clause that plaintiff and defendant bound themselves to file
suits with respect to the last two transactions in question only or exclusively in Manila. For, that agreement did
not change or transfer venue. It simply is permissive. The parties solely agreed to add the courts of Manila as
tribunals to which they may resort. They did not waive their right to pursue remedy in the courts specifically
mentioned in Section 2(b) of Rule 4. Renuntiatio non praesumitur.

VINLUAN,  ALEXANDER  MARA  J.  2C  UST  LAW  


Sweet Lines Inc. v. Hon. Judge Bernardo Teves, Leovigildo Tandog Jr. and Rogelio Tiro
G.R. No. L-37750, May 19,1978
FACTS:

Respondents ought two tickets in the branch office of Sweet Lines at Cagayan de Oro City (CDO) for Voyage
90 aswere scheduled to board in the petitioner’s vessel M/S Sweet Hope bound for Tagbilaran City via the port
of Cebu. However, upon knowing that the vessel will not anymore proceed to Bohol, respondents went again to
the branch office of Sweet Lines for proper relocation to M/S Sweet Town. And while on board on the said
vessel, the two were forced to hide at the cargoes section of the ship to avoid the inspection being conducted by
the Philippine Coastguard since the vessel already reached its passenger capacity. During the trip, respondents
alleged that they were exposed to the scorching heat of the sun and dust coming from the cargoes. They also
claimed that the tickets they bought in CDO were dishonored as they were constrained to pay for other tickets.
The incident prompted respondents to sue Sweet Lines for damages and for breach of contract of carriage
before the CFI of Misamis Oriental.

CFI - prompted respondents to sue Sweet Lines for damages and for breach of contract of carriage. Sweet Lines
moved to dismiss the case on the ground of improper venue basing the said dismissal on the condition printed at
the back of the tickets that where in any case, all actions arising out of the conditions and provisions of the said
tickets shall only be filed in the courts in the city of Cebu. However, such motion was dismiss.

SC - Sweet Lines filed an instant petition for prohibition for preliminary injunction; praying that the respondent
judge be restrained from proceeding further with the case filed by Tandog and Tiro for grave abuse of discretion
amounting to lack of jurisdiction.

ISSUE:
WON THE CONDITION CONSTITUTE A VALID WAIVER AS TO VENUE.

HELD: YES

The condition cited above is subversive of public policy on transfers of venue of actions. For, although venue
may be changed or transferred from one province to another by agreement of the parties in writing in relation to
Rule 4, Section 3, of the Rules of Court, such agreement will not be held valid where it practically negates the
action of the claimants, such as the private respondents herein. The philosophy underlying the provisions on
transfer of venue of actions is the convenience of the plaintiffs as well as his witnesses and to promote the ends
of justice. Considering the expense and trouble a passenger residing outside of Cebu City would incur to
prosecute a claim in the City of Cebu, he would most probably decide not to file the action at all. The condition
will thus defeat, instead of enhance, the ends of justice. Upon the other hand, petitioner has branches or offices
in the respective ports of call of its vessels and can afford to litigate in any of these places. Hence, the filing of
the suit in the CFI of Misamis Oriental, as was done in the instant case, will not cause inconvenience to, much
less prejudice, petitioner.

VINLUAN,  ALEXANDER  MARA  J.  2C  UST  LAW  


Republic of the Philippines vs. Sandiganbayan et al., G.R. No. 152154, July 15, 2003

FACTS:

Republic, through the Presidential Commission on Good Government (PCGG), filed a petition for forfeiture
before the Sandiganbayan, pursuant to Republic Act 1379. The petition sought the declaration of the amount of
US$356 million deposited in escrow in the PNB, as ill-gotten wealth of the Marcos family. The funds were
previously deposited in Swiss banks under the name of various foreign foundations. Before the case was set for
pre-trial, the Marcos children and then PCGG Chairman Magtanggol Gunigundo executed a “General
Agreement and the Supplemental Agreements” for a global settlement of the assets of the Marcos family. The
Agreement specified in its whereas clause that the Philippine government had "obtained a judgment from the
Swiss Federal Tribunal that the US$356 million belongs in principle to the Republic of the Philippines provided
certain conditionalities are met...”

SANDIGANBAYAN - the Marcos children filed a motion for the approval of said Agreements. While hearings
were being conducted on the said motion, the Republic filed a motion for summary judgment and/or judgment
on the pleadings. It initially granted Republic's motion for summary judgment on the ground that “there is no
issue of fact which calls for the presentation of evidence.” However, it later reversed itself and denied the
motion for summary judgment on the ground that the original copies of the authenticated Swiss decisions and
their "authenticated translations" have not been submitted to the court.

ISSUES:
WON CERTIORARI LIES
WON THERE CAN BE SUMMARY JUDGMENT.

HELD:

1. YES. This case was treated as an exception to the general rule governing petitions for certiorari. Normally,
decisions of the Sandiganbayan are brought before this Court under Rule 45, not Rule 65.[20] But where the
case is undeniably ingrained with immense public interest, public policy and deep historical repercussions,
certiorari is allowed notwithstanding the existence and availability of the remedy of appeal. In all the alleged ill-
gotten wealth cases filed by the PCGG, this Court has seen fit to set aside technicalities and formalities that
merely serve to delay or impede judicious resolution.

2. YES. Summary judgment is sanctioned in this jurisdiction by Section 1, Rule 35 of the 1997 Rules of Civil
Procedure, to wit: SECTION 1. Summary judgment for claimant.- A party seeking to recover upon a claim,
counterclaim, or cross-claim or to obtain a declaratory relief may, at any time after the pleading in answer
thereto has been served, move with supporting affidavits, depositions or admissions for a summary judgment in
his favor upon all or any part thereof. Summary judgment is proper when there is clearly no genuine issue as to
any material fact in the action. The theory of summary judgment is that, although an answer may on its face
appear to tender issues requiring trial, if it is demonstrated by affidavits, depositions or admissions that those
issues are not genuine but sham or fictitious, the Court is justified in dispensing with the trial and rendering
summary judgment for petitioner.
Under the rules, summary judgment is appropriate when there are no genuine issues of fact requiring the
presentation of evidence in a full-blown trial. Even if on their face the pleadings appear to raise issue, if the
affidavits, depositions and admissions show that such issues are not genuine, then summary judgment as
prescribed by the rules must ensue as a matter of law.

VINLUAN,  ALEXANDER  MARA  J.  2C  UST  LAW  


PHiltranco Services Enterprises, Inc. v. Felix Paras and
Inland Trailways, Inc. and Hon. Court of Appeals
G.R. No. 161909, April 25, 2012
FACTS:

Felix Paras, on his way home to Manila from Bicol Region, he boarded a bus owned and operated by Inland
Trailways, Inc. While the said bus was travelling along Maarlika Highway, Tiaong Quezon, it was bumped at
the rear by another bus owned and operated by Philtranco Service Enterprises, Inc. As a result of the strong and
violent impact, the Inland bus was pushed forward and smashed into a cargo truck parked along the outer right
portion of the highway and the shoulder thereof. Inland refused to give financial assistance to Paras, and the
latter filed a complaint for breach of contract of carriage against Inland. In its defense, Inland claims that the
proximate cause of the injuries of Paras was the bus of Philtranco. Inland, with leave of court, filed a third party
complaint against Philtranco and its bus driver. The RTC ordered Philtranco and its driver to pay damages to
Paras. All parties appealed to the CA which affirmed the liability of Philtranco and its driver. It awarded moral
damages and exemplary damages.

ISSUE:
WON THE AWARD OF MORAL DAMAGES AND TEMPERATE DAMAGES IS IMPROPER.

HELD: NO.

As a general rule, indeed, moral damages are not recoverable in an action predicated on a breach of contract. By
way of exception, it may be recoverable even if an action is predicated on breach of contract where (a) the
mishap results in death of a passenger, and (b) where the common carrier has been guilty of fraud or bad faith.

In this case, even if this action does not fall under either exception, the award is proper. Undeniably, Inland
filed its third-party complaint against Philtranco and its driver in order to establish in this action that they,
instead of Inland, should be directly liable to Paras for the physical injuries he had sustained because of their
negligence. The apparent objective of Inland was to obtain a different relief whereby the third-party defendants
would be held directly, fully and solely liable to Paras and Inland for whatever damages each had suffered from
the negligence committed by Philtranco and its driver. In other words, Philtranco and its driver were charged
here as joint tortfeasors who would be jointly and severally be liable to Paras and Inland.

Actual damages, to be recoverable, must not only be capable of proof, but must actually be proved with a
reasonable degree of certainty. The reason is that the court cannot simply rely on speculation, conjecture or
guesswork in determining the fact and amount of damages, but there must be competent proof of the actual
amount of loss, credence can be given only to claims, which are duly supported by receipts.

In this case, although both Paras and Inland both suffered from the negligence of Philtranco and its driver, they
were unable to show sufficient proof as to the actual amount of loss they sustained. Nevertheless, the award of
Temperate Damages was proper, per Art. 2224 of the Civil Code, which states that: “Temperate or moderate
damages, which are more than nominal but less than compensatory damages, may be recovered when the court
finds that some pecuniary loss has been suffered but its amount cannot, from the nature of the case, be proved
with certainty.”

VINLUAN,  ALEXANDER  MARA  J.  2C  UST  LAW  


Georgia T. Estel vs. Heirs of Recaredo P. Diego, Sr. G.R. No. 174082, January 16, 2012

FACTS:

The respondents entered into a contract of sale of a 306 –square-meter parcel of land with Georgia Estel. After
receiving the amount of P17,000.00 as downpayment, Estel delivered the physical and material possession of
the subject property to them; Estel, together with her two sons and five other persons, uprooted the fence
surrounding the disputed lot, after which they entered its premises and then cut and destroyed the trees and
plants found therein; Diegos prayed for the restoration of their possession, for the issuance of a permanent
injunction against Estel as well as payment of damages, attorney’s fees and costs of suit.

MTC - A Complaint for Forcible Entry, Damages and Injunction with Application for Temporary Restraining
Order filed by Recaredo P. Diego, Sr., and Recaredo R. Diego, Jr.; issued a Temporary Restraining Order
against Estel and any person acting in her behalf. It rendered a Decision dismissing Estel’s counterclaim and
ordered her, her agents and representatives.

RTC - rendered its Decision affirming the decision of the MTC.

CA - petition for review; romulgated its Decision which affirmed the Decision of the RTC. She filed a Motion
for Reconsideration, but the CA denied it. She therefore appealed before the Supreme Court.

ISSUE:
WON MTC ACQUIRED JURISDICTION OVER THE SUBJECT MATTER OF THE COMPLAINT
ON THE GROUNDS THAT THE RESPONDENTS FAILED TO ALLEGE THE LOCATION OF THE
DISPUTED AND FAILURE TO SPECIFICALLY ALLEGE FACTS CONSTITUTIVE OF FORCIBLE
ENTRY.
WON THE DEFECTS IN THE VERIFICATION AND CERTIFICATION OF NON-FORUM
SHOPPING IN THE COMPLAINT WILL MAKE IT AN UNSIGNED PLEADING.

HELD:

1. NO. Estel did not raise the issue of jurisdiction or venue in her Answer filed with the MTCC. Even if the
geographical location of the subject property was not alleged in the Complaint, she failed to seasonably object
to it in her Affirmative Defense, and even actively participated in the proceedings before the MTCC. Thus, she
is already estopped from raising the said issue in the CA or before this Court. As to the Diegos’ failure to allege
facts constitutive of forcible entry, it is settled that in actions for forcible entry, two allegations are mandatory
for the municipal court to acquire jurisdiction: the plaintiff must allege his prior physical possession of the
property, and he must also allege that he was deprived of his possession by means of force, intimidation, threats,
strategy, and stealth. In this case, it is clear that the Diegos sufficiently alleged in their Complaint the material
facts constituting forcible entry, as they explicitly claimed that they had prior physical possession of the subject
property since its purchase from Estel. They also particularly described in their complaint how Estel, her two
sons and five other persons, encroached upon the subject property and dispossessed them of the same.
2. NO. The alleged defect in the verification was not raised before the MTCC. Even granting that this matter
was properly raised before the court a quo, the Court finds that there is no procedural defect that would have
warranted the outright dismissal of respondents’ complaint as there is compliance with the requirement
regarding verification. A reading of the Diegos’ verification reveals that they complied with the the rule on
verification; they confirmed that they had read the allegations in the Complaint which were true and correct
based on their personal knowledge.
VINLUAN,  ALEXANDER  MARA  J.  2C  UST  LAW  
Pascual and Santos, Inc. v. The Members of the Tramo Wakas Neighborhood Association, Inc., G.R. No.
144880, November 17, 2004

FACTS:

The respondent members lodged before the Presidential Action Center a petition praying that ownership over
three parcels of land be awarded to them. In their petition, respondents alleged that petitioner claims ownership
of the subject lots which they have openly, peacefully and continuously occupied since 1957.

LMB - petition was referred to the Land Management Bureau (LMB) for investigation and hearing. The
Director of the LMB in favor of the respondents eventually decided the petition.

DENR - Its Motion for Reconsideration having been denied, petitioner lodged an appeal before the Office of the
DENR Secretary, which was subsequently dismissed for lack of merit and affirmed the decision of the Director
of the LMB. Petitioner’s appeal to the Office of the President was likewise dismissed.

CA - filed its Petition for Review; dismissed the appeal due to infirm Verification and Certification of non-
forum shopping. The Verification and Certification of non-forum shopping was signed merely by Estela
Lombos and Anita Pascual who allege that they are the duly authorized representatives of petitioner
corporation, without showing any proof whatsoever of such authority. For another, and importantly, the petition
for review was filed a day after the period petitioner corporation was supposed to do so. Petitioner filed a
Motion for Reconsideration, arguing that there was no showing that the persons acting on its behalf were not
authorized to do so. CA denied the MR. Hence, petitioner filed a Petition for Review on Certiorari to the SC.

ISSUES:

WON THE PERSONS WHO EXECUTED THE VERIFICATION AND CERTIFICATION OF NON-
FORUM SHOPPING ATTACHED TO PSI'S MANIFESTATION/PETITION FOR REVIEW FILED
WITH THE COURT OF APPEALS WERE AUTHORIZED TO DO SO.

HELD: YES

The requirement under the Rules of Court that the petitioner should sign the certificate of non-forum shopping
applies even to corporations, considering that the mandatory directives of the Rules of Court make no
distinction between natural and juridical persons. It is undisputed that when the petition for certiorari was filed
with the CA, there was no proof attached thereto that Lombos and Pascual were authorized to sign the
verification and non-forum shopping certification. Subsequent to the CA's dismissal of the petition, however,
petitioner filed a motion for reconsideration to which it attached a certificate issued by its board secretary
stating that on February 11, 2000 or prior to the filing of the petition, Lombos and Pascual had been authorized
by petitioner's board of directors to file the petition before the CA. This Court has ruled that the subsequent
submission of proof of authority to act on behalf of a petitioner corporation justifies the relaxation of the Rules
for the purpose of allowing its petition to be given due course. It must also be kept in mind that while the
requirement of the certificate of non-forum shopping is mandatory, nonetheless the requirements must not be
interpreted too literally and thus defeat the objective of preventing the undesirable practice of forum shopping.

VINLUAN,  ALEXANDER  MARA  J.  2C  UST  LAW  


ELSA D. MEDADO, petitioner, vs. HEIRS OF THE LATE ANTONIO CONSING, as represented by DR.
SOLEDAD CONSING, respondents.
G.R. No. 186720. February 8, 2012.
FACTS:
Spouses Meritus Rey Edado and, herein petitioner, Elsa D. Medado and the estate of the late Antonio Consing
(Estate of Consing), as represented by Soledad Consing, executed Deeds of Sale with Assumption of Mortgage,
wherein the latter sold to the said spouses the property known as Hacienda Sol situated in Cadiz City. As part of
the deal, Spouses Medado assumed the loan obligation of the Estate of Consing with the PNB. Subsequently,
however, the Estate of Consing offered the subject property to the government through the Department of
Agrarian Reform’s Voluntary Offer to Sell (VOS) Program.
RTC Bacolod - The Estate of Consing instituted an action for rescission and damages against the Spouses
Medado, PNB and the Register of Deeds (RD) of Cadiz City, due to alleged failure of the Spouses Medado to
meet the conditions in their agreement. Land Bank of the Philippines (LBP) issued in favor of the Estate of
Consing a certificate of deposit of cash and agrarian reform bonds, as compensation for the property covered by
the VOS.
RTC Cadiz City - Spouses Medado filed an action for injunction with prayer for the issuance of temporary
restraining order Because of the fear of the Spouses Medado that the whole proceeds of the VOS might be
released to the Estate of Consing, since the former believed that they the ones entitled to such proceeds by
virtue of the Deeds of Assignment with Assumption of Mortgage. They prayed that a writ of prohibitory
injunction be issued to restrain LBP from releasing the remaining amount of the VOS proceeds to the Estate of
Consing, and restraining the Estate of Consing from receiving these proceeds; and also prayed for the issuance
of a writ of mandatory injunction to compel LBP to release the remaining amount of the VOS to the spouses. It
ruled in favor of Spouses Medado.
CA - filed a petition for certiorari. It should be noted that the petition filed by the Heirs of Consing was signed
only by Soledad Consing and not by all of the petitioners in the CA. The CA ruled in favor the Estate of
Consing. The CA denied the subsequent motion for reconsideration filed by Spouses Medado.

ISSUES:
WON THE CA WAS CORRECT IN ADMITTING THE PETITION FOR CERTIORARI DESPITE
THE FACT THAT THE SAID PETITION WAS DEFECTIVE BECAUSE OF THE DEFICIENCIES IN
THE VERIFICATION AND CERTIFICATION AGAINST FORUM SHOPPING.

HELD: YES.
The Court said that by looking at the records, Soledad Consing possesses a Special Power of Attorney (SPA)
wherein Soldedad Consing was given the authority “to protect, sue, prosecute, defend and adopt whatever
action necessary and proper relative and with respect to her co-heirs’ right, interest and participation over the
subject property.” According to the Court, the authority of Soledad includes the filing of an appeal before the
CA, including the execution of a verification and certification against forum shopping therefor, being acts
necessary “to protect, sue, prosecute, defend and adopt whatever action necessary and proper” in relation to
their rights over the subject properties. The Court further said that the verification requirement is simply
intended to secure an assurance that the allegations in the pleading are true and correct, and not the product of
the imagination or a matter of speculation, and that the pleading is filed in good faith and that where the
petitioners are immediate relatives, who share a common interest in the property subject of the action, the fact
that only one of the petitioners executed the verification or certification of forum shopping will not deter the
court from proceeding with the action. Thus, the Court may simply order the correction of unverified pleadings
or act on them and waive strict compliance with the Rules. It is deemed substantially complied with when one
who has ample knowledge to swear to the truth of the allegations in the complaint or petition signs the
verification; and when matters alleged in the petition have been made in good faith or are true and correct.

VINLUAN,  ALEXANDER  MARA  J.  2C  UST  LAW  


Vivian T. Ramirez, et al. v. Mar Fishing Co., Inc., et al., G.R. No. 168208, June 13, 2012

FACTS:

The respondent, engaged in the business of fishing and canning of tuna, sold its principal assets to co-
respondent Miramar Fishing Co., Inc. through public bidding. The proceeds of the sale were paid to the Trade
and Investment Corporation of the Philippines to cover Mar Fishings outstanding obligation in the amount of
₱897,560,041.26. In view of that transfer, Mar Fishing issued a Memorandum informing all its workers that the
company would cease to operate by the end of the month. The respondent notified DOLE of the closure of its
business operation. Thereafter, Mar Fishing’s labor union, Mar Fishing Workers Union NFL and Miramar
entered into a Memorandum of Agreement. The Agreement provided that the acquiring company, Miramar,
shall absorb Mar Fishing’s regular rank and file employees whose performance was satisfactory, without loss of
seniority rights and privileges previously enjoyed. Unfortunately, petitioners, who worked as rank and file
employees, were not hired or given separation pay by Miramar.

NLRC - petitioners filed Complaints for illegal dismissal with money claims.

LA - found that Mar Fishing had necessarily closed its operations, considering that Miramar had already bought
the tuna canning plant. By reason of the closure, petitioners were legally dismissed for authorized cause.

Consequently, the LA ordered Mar Fishing to give separation pay to its workers. However, the NLRC reversed
the decision of the LA and pierced the veil of corporate fiction and ruled that Mar Fishing and Miramar were
one and the same entity, since their officers were the same. The labor court held that petitioners had no cause of
action against Miramar, since labor contracts cannot be enforced against the transferee of an enterprise in the
absence of a stipulation in the contract that the transferee assumes the obligation of the transferor.

CA - instantly dismissed the action for certiorari against the 225 other petitioners without ruling on the
substantive aspects of the case. However, by means of a Manifestation with Omnibus Motion, petitioners
submitted a Verification and Certification against forum shopping executed by 161 signatories. CA denied
petitioners contentions holding that even though litigation is not a game of technicalities, it does not follow that
the Rules of Court must be ignored.

ISSUE:

WON THE CA GRAVELY ERRED IN DISMISSING THEIR PETITION FOR REVIEW ON THE
GROUND THAT THEIR PLEADING LACKED A VERIFICATION AND CERTIFICATION
AGAINST FORUM SHOPPING.

HELD: NO.

Rules of Court provide that a petition for certiorari must be verified and accompanied by a sworn certification
of non-forum shopping. Failure to comply with these mandatory requirements shall be sufficient ground for the
dismissal of the petition. Considering that only 3 of the 228 named petitioners signed the requirement, the CA
dismissed the case against them, as they did not execute a Verification and Certification against forum
shopping. The lack of certification against forum shopping is not curable by mere amendment of a complaint,
but shall be a cause for the dismissal of the case without prejudice. Indeed, the general rule is that
subsequent compliance with the requirements will not excuse a party's failure to comply in the first
instance. Thus, on procedural aspects, the appellate court correctly dismissed the case. However, this Court has
recognized that the merit of a case is a special circumstance or compelling reason that justifies the relaxation of
the rule requiring verification and certification of non-forum shopping.

VINLUAN,  ALEXANDER  MARA  J.  2C  UST  LAW  


ATTY. FE Q. PALMIANO-SALVADOR, Petitioner,
vs. CONSTANTINO ANGELES, Respondent.
G.R. No. 171219, September 3, 2012
FACTS:

Respondent is one of the registered owners of a parcel of land at Sampaloc, Manila as evidence by a Transfer
Certificate of Title. The subject parcel of land was occupied by Galiga from 1979 to 1993 as a lessee with a
lease of contract. Subsequently, Petitioner alleged she bought on 1993 the subject parcel of land from Galiga
who represented that he was the owner, being one in possession. Petitioner remained in possession of said
property. Respondent sent a letter to petitioner demanding the latter to vacate the property which was not
heeded by petitioner.

Thus, Angeles filed, thru one Diaz, a complaint for ejectment with the MeTC. MeTC rendered its decision in
favor of Angeles. In appeal filed by Salvador, she alleged that Diaz, who filed the complaint, had no authority
from Angeles at the time of the filing of the suit. The appeal was denied by the RTC. The Motion for
Reconsideration filed by Salvador was likewise denied. Petitioner elevated the case to the CA via petition for
review but was dismissed. The CA affirmed the factual findings of lower courts that Galiga was a mere lessee
of respondent hence Galiga could not have validly transferred ownership to peitioner. Hence, the present
petition.

ISSUE:

WON THE COURT ACQUIRED JURISDICTION OVER THE COMPLAINT AND THE PLAINTIFF.

HELD: NO.

The complaint filed in the MeTC was filed in the name of the respondent but it was one Diaz who executed the
verification and certification alleging therein that he was respondent’s attorney-in-fact. No copy of any
document attached to the complaint to prove Diaz/s allegation regarding the authority supposedly granted to
him. In fact, it was only more than ayear after the complaint was filed that respondent attached to his Reply to
Salvador a document entitled Special Power of Attorney (SPA) supposedly executed in favor of Diaz. However,
said SPA was executed only more than a month after the complaint was filed appearing to have been notarize
by one Robert McGuire of Santa Barbara County. No certification from Philippine Consulate in San Francisco,
CA, USA, that said person is indeed a notary public. There is nothing on record to show that Diaz had been
authorized. Thus, the effect is as held by the Court in Tamondong v. CA, if a complaint is filed for and in behalf
of the plaintiff by one who is not authorize to do so, the complaint is not deemed filed. An unauthorize
complaint does not produce any legal effect. Hence, the court has no jurisdiction over the complaint and the
plaintiff. In order that the court to have authority to dispose of the case on the merits, it must acquire
jurisdiction over the subject matter and the parties. Courts acquire jurisdiction over the plaintiffs upon filing of
the complaint and to be bound by the decision, a party should first be subjected to the court’s jurisdiction. Since
no valid complaint was ever filed witht the MeTC, it did not acquire jurisdcition over the person of Angeles.
Thus, all proceedings before the MeTC were null and void.

VINLUAN,  ALEXANDER  MARA  J.  2C  UST  LAW  


Ceroferr Realty Corporation vs Court of Appeals and Ernesto D. Santiago G.R. 139539, February 5,2002

FACTS:

RTC – plaintiff filed a complaint against defendant Ernesto D. Santiago for “damages and injunction, with
preliminary injunction.” Ceroferr prayed that Santiago and his agenst be enjoined from claiming possession and
ownership aver Lot. 68 of the Tala Estate Subdivision, Quezon City; that Santiago and his agents be prevented
from making use of the vacant lot as a jeepney terminal; and that Santiago be ordered to pay Ceroferr P650.00
daily as lost income for the use of the lot until possession is restored to the latter.

There was a verification survey, followed by a relocation survey, whereby it would appear that the vacant lot is
inside Lot. 68. The outcome of the survey was vigorously objected to by defendant who insisted that the area is
inside his lot. It thus became clear, at least from the view point of the defendant, that the case would no longer,
merely involves a simple case of collection of damages and injunction which was the main objective of the
complaint but a review of the title of defendant vis-a-vis that of the plaintiff. At this point, defendant filed a
motion to dismiss the complaint premised primarily on his contention that the trial court cannot adjudicate the
issue of damages without passing over the conflicting claims of ownership of the parties over the disputed
portion.

The RTC issued the order that dismissed the case for lack of cause of action and lack of jurisdiction. Plaintiff
appealed this decision but the Court of Appeals promulgated a decision dismissing the appeal.

ISSUE:

WON CEROFERR’S COMPLAINT STATES A SUFFICIENT CAUSE OF ACTION


WON THE TRIAL COURT HAS JURISDICTION TO DETERMINE THE IDENTITY AND
LOCATION OF THE VACANT LOT INVOLVED IN THE CASE.

HELD:

1. YES. The rule of procedure require that the complaint must state a concise statement of the ultimate facts or
essential facts constituting the plaintiff`s cause of action. A fact is essential if it cannot be stricken out without
leaving the statement cause of action inadequate. A complaint states a cause of action when it has three
indispensable elements, namely: 1) a right in favor of the plaintiff by whatever means and under whatever law it
arises or created; 2) an obligation on the part of the named defendant to respect or not to violate such right; 3)
an act or omission on the part of such defendant violative of the right of plaintiff or constituting a breach of the
obligation of defendant to the plaintiff for which the latter may maintain an action for recovery for damages. If
these elements are not extant, the complaint becomes vulnerable to a motion to dismiss on the ground of failure
to state a cause of action. In this case, petitioner Ceroferr's cause of action has been sufficiently averred in the
complaint. If it were admitted that the right of ownership of petitioner Ceroferr to the peaceful use and
possession of Lot 68 was violated by respondent Santiago's act of encroachment and fencing of the same, then
petitioner Ceroferr would be entitled to damages.

2. YES. While the lack of jurisdiction of a court may be raised at any stage of an action, the party raising such
question may be estopped if he has actively taken part in the very proceedings which he questions and he only
objects to the court`s jurisdiction because the judgment or the order subsequently rendered is adverse to him.
In this case, respondent Santiago may be considered estopped to question the jurisdiction of the trial court for
he took an active part in the case. In his answer, respondent Santiago did not question the jurisdiction of the trial
court to grant the reliefs prayed for in the complaint. His geodetic engineers were present in the first and second
surveys that the LRA conducted. It was only when the second survey report showed results adverse to his case
that he submitted a motion to dismiss.
VINLUAN,  ALEXANDER  MARA  J.  2C  UST  LAW  
Asian Construction and Development Corp. vs. Lourdes K. Mendoza

FACTS:

RTC - respondent Lourdes K. Mendoza, sole proprietor of Highett Steel Fabricators (Highett), filed a Complaint
for a sum of money, against petitioner Asian Construction and Development Corporation, a duly registered
domestic corporation. In the complaint, respondent alleged that petitioner purchased from Highett various
fabricated steel materials and supplies and despite demand, petitioner failed and/or refused to pay; and that due
to the failure and/or refusal of petitioner to pay the said amount, respondent was compelled to engage the
services of counsel.

Petitioner moved for a bill of particulars on the ground that no copies of the purchase orders and invoices were
attached to the complaint to enable petitioner to prepare a responsive pleading to the complaint. To prove her
case, respondent presented the testimonies of the salesman of Highett who confirmed the delivery of the
supplies and materials to petitioner, and the General Manager of Highett. Petitioner argues that a charge or sales
invoice is not an actionable document; thus, petitioners failure to deny under oath its genuineness and due
execution does not constitute an admission thereof. Petitioner likewise insists that respondent was not able to
prove her claim as the invoices offered as evidence were not properly authenticated by her witnesses.

The RTC ruled in favor of the respondent and ordered the payment of the principal amount of the supplies and
the amount of the accrued interest, 150,000 as attorney's fees and cost of suit. The CA modified the decision of
the RTC.

ISSUES:

WON THE CHARGE INVOICES ARE ACTIONABLE DOCUMENTS.


WON THE DELIVERY OF THE ALLEGED MATERIALS WAS DULY PROVEN
WON RESPONDENT IS ENTITLED TO ATTORNEY’S FEES

HELD:

1. YES. From the provision of Rule 8 Sec. 7 of RoC, a document is actionable when an action or defense is
grounded upon such written instrument or document. In the instant case, the Charge Invoices are not actionable
documents per se as these only provide details on the alleged transactions. These documents need not be
attached to or stated in the complaint as these are evidentiary in nature. In fact, respondent's cause of action is
not based on these documents but on the contract of sale between the parties.

2. YES. Although the Charge Invoices are not actionable documents, the Purchase Orders, are sufficient to
prove that petitioner indeed ordered supplies and materials from Highett and that these were delivered to
petitioner. Moreover, contrary to the claim of petitioner, the Charge Invoices were properly identified and
authenticated by witness who was present when the supplies and materials were delivered to petitioner and
when the invoices were stamped received by petitioner's employee.

3. NO. However, with respect to the award of attorneys fees to respondent, it must be disallowed as the same as
the rationale for the award was not stated in the text of the RTC Decision but only in the dispositive portion.

VINLUAN,  ALEXANDER  MARA  J.  2C  UST  LAW  


SPOUSES FERNANDO and MA. ELENA SANTOS vs LOLITA ALCAZAR
March 12, 2014; GR 183034
FACTS:

Respondent Alcazar was the proprieter of Legazpi Color Center. Alcazar filed a complaint for sum of money
against petitioners for the collection of the value of paint and construction materials in the amount of 1,456,000.
The Acknowledgment executed by Petitioner Fernando was the basis of Alcazar’s cause of action. Under the
said Acknowledgment, Petitioner Fernando acknowledged his obligation with LCC to pay the value of the paint
and construction materials in the amount of 1,456,000. Petitioners specifically denied the allegations of Alcazar
and contended that the actionable document does not reflect their real indebtedness as well as the true contract
or intention of the parties. The document does not reflect the correct amount which is only P600,000. Thus,
petitioners sought the dismissal of the Complaint.

RTC - rendered judgment in favor of respondent Alcazar and ordered petitioners to pay plaintiff the sum of
1,456,000 plus interest, litigation and attorney’s fees. It held that petitioners admitted that they entered into
transactions with Alcazar for the delivery of paint and construction materials and that from the
Acknowledgment, petitioners admitted that their unpaid obligation is P1,456,000.

CA - the petitioners alleged that the TC erred in allowing Alcazar to present her evidence ex parte. The CA held
that petitioners failed to deny specifically under oath the genuineness and due execution of the
Acknowledgment, consequently, its genuineness and due execution are deemed admitted, there was thus no
need to present the original thereof, and petitioners’ liability was sufficiently established.

ISSUES:

WON THE PRE-TRIAL CONFERENCE IS A SHAM AS THERE ARE NO RECORDS TO SHOW


THAT IT WAS CONDUCTED.

HELD: NO

Court found them to be without merit. A pre-trial conference was held, and that petitioners' representative was
present therein. Moreover, the proceedings were covered by the required pre-trial order, which may itself be
considered a record of the pre-trial. In said order, the November 8, 2005 pre-scheduled hearing was particularly
specified. Thus, from the very start, petitioners knew of the November 8 hearing; if they failed to attend, no
fault may be attributed to the trial court.

VINLUAN,  ALEXANDER  MARA  J.  2C  UST  LAW  


Tantuico v. Republic G.R. No. 89114 December 2, 1991

FACTS:

In the case for reconveyance, reversion, accounting, restitution and damages filed by the Republic of the
Philippines against Benjamin Romualdez, Ferdinand Marcos, and Imelda Marcos, herein petitioner Francisco
Tantuico was impleaded as defendant, the theory that he acted in unlawful concert with Romualdez, et al. in the
misappropriation and theft of public funds, among others. After Tantuico’s motion for production and
inspection of documents was denied by the Sandiganbayan, he filed a Motion for a Bill of Particulars, alleging
that the complaint filed against him was couched in too general terms and shorn of particulars that would inform
him of the factual and legal bases thereof. The Sandiganbayan thereafter denied Tantuico’s motion on the
ground that the particulars sought by the latter are evidentiary in nature, thus not necessary in the filing of the
complaint. From such order of the Sandiganbayan, Tantuico moved for reconsideration, which motion was also
denied by the aforesaid court. This prompted Tantuico to file a petition for certiorari, mandamus, and
prohibition against the Sandiganbayn, on the ground of grave abuse of discretion, when such court denied his
motion for a bill of particulars, even though the complaint of the Republic of the Philippines failed to properly
provide a concise statement of the facts necessary for him to prepare his defense.

ISSUE:

WON THE COMPLAINT PROPERLY PROVIDED A STATEMENT OF THE FACTS:

HELD: YES

A complaint is defined as a concise statement of the ultimate facts constituting the plaintiff's cause or causes of
action. Like all other pleadings allowed by the Rules of Court, the complaint shall contain in a methodical and
logical form a plain, concise and direct statement of the ultimate facts on which the plaintiff relies for his claim,
omitting the statement of mere evidentiary facts. Its office, purpose or function is to inform the defendant
clearly and definitely of the claims made against him so that he may be prepared to meet the issues at the trial.
The complaint should inform the defendant of all the material facts on which the plaintiff relies to support his
demand; it should state the theory of a cause of action that forms the bases of the plaintiff's claim of liability.
Where the complaint states ultimate facts that constitute the three essential elements of a cause of action, the
complaint states a cause of action; otherwise, the complaint must succumb to a motion to dismiss on that ground
of failure to state a cause of action.

However, where the allegations of the complaint are vague, indefinite, or in the form of conclusions, the proper
recourse would be, not a motion to dismiss, but a motion for a bill of particulars. Anent the contention of the
Solicitor General that the petitioner is not entitled to a bill of particulars because the ultimate facts constituting
the three essential elements of a cause of action for recovery of ill-gotten wealth have been sufficiently alleged
in the complaint, it would suffice to state that in a motion for a bill of particulars, the only question to be
resolved is whether or not the allegations of the complaint are averred with sufficient definiteness or
particularity to enable the movant properly to prepare his responsive pleading and to prepare for trial. As
already discussed, the allegations of the complaint pertaining to the herein petitioner are deficient because the
averments therein are mere conclusions of law or presumptions, unsupported by factual premises.

VINLUAN,  ALEXANDER  MARA  J.  2C  UST  LAW  


D.M. FERRER & ASSOCIATES CORPORATION v. UNIVERSITY OF SANTO TOMAS
G.R. No. 189496. February 1, 2012
FACTS:
Petitioner and University of Santo Tomas Hospital, Inc. entered into a Project Management Contract for the
renovation of the 4th and 5thfloors of the Clinical Division Building, Nurse Call Room and Medical Records,
Medical Arts Tower, Diagnostic Treatment Building and Pay Division Building. Petitioner demanded from
USTHI the payment of the construction costs amounting to P17,558,479.39. However, UST, wrote a letter
informing petitioner that its claim for payment had been denied, because the Project Management Contract was
without the required prior approval of the board of trustees. Thus, petitioner filed a Complaint for sum of
money, breach of contract and damages against herein respondent UST and USTHI when the latter failed to pay
petitioner despite repeated demands. Thus, petitioner posited in part that UST may be impleaded in the case
under the doctrine of piercing the corporate veil, wherein respondent UST and USTHI would be considered to
be acting as one corporate entity, and UST may be held liable for the alleged obligations due to petitioner.
Subsequently, respondent filed its Motion to Dismiss. It alleged that the Complaint failed to state a cause of
action, and that the claim was unenforceable under the provisions of the Statute of Frauds.

RTC - granted the motion and dismissed the Complaint insofar as respondent UST was concerned, on the
ground that respondent was not a real party-in-interest, and that it was not privy to the contract executed
between USTHI and petitioner. Second, the court pointed out that the alleged verbal assurances of Fr. Dela
Rosa should have been in writing to make these assurances binding and demandable.

CA - petitioner filed a Petition for Certiorari under Rule 65 alleging that the trial court committed grave abuse
of discretion when it granted respondents Motion to Dismiss on the basis of the documents submitted in support
of the Complaint, and not solely on the allegations stated therein. It pointed out that the allegations raised
questions of fact and law, which should have been threshed out during trial, when both parties would have been
given the chance to present evidence supporting their respective allegations. It further issued the assailed
Resolution and dismissed the Petition on the ground that a petition under Rule 65 is the wrong remedy to
question the RTCs Order that completely disposes of the case. Instead, petitioner should have availed itself of
an appeal under Rule 41 of the Rules of Court.

ISSUES:

WON THE CA ERRED IN DISMISSING THE PETITION FOR CERTIORARI BY FAILING TO


CONSIDER THE EXCEPTIONS IN SEC. 1(G) OF RULE 41.
WON THE TRIAL COURT COMMITTED GRAVE ABUSE OF DISCRETION WHEN IT HELD
THAT THE COMPLAINT STATED NO CAUSE OF ACTION.

HELD:

1. YES. A petition for certiorari under Rule 65 is the proper remedy to question the dismissal of an action
against one of the parties while the main case is still pending. While Section 1, Rule 41 of the 1997 Rules of
Civil Procedure states that an appeal may be taken only from a final order that completely disposes of the case,
it also provides several exceptions to the rule, to wit: (a) an order denying a motion for new trial or
reconsideration; (b) an order denying a petition for relief or any similar motion seeking relief from judgment;
(c) an interlocutory order; (d) an order disallowing or dismissing an appeal; (e) an order denying a motion to set
aside a judgment by consent, confession or compromise on the ground of fraud, mistake or duress, or any other
ground vitiating consent; (f) an order of execution; (g) a judgment or final order for or against one or more of
VINLUAN,  ALEXANDER  MARA  J.  2C  UST  LAW  
several parties or in separate claims, counterclaims, cross-claims and third-party complaints, while the main
case is pending, unless the court allows an appeal therefrom; and (h) an order dismissing an action without
prejudice. In the foregoing instances, the aggrieved party may file an appropriate special civil action for
certiorari under Rule 65.

2. YES. While it is admitted that respondent UST was not a party to the contract, petitioner posits that the
former is nevertheless liable for the construction costs. In support of its position, petitioner alleged that (1) UST
and USTHI are one and the same corporation; (2) UST stands to benefit from the assets of USTHI by virtue of
the latters Articles of Incorporation; (3) respondent controls the business of USTHI; and (4) USTs officials have
performed acts that may be construed as an acknowledgement of respondents liability to petitioner. Obviously,
these issues would have been best resolved during trial. The RTC therefore committed grave abuse of discretion
when it dismissed the case against respondent for lack of cause of action. The trial court relied on the contract
executed between petitioner and USTHI, when the court should have instead considered merely the allegations
stated in the Complaint.

VINLUAN,  ALEXANDER  MARA  J.  2C  UST  LAW  


La Mallorca v. Court of Appeals, Mariano Beltran, et al., G.R, No. L-20761, July 27, 1966.

FACTS:

Private respondent Mariano Beltran, along with his wife and three children, Milagros, Racquel and Fe, were the
passengers of a bus owned and operated by La Mallorca. Upon reaching their destination, they all alighted from
the bus towards a shaded part of the side of the road. Respondent returned to the bus to retrieve their
belongings, and unknown to him, his daughter, followed him. Respondent stayed on the running board of the
bus, waiting for the conductor to hand him his bag, when the driver accelerated, without any signal coming
from the conductor, which prompted respondent to jump from the bus, which had moved about ten feet. Upon
his return to the place where he left his family, he went over to where a number of people were gathered,
looking down at the body of a girl who had her head crushed, and was lifeless. The girl turned out to be his
daughter, Racquel. Suits were initiated, first, for breach of contract of carriage against La Mallorca, and second,
for quasi-delict. La Mallorca, in its answer, stated that the two actions were incompatible, and this moved for
dismissal of the case. The trial court, as well as the Court of Appeals, which both found La Mallorca liable for
quasi-delict, although the contract of carriage was found to have already been terminated, denied this motion
filed by La Mallorca.

ISSUE:

WON THERE IS INCOMPATIBILITY BETWEEN THE TWO ACTIONS.

HELD: NO

The plaintiffs sufficiently pleaded the culpa or negligence upon which the claim was predicated when it was
alleged in the complaint that "the death of Raquel Beltran, plaintiffs' daughter, was caused by the negligence
and want of exercise of the utmost diligence of a very cautious person on the part of the defendants and their
agent." This allegation was also proved when it was established during the trial that the driver, even before
receiving the proper signal from the conductor, and while there were still persons on the running board of the
bus and near it, started to run off the vehicle. The presentation of proof of the negligence of its employee gave
rise to the presumption that the defendant employer did not exercise the diligence of a good father of the family
in the selection and supervision of its employees. And this presumption, as the Court of Appeals found,
petitioner had failed to overcome. Even assuming arguendo that the contract of carriage has already terminated,
herein petitioner can be held liable for the negligence of its driver, as ruled by the Court of Appeals, pursuant to
Article 2180 of the Civil Code.

VINLUAN,  ALEXANDER  MARA  J.  2C  UST  LAW  


Ledda v. BPI, G.R. No. 200868, November 21, 2012

FACTS:

Anita Ledda was a client of BPI who was issued a pre-approved credit card. This credit card and its terms and
conditions were delivered to Ledda; Ledda began to subsequently use the credit card for various purchases of
goods, services, and cash advances. Ledda defaulted in the payment of her credit card obligations. BPI sent a
demand letter, but this was received in Ledda on October 2, 2007 who defaulted payment nonetheless.

RTC - BPI filed the complaint; RTC declared Ledda in default for failing to file an Answer within the period
prescribed despite the service of the complaint and summons. Ledda somehow managed to file a motion for
reconsideration, which was granted by the RTC and thus allowed her to file her Answer Ad Cautelam. Ledda
and her counsel, however, failed to appear during the Pre-Trial. BPI was thus allowed to present evidence ex-
parte. The RTC later ruled in favor of BPI, making Ledda liable for the alleged amount.

CA - denied her appeal but modified the RTC’s decision. The CA rejected Ledda’s argument that the credit
card’s terms and conditions were actionable documents governed by Rule 8, sec. 7 of the Rules of Civil
Procedure. The CA held that BPI’s cause of action sprang from Ledda’s availment of the bank’s credit facilities
via the credit card and her refusal to pay the obligation it created. The CA used the case of Macalinao vs BPI in
order to reduce the payment of interest.

ISSUES:

WON THE CA ERRED IN HOLDING THAT THE CREDIT CARD’S TERMS AND CONDITIONS
WERE NOT ACTIONABLE DOCUMENTS UNDER RULE 8, SEC. 7 OF THE RULES OF CIVIL
PROCEDURE.

HELD: NO

Rule 8, section 7 provides:

Section 7. Action or defense based on document. — Whenever an action or defense is based upon a written
instrument or document, the substance of such instrument or document shall be set forth in the pleading, and
the original or a copy thereof shall be attached to the pleading as an exhibit, which shall be deemed to be a part
of the pleading, or said copy may with like effect be set forth in the pleading.

As the rule itself states, the cause of action must be based on a document to be attached to the complaint. BPI’s
cause of action, however, was not solely based on the credit card’s terms and conditions. It was also based on
Ledda’s 1) acceptance of the credit card, 2) use of the credit card to purchase goods or services or advances in
cash, and 3) her default in the payment of her obligations. As such, BPI had a sufficient cause of action with or
without the attachment of the credit card’s terms and conditions.

VINLUAN,  ALEXANDER  MARA  J.  2C  UST  LAW  


FINANCIAL BUILDING CORPORATION, petitioner, vs. FORBES PARK ASSOCIATION,
INC., respondent.

FACTS:

The USSR, owner of a residential lot located at Forbes Park Village in Makati City, engaged the services of
Financial Building for the construction of a multi-level building of the said lot. Forbes Park reminded the USSR
of existing regulations authorizing only the construction of a single-family residential building in each lot
within the village. It also elicited a reassurance from the USSR that such restriction has been complied with.
Financial Building submitted to the Makati City Government a second building plan for the construction of a
multi-level apartment building, which was different from the first plan for the construction of a residential
building submitted to Forbes Park. Forbes Park discovered the second plan and subsequent ocular inspection
confirmed the violation of the deed of restrictions. Thus, it enjoined further construction work. Forbes Park
suspended all permits of entry for the personnel and materials of Financial Building in the said construction site.
The parties attempted to meet to settle their differences but it did not push through.

RTC (1) - Complaint for Injunction and Damages with a prayer for Preliminary Injunction against Forbes Park.
The latter, in turn, filed a Motion to Dismiss on the ground that Financial Building had no cause of action
because it was not the real party-in-interest. The trial court issued a writ of preliminary injunction against
Forbes Park but the Court of Appeals nullified it and dismissed the complaint.

The Supreme Court affirmed the said dismissal in a Resolution.

RTC (2) - Forbes Park sought to vindicate its rights by filing a Complaint for Damages, against Financial
Building arising from the violation of its rules and regulations. The trial court rendered its decision in favor of
Forbes Park.

CA – affirmed the decision of the RTC.

ISSUE:

WON FORBES PARK IS BARRED IN CLAIMING DAMAGES.

HELD: YES. The instant case is barred due to Forbes Park’s failure to set it up as a compulsory counterclaim
in the prior injunction suit initiated by Financial Building against Forbes Park. Thus, a compulsory counterclaim
cannot be the subject of a separate action but it should instead be asserted in the same suit involving the same
transaction or occurrence, which gave rise to it. To determine whether a counterclaim is compulsory or not, we
have devised the following tests: (1) Are the issues of fact or law raised by the claim and the counterclaim
largely the same? (2) Would res judicata bar a subsequent suit on defendant’s claim absent the compulsory
counterclaim rule? (3) Will substantially the same evidence support or refute plaintiff’s claim as well as the
defendant’s counterclaim? and (4) Is there any logical relation between the claim and the counterclaim?
Affirmative answers to the above queries indicate the existence of a compulsory counterclaim. Thus, the filing
of a motion to dismiss and the setting up of a compulsory counterclaim are incompatible remedies. In the event
that a defending party has a ground for dismissal and a compulsory counterclaim at the same time, he must
choose only one remedy. If he decides to file a motion to dismiss, he will lose his compulsory counterclaim. But
if he opts to set up his compulsory counterclaim, he may still plead his ground for dismissal as an affirmative
defense in his answer. The latter option is obviously more favorable to the defendant although such fact was lost
on Forbes Park.

VINLUAN,  ALEXANDER  MARA  J.  2C  UST  LAW  


Natividad Lim vs. National Power Corporation, and Spouses Roberto Ll. Arcinue and Arabela Arcinue

FACTS:

NPC filed an expropriation suit against petitioner petitioner before the RTC of Lingayen, covering Lots 2373
and 2374 that the NPC needed for its Sual Coal-Fired Thermal Power Project. Since Lim was residing in the
United States, the court caused the service of summons on her through her tenant, a certain Wilfredo
Tabongbong.

RTC - ordered the issued writ of possession in NPC’s favor, however, Lim, represented by her husband Delfin,
filed an omnibus motion to dismiss the action and to suspend the writ of possession, questioning the RTC’s
jurisdiction over Lim’s person and the nature of the action. Respondent spouses iled a motion for leave to admit
complaint in intervention, alleging that they owned and were in possession of Lot 2374, one of the two lots
subject of the expropriation. The RTC granted the Arcinues’ motion and required both the NPC and Lim to
answer the complaint-in-intervention within 10 days from receipt of its order.

The RTC issued an order of default against both Lim and the NPC. The RTC pointed out that the Arcinues’
failure to explain their resort to service by registered mail had already been cured by the manifestation of Lim’s
counsel that he received a copy of the Arcinues’ motion 10 days before its scheduled hearing.

CA - Lim to file a petition for certiorari; rendered a decision that affirmed the RTC’s order of default. Lim filed
a motion for reconsideration but the CA denied it, prompting her to file the present petition for review.

ISSUES:

WON THERE IS A VALID ORDER OF DEFAULT OF THE RTC ENTERED AGAINST LIM
WON THERE IS A GRAVE ABUSE OF DISCRETION BY THE RTC IN ALLOWING RESPONDENT
FAILURE TO EXPLAIN IN THEIR MOTION WHY THEY SERVED A COPY OF IT ON THE
ADVERSE PARTY BY REGISTERED MAIL RATHER THAN BY PERSONAL SERVICE.

HELD:
1. NO. Section 4, Rule 19 of the Rules of Civil Procedure requires the original parties to file an answer to the
complaint-in-intervention within 15 days from notice of the order admitting the same, unless a different period
is fixed by the court. This changes the procedure under the former rule where such an answer was regarded as
optional. Thus, Lim’s failure to file the required answer can give rise to default. She remained unable to show
that her failure to file the required answer was due to fraud, accident, mistake, or excusable negligence. And,
although she claimed that she had a meritorious defense, she was unable to specify what constituted such
defense.

2. NO. The fact is that Lim's counsel expressly admitted having received a copy of the Arcinues' motion for
judgment 10 days before its scheduled hearing. This means that the Arcinues were diligent enough to file their
motion by registered mail long before the scheduled hearing. Personal service is required precisely because it
often happens that hearings do not push through because, while a copy of the motion may have been served by
registered mail before the date of the hearing, the adverse party receives such already after the hearing. Thus,
the rules prefer personal service. But it does not altogether prohibit service by registered mail when such
service, when adopted, ensures as in this case receipt by the adverse party.

VINLUAN,  ALEXANDER  MARA  J.  2C  UST  LAW  


MAGDIWANG REALTY CORPORATION, RENATO P. DRAGON and ESPERANZA TOLENTINO,
Petitioners, vs.
THE MANILA BANKING CORPORATION, substituted by FIRST SOVEREIGN ASSET
MANAGEMENT (SPV-AMC), INC., Respondent.
G.R. No. 195592 September 5, 2012

FACTS:

The case involves a complaint for sum of money filed before the RTC by herein respondent, against herein
petitioners, after said petitioners allegedly defaulted in the payment of their debts under the five promissory
notes they executed in favor of TMBC. All promissory notes included stipulations on the payment of interest
and additional charges in case of default by the debtors. Despite several demands for payment made by TMBC,
the petitioners allegedly failed to heed to the bank’s demands, prompting the filing of the complaint for sum of
money. Instead of filing a responsive pleading, the petitioners filed a Motion for Leave to Admit Attached
Motion to Dismiss and a Motion to Dismiss, raising therein the issues of novation, lack of cause of action
against individuals Dragon and Tolentino, and the impossibility of the novated contract due to a subsequent act
of the Congress.

RTC ordered the petitioners in default. The petitioners’ motion for the trial court in its Order denied
reconsideration dated August 2, 2005. The ex parte presentation of evidence by the bank before the trial court’s
Presiding Judge was scheduled in the same Order. Unsatisfied with the RTC orders, the petitioners filed with
the CA a petition for certiorari. CA Ruling on RTC’s Order of Default – Affirmed RTC’s order of default,
holding that the RTC did not commit grave abuse of discretion when it declared herein petitioners in default.

RTC - TMBC’s presentation of evidence ex parte proceeded in which it favored the TMBC.

CA - Dismissed the petitioner’s appeal and AFFIRMED RTC’s decision.

ISSUE:
WON THE PRESCRIPTIVE PERIOD WAS LEGALLY INTERRUPTED WHEN PETITIONERS
PROPOSED RESTRUCTURING OF THEIR LOANS

HELD: YES

The ten (10)-year prescriptive period to file an action based on the subject promissory notes was interrupted by
the several letters exchanged between the parties. This is in conformity with the second and third circumstances
under Article 1155 of the New Civil Code (NCC) which provides that the prescription of actions is interrupted
when: (1) they are filed before the court; (2) there is a written extrajudicial demand by the creditors; and (3)
there is any written acknowledgment of the debt by the debtor. In TMBC’s complaint against the petitioners, the
bank sufficiently made the allegations on its service and the petitioners’ receipt of the subject demand letters,
even attaching thereto copies thereof for the trial court’s consideration. During the bank’s presentation of
evidence ex parte, the testimony of witness Mr. Megdonio Isanan was also offered to further support the claim
on the demand made by the bank upon the petitioners. In the absence of a timely objection from the petitioners
on these claims, no error can be imputed on the part of the trial court, and even the appellate court, in taking due
consideration thereof.

VINLUAN,  ALEXANDER  MARA  J.  2C  UST  LAW  


Philippine Tourism Authority vs. Philippine Golf Development & Equiptment; GR no. 176628; March
19, 2012

FACTS:
PTA, an agency of the Department of Tourism, whose main function is to bolster and promote tourism, entered
into a contract with Atlantic Erectors, Inc. (AEI) for the construction of the Intramuros Golf Course Expansion
Projects for a contract price of P57,954,647.94. The civil works of the project commenced. Since AEI was
incapable of constructing the golf course aspect of the project, it entered into a sub-contract agreement with
PHILGOLF, a duly organized domestic corporation, to build the golf course amounting to P27,000,000.00. The
sub-contract agreement also provides that PHILGOLF shall submit its progress billings directly to PTA and, in
turn, PTA shall directly pay PHILGOLF. PHILGOLF filed a collection suit against PTA amounting to
P11,820,550.53, plus interest, for the construction of the golf course. Within the period to file a responsive
pleading, PTA filed a motion for extension of time to file an answer.

RTC - grantied two successive motions for extension of time, PTA failed to answer the complaint. Thus, the
RTC rendered a judgment of default ruling in favor of the defendant, which ordered the defendant to pay the
plaintiff the said outstanding obligation plus interest of 12% per annum, attorney’s fees, cost of litigation, moral
damages, nominal damages and exemplary damages.

CA - PTA filed a petition for certiorari with the CA, imputing grave abuse of discretion on the part of the RTC
for granting the motion for execution pending appeal. The CA ruled in favor of PTA and set aside the order
granting the motion for execution pending appeal.

PTA withdrew its appeal of the RTC decision and, instead, filed a petition for annulment of judgment under
Rule 47 of the Rules of Court. The petition for annulment of judgment was premised on the argument that the
gross negligence of PTAs counsel prevented the presentation of evidence before the RTC. CA dismissed the
petition for annulment of judgment for lack of merit. PTA questions this CA action in the present petition for
certiorari.

ISSUES:

WON THE NEGLIGENCE OF THE PTAs COUNSEL AMOUNTER TO AN EXTRINSIC FRAUD


WARRANTING AN ANNULMENT OF JUDGMENT.
WON THERE A PETITION FOR ANNULMENT OF JUDGMENT IS A PROPER REMEDY.

HELD:

1. NO. Extrinsic fraud refers to any fraudulent act of the prevailing party in the litigation which is committed
outside of the trial of the case, whereby the unsuccessful party has been prevented from exhibiting fully his
case, by fraud or deception practiced on him by his opponent.1[9] Under the doctrine of this cited case, we do
not see the acts of PTAs counsel to be constitutive of extrinsic fraud. The records reveal that the judgment of
default was sent via registered mail to PTAs counsel. However, PTA never availed of the remedy of a motion to
lift the order of default. Since the failure of PTA to present its evidence was not a product of any fraudulent acts
committed outside trial, the RTC did not err in declaring PTA in default.

2. NO. Annulment of Judgment is not the proper remedy. PTAs appropriate remedy was only to appeal the RTC
decision. Annulment of Judgment under Rule 47 of the Rules of Court is a recourse equitable in character and
                                                                                                                         
 

VINLUAN,  ALEXANDER  MARA  J.  2C  UST  LAW  


allowed only in exceptional cases where the ordinary remedies of new trial, appeal, petition for relief or other
appropriate remedies are no longer available through no fault of petitioner. In this case, appeal was an available
remedy. There was also no extraordinary reason for a petition for annulment of judgment, nor was there any
adequate explanation on why the remedy for new trial or petition for relief could not be used. The Court is
actually at a loss why PTA had withdrawn a properly filed appeal and substituted it with another petition, when
PTA could have merely raised the same issues through an ordinary appeal.

VINLUAN,  ALEXANDER  MARA  J.  2C  UST  LAW  


LETICIA DIONA, represented by her Attorney-in-Fact, MARCELINA DIONA, vs.
ROMEO A. BALANGUE, SONNY A. BALANGUE, REYNALDO A. BALANGUE, and ESTEBAN A.
BALANGUE, JR., G.R. No. 173559, January 7, 2013

FACTS:

Respondents obtained a loan of P45,000.00 from petitioner payable in six months and secured by a Real Estate
Mortgage over their property located in Marulas, Valenzuela. When the debt became due, respondents failed to
pay notwithstanding demand.

RTC - petitioner filed a Complaint praying that respondents be ordered to pay the principal obligation, the
damages, attorney’s fees, and to issue a decree of foreclosure for the sale at public auction of the
aforementioned parcel of land. Respondents were served with summons thru respondent Sonny A. Balangue
(Sonny). With the assistance of Atty. Coroza of the PAO, they filed a Motion to Extend Period to Answer.
Despite the requested extension, however, respondents failed to file any responsive pleadings. Thus, upon
motion of the petitioner, the RTC declared them in default and allowed petitioner to present her evidence ex
parte. RTC decided in favour of the petitioner, ordering the respondents to pay the petitioner. Then, petitioner
filed a Motion for Execution but was interposed by the respondent wherein they filed a Motion to Set Aside
Judgment. RTC ordered the issuance of a Writ of Execution. In order to satisfy the writ, petitioner moved for
the public auction of the mortgaged property, which the RTC granted.

CA - Petition for Certiorari under Rule 65 of the Rules of Court. CA declared that the RTC exceeded its
jurisdiction in awarding the 5% monthly interest but at the same time pronouncing that the RTC gravely abused
its discretion in subsequently reducing the rate of interest to 12% per annum. Furthermore, the court held that
the proper remedy is not to amend the judgment but to declare that portion as a nullity. Void judgment for want
of jurisdiction is no judgment at all, it cannot be the source of any right nor the creator of any obligation, and no
legal rights can emanate from it. CA annulled the said order. Respondents filed with the CA a Petition for
Annulment of Judgment and Execution Sale with Damages, contending that the portion of the RTC Decision
granting petitioner 5% monthly interest rate is in gross violation of Section 3(d) of Rule 9 of the Rules of Court
and of their right to due process. Initially, the CA denied due course to the Petition. Thus, it ruled in favor of
the respondnents.

ISSUE:

WON THE CA ERRED WHEN IT GRANTED RESPONDENTS’ PETITION FOR ANNULMENT OF


JUDGMENT OF THE DECISION OF THE RTC DESPITE THE FACT THAT THE SAID DECISION
HAS BECOME FINAL AND ALREADY EXECUTORY CONTRARY TO THE DOCTRIN OF
IMMUTABILITY OF JUDGMENT.

HELD: NO.

A Petition for Annulment of Judgment under Rule 47 of the Rules of Court is a remedy granted only under
exceptional circumstances where a party, without fault on his part, has failed to avail of the ordinary remedies
of new trial, appeal, petition for relief or other appropriate remedies. Said rule explicitly provides that it is not
available as a substitute for a remedy which was lost due to the party’s own neglect in promptly availing of the
same.

While under Section 2, Rule 47 of the Rules of Court a Petition for Annulment of Judgment may be based only
on the grounds of extrinsic fraud and lack of jurisdiction, jurisprudence recognizes lack of due process as
additional ground to annul a judgment. Grant of 5% monthly interest is way beyond the 12% per annum interest
sought in the Complaint and smacks of violation of due process. It is settled that courts cannot grant a relief not
VINLUAN,  ALEXANDER  MARA  J.  2C  UST  LAW  
prayed for in the pleadings or in excess of what is being sought by the party. Due process considerations require
that judgments must conform to and be supported by the pleadings and evidence presented in court. It is
improper to enter an order which exceeds the scope of relief sought by the pleadings, absent notice which
affords the opposing party an opportunity to be heard with respect to the proposed relief. The fundamental
purpose of the requirement that allegations of a complaint must provide the measure of recovery is to prevent
surprise to the defendant. The reason behind Section 3(d), Rule 9 of the Rules of Court is to safeguard
defendant’s right to due process against unforeseen and arbitrarily issued judgment. This is akin to the very
essence of due process. It embodies "the sporting idea of fair play" and forbids the grant of relief on matters
where the defendant was not given the opportunity to be heard thereon.

VINLUAN,  ALEXANDER  MARA  J.  2C  UST  LAW  


MANCHESTER DEVELOPMENT CORPORATION
vs.
COURT OF APPEALS, CITY LAND DEVELOPMENT CORPORATION,

FACTS:

This was originally a case of an action for torts and damages and specific performance with a prayer for
temporary restraining order. The damages were not specifically stated in the prayer but the body of the
complaint assessed a P 78.75M. damages suffered by the petitioner. The amount of docket fee paid was only
P41O.OO. The petitioner then amended the complaint and reduced the damages to P1O M only.

ISSUES:

WON AN AMENDED COMPLAINT VESTS JURISDICTION IN THE COURT.

HELD: NO.

The docket fee, its computation, should be based on the original complaint. A case is deemed filed only upon
payment of the appropriate docket fee regardless of the actual date of filing in court. Here, since the proper
docket fee was not paid for the original complaint, it’s as if there is no complaint to speak of. As a consequence,
there is no original complaint duly filed which can be amended. So, any subsequent proceeding taken in
consideration of the amended complaint is void. Manchester’s defense that this case is primarily an action for
specific performance is not merited. The Supreme Court ruled that based on the allegations and the prayer of the
complaint, this case is an action for damages and for specific performance. Hence, it is capable of pecuniary
estimation.

Further, the amount for damages in the original complaint was already provided in the body of the complaint.
Its omission in the PRAYER clearly constitutes an attempt to evade the payment of the proper filing fees. To
stop the happenstance of similar irregularities in the future, the Supreme Court ruled that from this case on, all
complaints, petitions, answers and other similar pleadings should specify the amount of damages being prayed
for not only in the body of the pleading but also in the prayer, and said damages shall be considered in the
assessment of the filing fees in any case. Any pleading that fails to comply with this requirement shall not bib
accepted nor admitted, or shall otherwise be expunged from the record

VINLUAN,  ALEXANDER  MARA  J.  2C  UST  LAW  


SUN INSURANCE OFFICE V. HON. MAXIMIANO C. ASUNCION, Presiding Judge and MANUEL
CHUA UY PO TIONG.

FACTS:

Petitioner Sun Insurance Office filed a complaint for the consignation of a premium refund on a fire insurance
policy with a prayer for the judicial declaration of its nullity against private respondent Manuel Uy Po Tiong.
Private respondent was declared in default for failure to file the required answer within the reglementary period.
On the other hand, private respondent filed a complaint for the refund of premiums and the issuance of a writ of
preliminary attachment against petitioner. Although the prayer in the complaint did not quantify the amount of
damages sought said amount may be inferred from the body of the complaint to be about Fifty Million Pesos.
Only the amount of P21O.OO was paid by private respondent as docket fee which prompted petitioners' counsel
to raise his objection, which was disregarded by respondent Judge. The Court thereafter returned the said
records to the trial court with the directive that they be re-raffled to the other judges to the exclusion of Judge
Castro. The Court issued a Resolution directing the judges to reassess the docket fees and requires all clerks of
court to issue certificates of re-assessment of docket fees. All litigants were likewise required to specify in their
pleadings the amount sought to be recovered. Judge Maximiano Asuncion, to whom Civil Case was thereafter
assigned, issued an Order requiring the parties in the case to comment on the Clerk of Court's letter-report.
Petitioners then filed a petition for certiorari with the Court of Appeals questioning the said order of Judge
Asuncion. Court of Appeals rendered a decision ruling, among others, Denying due course to the petition
insofar as it seeks annulment of the order

ISSUE:

WON A COURT ACQUIRES JURISDICTION OVER A CASE WHEN THE CORRECT AND
PROPER DOCKET FEE HAS NOT BEEN PAID.

HELD: YES

It is not simply the filing of the complaint or appropriate initiatory pleading, but the payment of the prescribed
docket fee, that vests a trial court with jurisdiction over the subject matter or nature of the action. Where the
filing of the initiatory pleading is not accompanied by payment of the docket fee, the court may allow payment
of the fee within a reasonable time but in no case beyond the applicable prescriptive or reglementary period. It
shall be the responsibility of the Clerk of Court or his duly authorized deputy to enforce said lien and assess and
collect the additional fee. The petition is DISMISSED for lack of merit.

The Clerk of Court of the court a quo is instructed to reassess and determine the additional filing fee that should
be paid by private respondent considering the total amount of the claim sought in the original complaint and the
supplemental complaint as may be gleaned from the allegations and the prayer thereof and to require private
respondent to pay the deficiency.

VINLUAN,  ALEXANDER  MARA  J.  2C  UST  LAW  


Ballatan v. CA, 304 SCRA 34
FACTS:

Eden Ballatan, together with other petitioners, is living in and registered owners of Lot No. 24. Respondent
Winston Go is living in and registered owners of Lot No. 25 and 26. And Li Ching Yao is living in and the
registered owner of Lot. 27. The Lots are adjacent to each other. When Ballatan constructed her house in her
lot, she noticed that the concrete fence and side pathway of the adjoining house of respondent Winston Go
encroached on the entire length of the eastern side of her property. Her contractor of this discrepancy, who then
told respondent Go of the same, informed her. Respondent, however, claims that his house was built within the
parameters of his father’s lot; and that this lot was surveyed by engineer Jose Quedding of the Araneta Institute
of Agriculture. Ballatan made written demands to the respondent to remove their improvements and since the
latter wasn’t answering the petitioner filed accion publiciana in court. Go’s filed their “Answer with Third-Party
Complaint” impleading as third party defendants respondents Li Ching Yaoand including Engineer Quedding as
well as the Araneta Institute of Agriculture.

RTC - ruled in favor of the petitioner-ordering respondent Go to demolish their improvements and pay damages
to Petitioner but dismissing the third-party complaint.

CA - affirmed the dismissal of the third party-complaint as to Araneta Institute of Agriculture but reinstated the
the complaint against Yao and the Engineer. CA also affirmed the demolition and damages awarded to
petitioner and added that Yao should also pay respondent for his encroachment of respondent Go’s property.
Jose Quedding was also ordered to pay attorney’s fees for his negligence.

ISSUE:

WON THE CA ERRED IN NOT DISMISSING THE THIRD-PARTY COMPLAINT DUE TO NON-
PAYMENT OF ANY FILING OR DOCKET FEES.

HELD: NO

The appellate court correctly dismissed the third-party complaint against Araneta Institute of Agriculture. The
claim that the discrepancy in the lot areas was due to Araneta Institute of Agriculture's fault was not
proved. The appellate court, however, found that it was the erroneous survey by Engineer Quedding that
triggered these discrepancies. And it was this survey that respondent Winston Go relied upon in constructing his
house on his father's land. He built his house in the belief that it was entirely within the parameters of his
father's land. In short, respondents Go had no knowledge that they encroached on petitioners' lot. They are
deemed builders in good faith until the time petitioner Ballatan informed them of their encroachment on her
property.

VINLUAN,  ALEXANDER  MARA  J.  2C  UST  LAW  


SPS. GREGORIO GO AND JUANA TAN GO V. JOHNSON Y. TONG, G.R. NO. 151942, NOVEMBER
27, 2003

FACTS:

The petitioner purchased a cashier’s check from the Far East Bank and Trust Company (FEBTC) Lavezares,
Binondo Branch in the amount of P500,000.00, payable to the respondent. On petitioner Juanas’ instruction, the
cashier’s check bore the words ‘Final Payment/Quitclaim’ after the name of payee respondent allegedly to
insure that respondent would honor his commitment that he would no longer ask for further payments for his
interest in the informal business partnership which he and she had earlier dissolved. Respondents counsel
subsequently wrote to the manager of FEBTC Lavezares Branch informing that the words Final
Payment/Quitclaim on the check had been unintentionally and inadvertently erased without being initialed by
the bank or the purchaser thereof and thus requesting that the check be replaced with another payable to
‘Johnson Tong-Final Settlement/Quitclaim’ with the same amount, the bank charges therefor to be paid by his
client-respondent. FEBTC did not grant the request of respondent’s counsel.

RTC Manila - respondent filed a complaint against FEBTC and petitioner Juana and her husband Gregorio Go
for sum of money, damages, and attorney’s fees.

Manila Prosecutors Office - petitioners’ son, filed a criminal complaint against respondent for falsification of
the check. The criminal complaint was, however, subsequently dismissed.

Respondent filed a Motion for Leave to File a Supplemental Complaint and to Admit the Attached
Supplemental Complaint which Supplemental Complaint alleged that petitioners used their son to file the
criminal complaint for falsification against him which caused damages, hence, prayed for an increase in the
amount of moral and exemplary damages sought to be recovered from P2.5 million to P55 million and prayed
for the award of actual damages of P58,075.00. The motion was set for hearing. Copy of the motion to
petitioners was sent by registered mail.

Public Respondent granted the motion and admitted the Supplemental Complaint noting that petitioners had
been furnished copy of the Motion for Leave but that there had been no comment thereon.

CA - Petition for Certiorari before the CA, alleged that respondent judge committed grave abuse of discretion
when he issued the Orders allowing the release of the P500,000.00 deposit to respondent, allowing the payment,
on staggered basis, of the docket fees for the Supplemental Complaint and denying the Motion for
Reconsideration of the Order. CA that they questioned the Order. Moreover, the appellate court held that,
anyway, respondent was entitled to the deposit, which represented the amount indicated on the check that
belonged to him. CA held that payment of the prescribed docket fee within a reasonable period is permitted but
in no case beyond the applicable prescriptive or regular period. In that case, the court a quo opined that the
docket fee payment scheme imposed by the respondent judge cannot be said to have been issued with grave
abuse of discretion.

ISSUES:

WON THE PETITION FOR CERTIORARI UNDER RULE 65 WAS PROPER.


WON THE RESPONDENT JUDGE AND THE CA ERRED IN ALLOWING RESPONDENT TO PAY THE
DOCKET FEE ON A STAGGERED BASIS.

VINLUAN,  ALEXANDER  MARA  J.  2C  UST  LAW  


HELD:

1. NO. The proper remedy should be based on Rule 45 as what is assailed is an error of judgment of the CA and
not that of grave abuse of discretion amounting to lack or excess jurisdiction. When an error of judgment of the
CA is brought up to this Court for review, the action is properly designated as a petition for review and not a
special civil action. Thus, while the instant Petition is one for certiorari under Rule 65 of the Rules of Court, the
assigned errors are properly addressed in a petition for review under Rule 45. Accordingly, when parties adopt
an improper remedy, their petitions may be dismissed outright. However, the Supreme Court is empowered by
the Constitution, in the interest of substantial justice, to provide leniency on procedural technicalities in order to
rule speedily on cases and demonstrate that even without the procedural infirmity, the Petition should be
rejected due to its lack of merits.

2. NO. Plainly, while the payment of the prescribed docket fee is a jurisdictional requirement, even its
nonpayment at the time of filing does not automatically cause the dismissal of the case, as long as the fee is paid
within the applicable prescriptive or reglementary period; more so when the party involved demonstrates a
willingness to abide by the rules prescribing such payment. As a rule, docket fees should be paid upon the filing
of the initiatory pleadings. However, for cogent reasons to be determined by the trial judge, staggered payment
thereof within a reasonable period may be allowed. Unless grave abuse of discretion is demonstrated, the
discretion of the trial judge in granting staggered payment shall not be disturbed.

VINLUAN,  ALEXANDER  MARA  J.  2C  UST  LAW  


NATIONAL STEEL CORPORATION, vs. COURT OF APPEALS, HON. ARSENIO J. MAGPALE, and
JOSE MA. P. JACINTO, G.R. No. 123215. February 2, 1999

FACTS:

Disagreement between petitioner National Steel Corporation and the respondent NSC-HDTC Monthly/Daily
Employees Organization-FFW (Union) arose regarding the giving of year-end incentives and Productivity and
Quality Bonus to the employees of NSC. Having agreed on the submission of the case for voluntary arbitration,
the parties appeared before the voluntary arbitrator, Rene Ofreneo, who found no merit in the demand of the
Union for a productivity and quality bonus, but ordered NSC to distribute the year-end award, as according to
the Collective Bargaining Agreement and such practice having been done for four years by petitioner. Upon this
decision, petitioner filed a Partial Motion for Reconsideration regarding the award of the year-end incentive,
which Motion was denied by Ofreneo. Petitioner filed a petition for review with the Court of Appeals.

CA- petitioner’s petition on the ground of failure to comply with the requirements of law regarding the law on
forum shopping. It held that Atty. Padilla, one of the counsels of record of petitioner was a mere retained
counsel, who had no direct interest with the outcome of the suit. As such, said the CA, Atty. Padilla had no
authority under the law to sign the certification in behalf of the petitioner.

ISSUE:

WON THE PETITION FOR REVIEW SHOULD BE DENIED.

HELD: YES

In the instant case, the certification was signed by NSC’s counsel. NSC argues that contrary to the findings of
the Court of Appeals, NSC’s counsel of record was duly authorized to represent them not only before the
Voluntary Arbitrator but also to prepare the petition for review filed before the Court of Appeals. To support
this claim, petitioner attached to its petition before this Court a Secretary’s Certificate. Counsel of petitioner,
Atty. Padilla also submitted a Verification cum Certification where he stated that he prepared the petition upon
the explicit instructions of the VP-Marketing & Resident Manager of petitioner corporation. While it is admitted
that the authorization of petitioner’s counsel was submitted to the appellate court only after the issuance of its
Resolution dismissing the petition based on non-compliance with the Circular regarding forum-shopping, the
Supreme Court holds that in view of the peculiar circumstances of the present case and in the interest of
substantial justice, the procedural defect may be set aside, pro hac vice. As held by the Court: "Technical rules
of procedure should be used to promote, not frustrate, justice. While the swift unclogging of court dockets is a
laudable objective, the granting of substantial justice is an even more urgent ideal." By recognizing the
signature of the authorized counsel in the certification, no circumvention of the rationale, that is to prevent the
ills of forum shopping, is committed.

VINLUAN,  ALEXANDER  MARA  J.  2C  UST  LAW  

También podría gustarte